13.07.2015 Views

Revista (format .pdf, 1.2 MB) - RECREAÅ¢II MATEMATICE

Revista (format .pdf, 1.2 MB) - RECREAÅ¢II MATEMATICE

Revista (format .pdf, 1.2 MB) - RECREAÅ¢II MATEMATICE

SHOW MORE
SHOW LESS
  • No tags were found...

Create successful ePaper yourself

Turn your PDF publications into a flip-book with our unique Google optimized e-Paper software.

Anul VIII, Nr. 1Ianuarie – Iunie 2006RECREAŢ II<strong>MATEMATICE</strong>REVISTĂ DE MATEMATICĂ PENTRU ELEVI Ş I PROFESORIe iπ =−1Editura “Recreaţ ii Matematice”IAŞ I - 2006


Semnificaţia formulei de pe copertă:iπÎntr-o formă concisă, formula e = −1 leagă cele patru ramuri fundamentaleale matematicii:ARITMETICA reprezentată de 1GEOMETRIAreprezentată de πALGEBRAreprezentată de iANALIZA MATEMATICĂreprezentată de eRedacţia revistei :Petru ASAFTEI, Dumitru BĂTINEŢU-GIURGIU (Bucureşti), Temistocle BÎRSAN, DanBRÂNZEI, Cătălin - Cristian BUDEANU, Constantin CHIRILĂ, Eugenia COHAL, AdrianCORDUNEANU, Mihai CRĂCIUN (Paşcani), Gabriel DOSPINESCU (student, Paris),Marius FARCAŞ, Paraschiva GALIA, Paul GEORGESCU, Mihai HAIVAS, GheorgheIUREA, Lucian Georges LĂDUNCĂ, Mircea LUPAN, Dan Ştefan MARINESCU(Hunedoara), Gabriel MÎRŞANU, Andrei NEDELCU, Gabriel POPA, Dan POPESCU(Suceava), Florin POPOVICI (Braşov), Maria RACU, Ioan SĂCĂLEANU (Hârlău), IoanŞERDEAN (Orăştie), Dan TIBA (Bucureşti), Adrian ZAHARIUC (Bacău), AdrianZANOSCHI.Adresa redacţiei:Catedra de Matematică – Universitatea Tehnică “Gh. Asachi” IaşiBd. Carol I, nr.11, 700506, IaşiTel. 032 – 213737 / int. 123E-mail: recreatii.matematice@gmail.comhttp://www.recreatiimatematice.uv.roCOPYRIGHT © 2006, ASOCIAŢIA “RECREAŢII <strong>MATEMATICE</strong>”Toate drepturile aparţin Asociaţiei “Recraţii Matematice”. Reproducerea integrală sauparţială a textului sau a ilustraţiilor din această revistă este posibilă numai cu acordul prealabilscris al acesteia.TIPĂRITĂ LA SL&F IMPEX IAŞIBd. Carol I, nr. 3-5Tel. 0788 498933E-mail: simonaslf@yahoo.com


Elogiu adus revistei "Gazeta Matematică"la 110 ani de apariţie neîntreruptăCum podul de la Cernavodă îşi întinde braţele peste apele Dunării, aşa şi GazetaMatematică îşi întinde existenţa, care are începutul în secolul al XIX-lea, pesteîntreg secolul al XX-lea şi continuă să-şi aducă aportul la dezvoltarea învăţământuluişi ştiinţelor matematice din ţara noastră şi în acest nou secol, al XXI-lea.Gazeta Matematică, prin cele 110 tomuri durate în timp unul după altul,estepiramida Keops a publicisticii periodice româneşti, punctul de maxim absolut alacesteia. Este a doua publicaţiedematematică din lume ce se adresează tineretuluişi prima de acest fel în privinţa apariţiei neîntrerupte şi longevităţii.Aavutoexistenţă zbuciumată şi cu multe momente dramatice, existenţă strânslegată de soarta învăţământului din şcolile româneşti - cel matematic mai ales - dar şide cea a poporului român. A trecut prin două războaie mondiale, reforme neinspirateale învăţământului public, regimuri adverse poporului român etc. Să amintim doarun singur episod din existenţa Gazetei. Înprimulrăzboi mondial, în urma ocupăriicapitalei, Iaşul devine centrul politic şi administrativ al ţării; tot aici se refugiază şiGazeta Matematică, care, datorită devotamentului şi strădaniilor lui Traian Lalescu,Vasile Teodoreanu şi altor membri din redacţie, continuă săfietipărită şi expediată,ajungând chiar şi în mâinele abonaţilor aflaţi în primele linii ale frontului.Sub deviza Entuziasm, armonie, muncă dezinteresată, sacrificii continue au fostdepăşite toate greutăţile materiale şi vicisitudinile vremurilor; oameni minunaţi strânşiîn jurul şi pătrunşi de spiritul Gazetei Matematice, au asigurat prin munca şi bărbăţialor mersul ei înainte. În prima jumătate de veac, la cârma destinelor revisteiau fost "cei 4 stâlpi ai Gazetei Matematice": Ion Ionescu, Gheorghe Ţiţeica, AndreiIoachimescu şi Vasile Cristescu. Merită elogiată, de asemenea, contribuţia lui NicolaeTeodorescu, care a condus cu multă competenţă, în ultimul sfert al secolului trecut,activităţile la Gazetă şi cele legate de ea, devenite între timp mult mai diverse.Tinerii talentaţi din generaţii succesive au "frecventat" şcoala Gazetei Matematice,undeşi-au <strong>format</strong> deprinderile şi tehnicile de lucru, şi-au şlefuit raţionamentulmatematic sau şi-au văzut publicate primele încercări originale în paginile acesteia.Nume de viitori iluştri matematicieni români se găsesc menţionate în paginileGazetei Matematice printre rezolvitorii şi propunătorii de probleme, premianţiiunor concursuri sau ca autori de note originale interesante, iar unii dintre ei asumându-şiresponsabilităţi redacţionale. Lista lor fiind prea lungă, amintim doar câteva nume:Gh. Ţiţeica, C. Popovici, T. Lalescu, N. Abramescu, O. Mayer, Al. Pantazi, D.Barbilian, Fl. Vasilescu, N. Ciorănescu, T. Popoviciu, Gr. Moisil, N. Teodorescu etc.În aceşti 110 ani de existenţă, Gazeta Matematică adevenitocariatidăaculturiiromâneşti, simbol al permanenţei şi continuităţii, componentă aînvăţământuluimatematic din ţara noastră, pepinieră de talente matematice.Caorecunoaştere a meritelor sale, Preşedintele României, prin decretul 988 din 7octombrie 2005, conferă Gazetei Matematice Ordinul "Meritul Cultural" în gradulde Ofiţer, categoria H "cercetare ştiinţifică", pentrucontribuţia deosebită lapromovareaînvăţământului şi cercetării aprofundate a ştiinţelor matematice.Prof. dr. Temistocle BÎRSAN1


100 de ani de la naşterea matematicianuluiGrigore C. MoisilMulte capitole ale matematicii mi-au fostdragi. Matematica e una (Gr.C.Moisil)În Istoria matematicii în România, GeorgeŞt. Andonie îl prezintă peGrigore C. Moisil ca "ofericită întruchipare a dominantelor matematicii noastre:dinamism, varietate, tendinţa spre universalitate".Afostunuldintreceimaimarişi talentaţi matematicieniromâni şi, indiscutabil, cel mai prolific. Atacândcu succes aproape toate domeniile matematiciipure şi aplicate s-a dovedit un creator în continuă înnoire.Jovial şioptimist,cuunumorplindesevă, eraun povestitor fermecător.Opera lui Grigore C. Moisil nu este doar creaţiaindividuală aunuiomdeosebitdeînzestrat. Easebazează petradiţia "dinastiei" Moisileştilor, pe sprijinulpermanent al familiei care a stimulat şi sprijinitinteligenţa sa sclipitoare. S-a mândrit întotdeauna căse trage dintr-o familie de grăniceri năsăudeni. Familia sa, originară din Maramureş,a descins în comuna Şanţ, în imediata vecinătate a Năsăudului. Străbunicul matematicianului,care purta numele de Grigore, a fost primul cărturar care s-a ridicat dincomuna Şanţ. A fost preot, profesor şi primul director al celui de al patrulea liceuromânesc înfiinţat în Austria, la Năsăud. Fiul acestuia, Constantin, a obţinut titlulde doctor în ştiinţe filologice la Universitatea din Viena şi a funcţionat 32 de ani caprofesor la Năsăud. Unul dintre fii acestuia, numit tot Constantin, tatăl matematicianului,a urmat, cu sprijinul lui Al. Odobescu, şcoala Normală Superioară dinBucureşti; a funcţionat ca profesor la Focşani, Tulcea şi Bucureşti. Ulterior, a părăsitprofesoratul consacrându-se arheologiei şi numismaticii, devenind un reputat specialistîn acest domeniu şi membru al Academiei Române. La Tulcea s-a căsătorit cuinstitutoarea Elena Nicolescu, care a devenit ulterior directoarea Şcolii "EnăchiţăVăcărescu" din Bucureşti. La Tulcea s-au născut primii trei copii ai familiei; Grigore(10 ianuarie 1906), Florica (cercetătoare la Biblioteca Academiei; căsătorită cuacad.Emil Condurache) (1909) şi Ioan (1910); ultimul copil, Gheorghe, s-a născut în 1917la Vaslui (în timpul refugiului). Ambii fraţi au fost ingineri, profesori universitari.Dotată cu o inteligenţă vieşi un umor sănătos, mama, Elena, a avut un rol decisivîn formarea lui Grigore, care i-a urmat cu sfinţenie sfaturile, în întreaga sa viaţă. Dela ea a moştenit deviza: "Nu crede tot ce ţi se spune, judecă tusingur".Grigore C. Moisil aurmatşcoala primară în Bucureşti şi liceul la "Mihail Kogălniceanu"din Vaslui (1916 — 1918) şi "Spiru Haret" din Bucureşti (1918 — 1923). S-aînscris apoi la secţia de matematică delaFacultateadeŞtiinţe a Universităţii dinBucureşti, unde a a vut ca profesori pe D. Pompeiu, Gh. Ţiteica, A. Davidoglu,Tr. Lalescu. Primul i-a fost mentor nu numai în matematică, ci şi în anumite regulide viaţă. Într-un articol ("Viaţa studenţească", nr. 11, 1967) mărturiseşte: "For-2


marea generaţiei matematice din care fac parte coincide cu începuturile matematiciiabstracte româneşti. Generaţiameaapăşit cu dreptul. Ea a profitat de faptul de afi avut ca profesori oameni de ştiinţă şi ai căror profesori şi ei oameni de ştiinţă".Ca student a participat şi la cursuri de istorie (N. Iorga), filozofie, sociologie (MihailDragomirescu), istoria artelor. A urmat în paralel şi secţia de construcţii de laInstitutul Politehnic Bucureşti la care a renunţat (în anul al IV-lea) când a obţinutdoctoratul şi a plecat în străinătate.A luat doctoratul la 4 iunie 1929 în faţa unei comisii prezidatădeGh. Ţiţeica şi dincare făceau parte Dimitrie Pompeiu şi Anton Davidoglu. În teza de doctorat întitulatăMecanica analitică a sistemelor continue a studiat analitic mecanica sistemelor cuun număr infinit de grade de libertate folosind metoda funcţională (noţiunea defuncţională fusese introdusă cupuţintimpînurmădeVito Volterra). În 1930 pleacă,cu o bursă a ministerului la Paris, unde ia contact cu Jaques Hadamard, Paul Levy,Henri Villat, Paul Montel şi Elie Cartan, care apreciază elogios contribuţiile originaledin teza de doctorat.La 1 iulie 1931 îşi trece docenţa în specialitatea analiză matematică, la Universitateadin Bucureşti. Se întoarce la Paris unde urmează cursul lui Vito Volterra.În toamna anului 1932 se stabileşte la Iaşi fiind numit conferenţiar la Universitatea"Al. I. Cuza". Matematicianul Ion Creangă, fostprofesorşi rector al Universităţii îşiaminteşte: În acel timp eram student în anul al III-lea al secţiei de matematică dela Universitatea din Iaşi; în curând am aflat că lasecţia noastră aînceputuncursde factură modernă predat de un tânăr matematician, deja cu renume <strong>format</strong> şi carerevoluţionează concepţia noastră despre algebră. Am fost atras de acest curs, am începutsă-l audiez şi în curând am fost furat de noutăţile atât de atractive cuprinse înlecţiile cursului. Prelegerile lui Moisil ne-au deschis porţile spre fermecătoarea lumea structurilor algebrice, a laticelor, a împletirii strânse dintre procesele de logică şiabstractizarea teoriei mulţimilor. Perioada de 10 ani petrecuţi în Iaşi a fost de mareimportanţă pentrucreaţia sa ştiinţifică şi pentru desăvârşirea personalităţii sale. Învârstăde26deani,agăsit la Iaşi o atmosferădeînaltăcultură. Peste ani îşi amintea:La Iaşi era o extraordinară densitate de oameni deştepţi pe metrul pătrat. Aiciagăsitmatematicieni de mare valoare ştiinţificăşi spiritualăşi a rămas toatăviaţa prieten cucei care îl primiseră cusimpatielasosireaînIaşi: Alexandru şi Vera Myller, SimionSanielovici, Octav Mayer, Mendel Haimovici, Ilie Popa, Adolf Haimovici şi asistentulsău din acea perioadă Ion Creangă. În Biblioteca Seminarului Matematic din Iaşi agăsit cărţile care aveau să facă din el un matematician modern. Proaspăt titularizatca profesor în 1935, în introducerea primului curs de algebră abstractămodernăţinutîn România, afirmă: La Iaşi am citit multe cărţi de algebră, dar cartea directoarea fost cea a lui B. L. Van der Warden "Moderne Algebra". Era acolo un nou modde a concepe matematica şi anume algebra, dar nu numai algebra; matematica eraconcepută nucaoştiinţă acantităţii, ci ca o ştiinţă a structurii. Peste câţva ani auapărut alte două cărţi care evidenţiau acelaşi mod de a privi matematica: "Topologia"lui Kuratowski şicartealuiSt.Banachasupraspaţiilor care îi poartă numele. Seputea, cu aceste volume şi punând în fruntea lor "Teoria numerelor transfinite" a luiW. Sierpinski, organiza un curs de matematici în înţelesul de studiu al structurilor.Înţelegeam încet, încet că matematica se schimbase. Se schimbă. Se va schimba.3


În Iaşi s-a simţit în largul său, a legat numeroase prietenii, participând cu exuberanţaspecificătinereţii la viaţă acestui oraş pentrucareapăstrat permanent o afecţiunenedesminţită. S-au creat legende în legătură cuviaţa boemă atânărului "răsfăţat"al Iaşului. La restaurantul de lângă vecheaclădire a Academiei Mihăilene s-apăstrat într-un colţ discret,pânălademolarealocalului,omasă cunoscută subnumelede "masa lui Moisil". Se spune că la restaurantul "Corso" din centrul oraşului,îi plăcea să asculte orchestra interpretând un vals a cărui melodie şi versuri eraucompuse chiar de Moisil.Tot la Iaşi s-a petrecut un eveniment care i-a marcat întreaga viaţă şi creaţie.O cunoaştepeVioricaConstantecucaresevacăsători. Viorica Moisil i-a statalături în permanenţă, l-a sprijinit şi stimulat, i-a asigurat calmul şi confortul necesarecreaţiei. După moartea savantului, pe baza scrisorilor şi altor documente de familie,i-a dedicat o carte minunată scrisă cu talent, dragoste şi discreţie "Un om ca oricarealtul. Grigore C. Moisil", apărută în 1979 în editura Albatros.În anul 1942 s-a creat la Facultatea de Matematică aUniversităţii din Bucureşticatedra de analiză superioară şi logică la care este încadrat Grigore C. Moisil. Dupăperioada 1946-1948 când a fost ambasador al României în Turcia, revine la UniversitateaBucureşti unde a predat cursuri de elasticitate, algebră şi maşini de calcul.În 1948 devine mebru activ al Academiei Române şi şeful secţiei de algebră delaInstitutul de Matematică al Academiei, nou înfiinţat. În 1948 este ales preşedinte alSocietăţii Române de Matematică, post pe care îl va ocupa toată viaţa.DupăceGrigore C. Moisil formeazălaBucureşti o veritabilăşcoală de mecanicasolidelor deformabile, începând din 1949 ia naştere în jurul său Şcoala de teorie algebricăa mecanismelor automate. Alături de ruşii V. I. Şestacov şi M. Gavrilov şi americanulShannon este fondatorul acestei teorii, care are la bază utilizarea algebrelorBoole în studiul automatelor. În această direcţie publică douătratate:Teoria algebricăa mecanismelor ordonate şi Teoria algebrică a schemelor cu contacte şi relee.Începând din 1955 călătoreşte foarte mult, fiind invitat la congrese, sesiuni decomunicări, cursuri sau conferinţe. Devine membru al Academiei din Bologna şi alInstitutului Internaţional de Filozofie din Paris.Grigore C. Moisil are lucrări importante în analiza funcţională, mecanica teoretică,geometrie diferenţială şi algebră. Partea cea mai originală dincreaţiasaoconstituie preocupările de logică matematică (începuteînperioadadelaIaşi, unde aţinut şi primele cursuri de logică matematică din România), care l-au condus la consideraţiifilozofice asupra matematicii şi la teoria algebrică a mecanismelor automate.Aceste preocupări i-au asigurat un loc cu totul aparte în matematica românească.S-a stins din viaţă la 21 mai 1973, la Ottawa, în Canada, în timpul unei vizite încare a pus jaloanele colaborării între in<strong>format</strong>icienii candieni şi cei români. La un ande la dispariţia sa, fostul său elev, Mircea Maliţa îl caracteriza: Moisil a fost mai multdecât un savant, a fost mai mulţi savanţi întruniţi în sesiune permanentă sau luânduşilocul unul altuia în cicluri succesive mari, reprezentate de temele fundamentale pecare le-a abordat. A fost până în ultimele zile deschizător de drumuri, inovator. Înaceastă aventură spirituală nu a admis dilentatismul superficial.4Prof. dr. Petru MINUŢ


Asupra problemei 809 din Gazeta Matematică,volumul VIII (1902—1903)D. M. BĂTINEŢU - GIURGIU 1Cu ocazia aniversării a 110 ani de apariţieneîntreruptă a Gazetei MatematiceÎn istoria matematicii din ţara noastră Traian Lalescu reprezintă un creatorde diversitate rară, un mare animator al generaţiei sale de matematicieni, un omdotat cu o mare putere de muncă şi inteligenţă scânteietoare, un profesor înzestratcu deosebit talent pedagogic.Traian Lalescu s-a născut la 12/24 iulie 1882, în Bucureşti. Studiile primare le-afăcut la Bucureşti, primele două clase de gimnaziu la Craiova (1892-1894), iar claseleaIII-aşi a IV-a la Roman (1894-1896). Clasele a V-a şiaVI-ale-afăcut la LiceulInternat din Iaşi (actualul Colegiu Naţional "C. Negruzzi") în perioada 1896-1900.În liceu, ca şi în gimnaziu, Lalescu a fost premiantul I al clasei şi a primit premiulde onoare al şcolii (Lalescu se află trecut pe tabela de onoare a Liceului Internat dinIaşi).Chiar din clasa a VI-a a liceului (februarie 1898), Lalescu ajunge corespondent laGazeta Matematică. Profesorul său de mai târziu, inginerul Ion Ionescu, relateazădespre Traian Lalescu că: "Intrarea lui în rândul corespondenţilor "Gazetei Matematice"nu s-a făcut ca de obicei, în mod timid, lent, progresiv, ci deodată, intens, lamaximum posibil. A fost un caz unic de apariţiune la "Gazeta Matematică" deactivitate prodigioasă a unui tânăr licean!".În v. VIII (1902-1903), la pagina 244, Traian Lalescu a propus Problema 809, cuurmătorul enunţ:d 2n+1Să searatecă:µx 2ndx 2n+1 · sh 1 ch 1= − xx x 2n+2 .La pag. 283 din Gazeta Matematică, v. IX (1903-1904), este publicată soluţiadată de Traian Lalescu acestei probleme, urmată deonotă:"Se ştie că:sh x = x 1! + x33!Vom avea deci:şi, prin urmare:x 2n sh 1 x = P (x)+ 1x2n+1x2n+ ···+ + ··· şi ch x =1+x2 + ···+(2n +1)! 2! (2n)! + ··· .sh 1 x = 1 x + 13!x 3 + ... 1+ ···(2n +1)!x2n+1 (2n +1)!x + 1(2n +3)!x 3 + ···+ 1+ ··· ,(2n +2p +1)!x2p+1 P (x) fiind un polinom întreg în x de gradul 2n.1 Profesor, Colegiul Naţional "Matei Basarab", Bucureşti5


Seria din membrul al II-lea, uniform convergentă în tot planul exceptând originea,e derivabilă termen cu termen şi rezultatele găsite sunt serii convergente pe aceeaşiîntindere, ale căror sume sunt date de derivatele de acelaşi ordin ale membrului I.Observând acum că:d 2n+1dx 2n+1 (P (x)) = 0, d 2n+1 µ 1 (2p +1)(2p +2)···(2p +2n +1)dx 2n+1 x 2p+1 = −x 2n+2p+2şi că, prin urmared 2n+1 µ11dx 2n+1 (2p +2n +1)!x 2p+1 = −(2p)!x 2p · 1x 2n+2 ,obţinemd 2n+1 µx 2ndx 2n+1 sh 1 xch 1= − xx 2n+2 .(T.L.)Notă. Această problemă a fost rezolvată de D-nii: N. Abramescu, Gr. Orăşanu,G. Constantinescu, M. Radu, C. Gheorghiu şi I. G. Niculescu.În acelaşi mod se pot demonstra şi formulele:d 2n+1 µx 2ndx 2n+1 sin 1 cos 1 =(−1) n+1 xxx 2n+2 ;= − 1 µx 2n+2 1+ 12!x 2 + ···+ 1+ ···(2p)!x2p d n ³1/x´dx n x n−1 e =(−1) n e 1/xx n+1 etc. "Ca un omagiu adus marelui matematician român Traian Lalescu, vom da acesteiprobleme o nouă soluţie, accesibilă elevilor actualului liceu.Să considerăm funcţiile f n : R ∗ → R, f n (x) =x 2n sh 1 , unde n ∈ N. Se constatăimediatcă f n este indefinit derivabilă, oricare ar fi n ∈ N. Ne propunem săxdemonstrăm căf n (2n+1) (x) =− 1x 2n+2 ch 1 , ∀n ∈ N (1)xprin metoda inducţiei matematice, folosind formula lui Leibniz de derivare a produsuluia două funcţii indefinit derivabile, adicănX(uv) n = Cnu k (n−k) v (k) , ∀n ∈ N ∗ . (2)k=0µAvem f0 0 (x) = shx 1 0= − 1 x 2 ch 1 ,decipentrun =0formula (1) se verifică.xDe asemenea avem:f 1 (x) =x 2 sh 1 x , deci f 1 0 (x) =2x · sh 1 x − ch 1 x ;f 001 (x) =2sh 1 x − 2 x ch 1 x + 1 x 2 ch 1 x ;f1 000 (x) =− 2 x 2 ch 1 x + 2 x 2 ch 1 x + 2 x 3 sh 1 x − 1 x 4 ch 1 x = − 1 x 4 ch 1 x ,deci şi pentru n =1formula (1) se verifică.Presupunem că formula(1) este adevărată pentrun ∈ N ∗ adică arelocrelaţia6


f n (2n+1) (x) =− 1x 2n+2 ch 1 x , (3)şi demonstrăm că eaesteadevărată şi pentru n +1,adicăavemf (2n+3)n+1 (x) =− 1x 2n+4 ch 1 x . (4)Să observăm căf n+1 (x) =x 2 f n (x) , ∀n ∈ N, (5)şi atunci, cu ajutorul formulei (2), avem:n+1 (x) = ¡ x 2 f n+1 (x) ¢ 2n+3(2n+3)X= C2n+3f k n (2n+3−k) (x) · ¡x 2¢ (k)=f (2n+3)=C2n+3f 0 n(2n+3) (x) · x 2 + C2n+3f 1 n (2n+2) (x) · 2x + C2n+3f 2 (2n+1) (x) · 2=k=0=x 2 f n(2n+3)Conform presupunerii relaţia (3) fiind adevărată, rezultă că:f n(2n+2) (x)=şi atuncif (2n+3)n (x) =(x)+2(2n +3)xf n(2n+2) (x)+(2n +3)(2n +2)f n (2n+1) (x), ∀n∈N, (6)³³f (2n+1)nf (2n+2)nµ´0 1(x) =−x 2n+2 ch 1 0=xµ´0 2n +2(x) =x 2n+3 ch 1 x + 1x 2n+4 sh 1 0=x2n +2x 2n+3 ch 1 x + 1x 2n+4 sh 1 x , ∀n∈N,(2n +2)(2n +3)= −x 2n+4 ch 1 2n +2−x x 2n+5 sh 1 2n +4−x x 2n+5 sh 1 x − 1x 2n+6 ch 1 x == − (2n +2)(2n +3)x2 +1x 2n+6 ch 1 4n +6−x x 2n+5 sh 1 x . (8)Dacă ţinem seama de relaţiile (3), (7) şi (8), relaţia (6) devinef (2n+3)n+1 (x) =− (2n+2)(2n+3)x2 +1x 2n+4 ch 1 x − 4n+6x 2n+5 sh 1 x + 2(2n+2)(2n+3)x 2n+2 ch 1 x ++2(2n +3)x 2n+3 sh 1 (2n +2)(2n +3)−x x 2n+2 ch 1 x =− ¡ (2n+2)(2n+3)x 2 +1 ¢ ch 1 x +2x2 (2n+2)(2n+3)ch 1 x −− (2n +2)(2n +3)x 2 ch 1 = − 1x x 2n+4 ch 1 x ,= 1x 2n+5 µceea ce demonstrează cărelaţia (4) este adevărată.Conform principiului inducţiei matematice, rezultă căf n (2n+1) (x) =− 1x 2n+2 ch 1 , ∀n ∈ N.xBibliografie1. G. Şt. Andonie - Istoria matematicii în România, v.1,Ed.Şt., Buc., 1965.2. M. D. Bătineţu-Giurgiu, M. Bătineţu-Giurgiu, I. Bîrchi-Damian, A. Semenescu- Analiză matematică. Probleme pentru clasa a XI-a, Ed.MatrixRom,Buc.,2003.3. Colecţia "Gazeta Matematică", 1895-2005.7(7)


Câteva proprietăţi ale subgrupurilor finitedin GL n (Z)Gabriel DOSPINESCU 1Cu ocazia aniversării a 110 ani de apariţieneîntreruptă a Gazetei Matematice1. Introducere: lema lui Serre. Ceea ce veţi citi în continuare este o încercaretimidă de a expune o colecţie de rezultate referitoare la subgrupurile finite dinGL n (Z). Se prea poate ca demonstraţiile care urmează să fie cunoscute; autorul le-agăsit "aproape" singur şi crede că merităsă fie prezentate. Articole (mai serioase)despre proprietăţile acestor subgrupuri s-au scris multe şi, cu siguranţă, se vor maiscrie, căci problemele referitoare la ele sunt dificile şi multe dintre ele îşi aşteaptă deani buni rezolvările. Îl invităm pe cititorul interesat de rezultate mai profunde săcitească articolele din bibliografie, mult mai tehnice şi mai specializate. Se pare că în[3] arfiodescrieresuperbăaaceloraşi (sau chiar a mai multor) rezultate, însă, dinpăcate, nu am avut acces la acest articol, aşa că nu putem decât să-l recomandăm"orbeşte" cititorilor interesaţi de asemenea aspecte.Iată, mai întâi, ce rezultate vom demonstra (sau doar aminti). Vom deduce formasimplă ateoremeiJordan-Zassenhaus (cu ajutorul lemei lui Serre, de care am luatcunoştinţădin[7]) relativ la finitudinea claselor de izomorfism ale subgrupurilor finiteale lui GL n (Z), apoi vom demonstra că orice subgrup finit din GL n (Z) are cel mult(2n)! elemente şi că există 9 clase de izomorfism pentru subgrupurile lui GL 2 (Z).Vom începe cu lema lui Serre, un rezultat de o frumuseţe deosebită, care permiteoprimă majorare a ordinului subgrupurilor finite din GL n (Z); utilitatea acesteia nepermite să onumim"teoremă". Toate grupurile despre care va fi vorba în continuareau cel puţin două elemente.Teorema 1 (Lema lui Serre). Fie G ⊂ GL n (Z) un grup finit şi p>2 un numărprim. Considerăm aplicaţia ϕ : GL n (Z) → GL n (Z p ) care asociază fiecărei matriciA matricea claselor de resturi modulo p ale elementelor din A. Atunci restricţiaacestei aplicaţii la G este injectivă.Demonstraţie. Desigur, ϕ este bine definită şi este un morfism între grupurileGL n (Z) şi GL n (Z p ) (aşa cum se verifică imediat). Să presupunem că restricţia aplicaţieiϕ la G nu este injectivă, deci există A ∈ G, A 6= I n astfel încât ϕ(A) =ϕ(I n ).Asta înseamnă că putem scrie A = I n +pB, unde B ∈ M n (Z). Fieλ 1 ,λ 2 ,...,λ n valorileproprii ale matricii B; seştie atunci că A are valorile proprii 1+pλ i , 1 ≤ i ≤ n.Acum să privimcuatenţie sumele S k = λ k 1 + λ k 2 + ···+ λ k n (pentru k număr natural):toate vor fi numere întregi (cel mai simplu argument este teorema fundamentală apolinoamelor simetrice, căci toate aceste sume sunt polinoame cu coeficienti întregiîn sumele simetrice fundamentale ale numerelor λ 1 ,λ 2 ,...,λ n , iar aceste sume simetricesunt - modulo un semn plus sau minus - coeficienţii polinomului caracteristic almatricii B ∈ M n (Z), deciîntregi). Însă, G fiind finit, putem scrie A |G| = I n , decitrebuie să avem(1 + pλ i ) n =1, pentru fiecare 1 ≤ i ≤ n, iar de aici obţinem imediatcă |λ i | < 1, ∀1 ≤ i ≤ n. Or, aceasta înseamnăcăşiruldenumereîntregi(S k ) k≥1 tinde1 Student, École Normale Supérieure, Paris8


la zero, deci trebuie ca toţi termenii săi să fienuli(delaunrangîncolo). Osimplăaplicare a formulelor lui Newton ne va duce la concluzia că e necesar, pentru asta, catoţi λ i să fie egali cu 0; dar atunci toate valorile proprii ale matricii A sunt egale cu 1,deci (teorema Cayley-Hamilton) eaeste"rădăcină" a polinomului (X −1) n .Cumamvăzut, mai este rădăcină şi pentru X |G| − 1, deci va fi rădăcină pentru cel mai maredivizor comun al acestor polinoame, care este X − 1: adică A = I n (alt argument arfi că identitatea este singura matrice unipotentă diagonalizabilă, iar matricea A areaceste două proprietăţi: este unipotentă -căcitocmaiamarătat că toate valorile saleproprii sunt egale cu 1 -şi diagonalizabilă, deoarece polinomul său minimal nu aredecât rădăcini simple, fiind un divizor al lui X |G| − 1)şi teorema 1 este demonstrată.Săexaminăm puţin consecinţele acestei teoreme; obţinem imediat că ϕ(G) (imaginealui G prin morfismul ϕ) este un subgrup cu |G| elemente din GL n (Z p ). ÎnsăGL n (Z p ) are exact (p n − 1)(p n − p) ···(p n − p n−1 ) elemente (lăsăm cititorului caexerciţiu demonstraţia acestui rezultat clasic). Rezultă atunci, din teorema lui Lagrange,că |G| divide pe (p n − 1)(p n − p) ···(p n − p n−1 ), pentru orice subgrup finitG ⊂ GL n (Z) şi orice p>2 prim. În particular, există unnumăr finit de ordineposibile ale matricilor din GL n (Z) (participanţii la olimpiade - şi nu numai ei - trebuiesă-şi fi amintit celebra problemă: orice matrice din GL 2 (Z) are ordinul 1, 2,3, 4, sau6; încercaţi să demonstraţi aceasta pentru n =3!;maimult,curajoşii sepot gândi la o variantă mult mai generală: mulţimile ordinelor posibile ale matricilordin GL 2k (Z) şi GL 2k+1 (Z) coincid, pentru orice k ≥ 1 natural). De asemenea, mairezultă (totcauncazparticular)că ordinul oricărei matrici din GL n (Z) divide pe(3 n − 1)(3 n − 3) ···(3 n − 3 n−1 ) (această problemă a fost propusă de autor în Rec-Mat, pe vremea când nu cunoştea lema lui Serre; de altfel, am reuşit să demonstrămcă ordinuloricărei matrici din GL n (Z) este mai mic decât A √ n ln n , unde A este oconstantă pozitivă ce nu depinde de n, dar nu despre asta ne-am propus să vorbimaici). Tot din lema lui Serre mai putem deduce şi varianta simplă ateoremeiluiJordan-Zassenhaus, căci am obţinut că orice subgrup finit al lui GL n (Z) are cel mult(3 n − 1)(3 n − 3) ···(3 n − 3 n−1 ) elemente, deci, cu siguranţă, există unnumăr finitdeclasedeizomorfism în GL n (Z). Desigur, de aici şi până la demonstrarea teoremeilui Jordan-Zassenhaus (care afirmă finitudinea numărului claselor de conjugare alesubgrupurilor finite ale lui GL n (Z)) mai e mult de muncă, şi, oricum, nu vom faceasta aici; recomandăm excelentul articol [7].2. Majorări pentru ordinele subgrupurilor finite ale lui GL n (Z). Şi iatăcă ne apropiem de un punct sensibil al acestei note, anume de obţinerea unei majorăribune pentru ordinul oricărui subgrup finit din GL n (Z); amobţinut deja că cel maimare divizor comun al numerelor(p n − 1)(p n − p) ···(p n − p n−1 ),p>2, p primeste un astfel de majorant. Minkowski ademonstratşi un rezultat asemănător pentrup =2,anumecă ordinul oricărui subgrup finit din GL n (Z) divide pe 2 n2 (2 n −1)(2 n −2) ···(2 n − 2 n−1 ). Din păcate această majorare este oricum, dar nu uşoară şi estedeparte de a fi cea mai bună. Vom încerca sădăm un rezultat mai "simplu" (în sensulcă formula e mai simplă) care este, şi el, departe de valoarea optimală conjecturată.Teorema 2. Orice subgrup din GL n (Z) are cel mult (2n)! elemente; de fapt,9


ordinul oricărui subgrup din GL n (Z) divide pe (2n)!.Menţionăm că o majorare bună pentru ordinul maxim al subgrupurilor din GL n (Z)este, după câteştim noi, o problemă deschisăşi foarte dificilă. Cititorulvafiobservato minorare aproape evidentă: există subgrupuri cu 2 n · n! elemente (gândiţi-vă, deexemplu, la matricile ce au exact un 1 sau −1 pe fiecare linie şi pe fiecare coloană,în rest zerouri!). Cel mai bun rezultat obţinut până în prezent pare să fie o majorarede forma C n · (n!) 1+ε , unde C este o constantă care depinde de ε, nuşi de n, însăaceasta necesită un efort considerabil, pe care nu-l vom face aici. Invităm cititorul săgăsească mai multe detalii în [5], unde există chiarşi o menţiune referitoare la faptulcă 2 n · n! este valoarea maximă a ordinului unui subgrup finit din GL n (Z) pentrutoţi n 6∈ {2, 4, 6, 7, 8, 9, 10} (afirmaţie atribuită acolo lui W. Feit).Să revenim acum la Teorema 2, a cărei origine nu o ştim - ştim doar că aapărut în[7] fără menţiuni suplimentare şi fără. . . demonstraţie. Demonstraţia (cel puţin ceape care am găsit-o noi) cere răbdare din partea cititorului, precum şi nişte rezultateajutătoare, pe care le vom numi tot teoreme, datorită frumuseţii şi utilităţii lor.Teorema 3. Fie G ⊂ GL n (Z) un subgrup finit. Atunci, pentru orice k ∈ N ∗ ,|G| este un divizor al număruluiX(tr(g)) k .g∈GDemonstraţie. Înainte de toate, să spunem că nicimăcar nu e nevoie să presupunemcă elementele matricilor sunt numere complexe; acestea pot fi dintr-un corpcomutativ oarecare a cărui caracteristică estenumăr prim cu |G|. Demonstrăm maiîntâi afirmaţia pentru k =1.Săconsiderăm matriceaM = 1 Xg|G|pentru care, clar, avemM 2 = 1|G| 2 Xg∈GXgh = M,g∈G h∈Gdeoarece, pentru fiecare g ∈ G, avem(G fiind grup) {gh | h ∈ G} = G. EgalitateaM 2 = M implică faptulcă toate valorile proprii ale matricii M sunt 0 sau 1, decitr(M) (care este urma matricii M, deci suma valorilor proprii) este un număr întreg;or, folosind proprietăţile urmei, avemtr(M) = 1|G|Xtr(g),deci demonstraţia pentru k =1este încheiată (totodatăamrezolvatşi o problemămai veche de la concursul Putnam: dacă P tr(g) =0, G fiind un grup finit de matriciPg∈Gpătratice, atunci g =0;într-adevăr, egalitatea P tr(g) =0implică faptulcăg∈Gsuma valorilor proprii ale matricii M - definită camaisus-este0, deci toate valorileproprii sunt 0; atunci M este idempotentă şi nilpotentă, deci este matricea nulă).Acelaşi argument nu funcţionează însăpentruk ≥ 2 (din păcate); şi totuşi...Oclipă degraţie în algebra liniară a permis introducerea noţiunii de produs tensorialadouămatrici. Astfel,dacă A ∈ M n (K) şi B ∈ M p (K), produsullortensorialestedefinit prin10g∈Gg∈G


⎛⎞a 11 B ... a 1n B⎜A ⊗ B =⎝.. ... ⎟⎠ ∈ M np (K).a n1 B ... a nn BO proprietate fundamentală a produsului tensorial (uşor de verificat) este că(A ⊗ B) · (C ⊗ D) =(AC) ⊗ (BD), ∀A, C ∈ M n (K), ∀B,D ∈ M p (K);această egalitate ne permite să definim un subgrup G 0 ⊂ GL n 2(Z) prin G 0 = {g ⊗ g |g ∈ G} (relaţia de mai sus, precum şi faptul că det(A ⊗ B) =(detA) p · (det B) n ,pentru A, B ca mai sus, folosesc ca să arătăm că G 0 este subgrup al lui GL n 2(Z)).Acest subgrup are, evident, tot |G| elemente, deci îi putem aplica rezultatul dejademonstrat pentru a deduce că|G|| X tr(g ⊗ g) = X (tr(g)) 2g∈Gg∈G(dacă maifolosimşi formula foarte simplă tr(A ⊗ B) =tr(A) · tr(B)). Cititorul aînţeles acum modul în care va demonstra afirmaţia pentru orice k ∈ N ∗ (vom maispune doar că pentruk =3trebuie considerat G 00 = {(g ⊗ g) ⊗ g | g ∈ G}).Acum putem începe să demonstrăm Teorema 2. Să notăm x 1 >x 2 > ··· >x qelementele mulţimii {tr(g) | g ∈ G} şi să observăm că avemq ≥ 2 şi x 1 = n. Nefiindevidente (dar interesante şi în sine) vom demonstra aceste proprietăţi. În primulrând, am văzut că, dacă A ∈ G, atunciA |G| = I n , deci valorile proprii ale lui Asunt rădăcini ale unităţii, în particular ele au modulul 1. E clar atunci că avem| tr(A)| ≤ n, pentru orice A ∈ G; cumI n ∈ G, secheamăcă x 1 = n. Dar, sămaiobservăm, dacă A ∈ G − {I n } (şi existenţa unei asemenea matrici e asigurată depresupunerea făcută încă de la început), nu putem avea tr(A) =n, căci atunci toatevalorile proprii ale matricii A ar fi egale cu 1, ceea ce este imposibil (cititorul nu auitat argumentul final din demonstraţia teoremei 1); deci q ≥ 2. Înplus,dacănotămcu a 1 ,a 2 ,...,a q numărul apariţiilor numerelor x 1 ,x 2 ,...,x q respectiv în mulţimeaurmelor matricilor din G, teorema 3 afirmă că|G||a 1 x k 1 + a 2 x k 2 + ···+ a q x k q, ∀k ≥ 1.Desigur, mai avem şi |G| = a 1 + a 2 + ··· + a q ,precumşi a 1 =1(este suficientsă fiînţeles argumentele din acest paragraf pentru a ne convinge şi de acest lucru,precum şi de faptul că, dacă x q = −n, atunci şi a q =1;toateacesteobservaţii se vordovedi esenţiale în studiul subgrupurilor finite ale lui GL 2 (Z)). Iar avem nevoie deun rezultat ajutător.Teorema 4. Fie a 1 ,a 2 ,...,a q ,x 1 ,x 2 ,...,x q şi m numere întregi astfel încâtm | a 1 x k 1 + a 2 x k 2 + ···+ a q x k q, ∀k ∈ N ∗ .Atunci avem şim | a 1 (x 1 − x 2 ) ···(x 1 − x q ).Demonstraţie. Să considerăm seria formalăf(z) = a 11 − x 1 z + a 21 − x 2 z + ···+ a q1 − x q zşi să observăm că11


ÃqXqX! Ã qX!f(z) = a i + a i x i z + a i x 2 i z 2 + ··· ,i=1 i=1 i=1deci, folosind ipoteza, rezultă existenţa unor numere întregi b 0 ,b 1 ,b 2 ,... astfel încâtf(z) =m · P b j z j .Pedealtă parte, putem scrie şij≥0Pa1 (1 − x 2 z) ···(1 − x q z)f(z) =(1 − x 1 z)(1 − x 2 z) ···(1 − x q z) .Asta ne arată căseriaformală (de fapt, polinomul) de la numărător poate fi scris înforma Xa1 (1 − x 2 z) ···(1 − x q z)=m(1 − x 1 z)(1 − x 2 z) ···(1 − x q z) X b j z j ,j≥0qPdeci are toţi coeficienţii divizibili cu m, deundeobţinem că m | a i S (i)t , unde S (i)ti=1este a t-a sumă simetrică fundamentală înx 1 ,...,x i−1 ,x i+1 ,...,x q , ceea ce implicăşiqXqXqXm|x q−11 a i − x q−21 a i S (i)1 + ···+(−1) q−1 a i S (i)q−1saum|i=1qXi=1i=1i=1a i (x q−11 − x q−21 S (i)1 + ···+(−1) q−1 S (i)q−1 ).Cum, pentru i>1, avem(x 1 − x 1 ) ···(x 1 − x i−1 )(x 1 − x i+1 ) ···(x 1 − x q )=0,adicăx q−11 − x q−21 S (i)1 + ···+(−1) q−1 S (i)q−1 =0,ne rămâne doar căm | a 1 (x q−11 − x q−21 S (1)1 + ···+(−1) q−1 S (1)q−1 )=a 1(x 1 − x 2 ) ···(x 1 − x q ),ceea ce trebuia demonstrat.Iar asta încheie şi demonstraţia teoremei 2: din teoremele 3 şi 4 şi faptul că a 1 =1,rezultă că |G| divide (x 1 −x 2 ) ···(x 1 −x q ),careesteprodusulaq−1 numere naturalediferite şi cel mult egale cu 2n (deoarece urma oricărei matrici din G este un numărîntreg cuprins între −n şi n), deci divide şi pe (2n)!.Bibliografie1. G. P. Dresden - There are only nine finite groups of fractional linear transformswith integer coefficients, Mathematics Magazine, June 2004, 211-218.2. R. A. Horn, Ch. R. Johnson - Analiză matricială, Fundaţia Theta, Bucureşti,2001.3. J. Kuzmanovich, A. Pavlichenkov - Finite groups of matrices whose entries areintegers, American Mathematical Monthly, February 2002.4. T. J. Laffey - Lectures in integer matrices.5. D. N. Rockmore, Ki-Seng Tan - A note on the order of finite subgroups ofGL n (Z), Commutative Algebra, 2/1999.6. Ken-Ichi Tahara - On the finite subgroups of GL 3 (Z), Nagoya Math. Journal.7. Nicolas Tossel - Reseaux et théorèmes de finitude, Revue des mathèmatiques speciales,1-2/2005.12


Ceviene şi triunghiuri triomologiceTemistocle BÎRSAN 1Cu ocazia aniversării a 110 ani de apariţieneîntreruptă a Gazetei MatematiceÎn această Notă, pornind de la un triunghi oarecare, punem în evidenţă o configuraţiede triunghiuri triomologice cu acelaşicentrudegreutatecaşi triunghiul iniţial.Două triunghiuri, 4ABC şi 4XY Z, se numesc omologice dacădrepteleAX, BY ,CZ sunt concurente; punctul de concurenţă senumeşte centru de omologie al triunghiurilor.Triunghiurile date sunt triomologice dacă admit trei centre de omologie.1. Fie ABC un triunghi oarecare şi numerele α, β, γ ∈ R \{1} cu αβγ = −1. Pedreapta BC considerăm punctele A α , A β , A γ determinate de rapoartele A αBA α C = α,A β BA β C = β şi respectiv A γB= γ (utilizăm segmentele orientate pentru ca puncteleA γ CA γ , A β şi A γ să poată fi situate în orice poziţie pe BC, exceptând vârfurile B şi Cale 4ABC). Punctele B α ,B β ,B γ ∈ CA şi C α ,C β ,C γ ∈ AB se determină înmodsimilar. Condiţia αβγ = −1 asigură existenţa punctelor X α , Y α etc. definite prin{X α } = AA α ∩ BB β ∩ CC γ , {X β } = AA β ∩ BB γ ∩ CC α ,{X γ } = AA γ ∩ BB α ∩ CC β , {Y α } = AA α ∩ CC β ∩ BB γ , (1){Y β } = AA β ∩ CC γ ∩ BB α , {Y γ } = AA γ ∩ CC α ∩ BB β .Atât pe figură câtşi schematic dinA B CX α (α β γ)X β (β γ α)X γ (γ α β)A B C(α γ β) Y α(β α γ) Y β(γ β α) Y γ(2)se poate urmări formarea acestor puncte şi a triunghiurilor X α X β X γ şi Y α Y β Y γ .C βC αAX β1α =−31β =−2γ =− 6B γB βBαBC γGX αY αY βY γX γA α A βA γC1 Prof.dr.,Catedradematematică, Univ. Tehnică "Gh.Asachi",Iaşi13


Se observă că 4ABC şi 4X α X β X γ sunt invers orientate, pe când 4ABC şi4Y α Y β Y γ sunt la fel orientate.Propoziţia 1. Triunghiurile X α X β X γ şi Y α Y β Y γ sunt triomologice, centrele lorde omologie fiind vârfurile triunghiului ABC.Demonstraţie. Vom arăta următoarele:(i) 4X α X β X γ şi 4Y α Y β Y γ sunt omologice cu centrul A;(ii) 4X α X β X γ şi 4Y β Y γ Y α sunt omologice cu centrul C;(iii) 4X α X β X γ şi 4Y γ Y α Y β sunt omologice cu centrul B.Aceste trei afirmaţii decurg din (1). Astfel, afirmaţia (1) revine la a vedea cădreptele X α Y α , X β Y β şi X γ Y γ sunt concurente în A. Cum din prima şi a patraegalitate din (1) rezultă că X α ,Y α ∈ AA α ,vomaveacă A ∈ X α Y α . La fel obţinemrelaţiile A ∈ X β Y β şi A ∈ X γ Y γ . Aşadar (i) este adevărată. Pe aceeaşi cale sedovedesc (ii) şi (iii). Q.e.d.Observaţie. În consecinţă, configuraţia conţine şi perechile de triunghiuri triomologice:4ABC şi 4X α X β X γ , 4ABC şi 4Y α Y β Y γ ; pentru prima pereche avem:4ABC, 4X α X β X γ ; Y α ,4ABC, 4X β X γ X α ; Y β ,4ABC, 4X γ X α X β ; Y γ ,iar pentru a doua avem:4ABC, 4Y α Y γ Y β ; X α ,4ABC, 4Y β Y α Y γ ; X β ,4ABC, 4Y γ Y β Y α ; X γ ,(pe un rând sunt scrise două triunghiuri, pe baza schemei (2), şi centrul lor deomologie).2. În această secţiune vom stabili o altă proprietateaconfiguraţiei: cele trei triunghiuriau acelaşi centru de greutate. Pentru aceasta, vom utiliza metoda vectorială.Avem nevoie de următoareaLemă. Fie ABC un triunghi oarecare şi punctele A 0 ∈ BC, B 0 ∈ CA. Dacăλ = A0 BA 0 C , µ = B0 CB 0 A şi λµ − λ +16= 1, atunci cevienele AA0 şi BB 0 au un punct deintersecţie X şi avem−→ 1r X =λµ − λ +1 (λµ−→ r A + −→ r B − λ −→ r C ) . (3)( −→ r X notează vectorul de poziţie al punctului X faţă deooriginearbitrară.)Demonstraţie. Cu teorema lui Thales se arată uşor că λµ − λ +1 = 0 ⇔AA 0 k BB 0 .Din λ = A0 BA 0 C şi µ = B0 Curmează căB 0 A−→ r A 0 = 1 −→ r B −λ −→ r C ,1 − λ−→ r B 0 = 11 − µ1 − λ−→ r A .−→ r C − µ1 − µ14BAB′XA′C


Ţinând cont de aceste relaţii, ecuaţiile vectoriale ale cevienelor: (AA 0 ) −→ r = −→ r A +u ( −→ r A 0 − −→ r A ), (BB 0 ) −→ r = −→ r B + v ( −→ r B 0 − −→ r B ) se scriu sub forma(AA 0 )−→ r =(1− u)−→ r A +u −→ r B −λu −→ r C ,1 − λ 1 − λ(4)(BB 0 )−→ r =(1− v)−→ r B +v −→ r C − µv −→ r A .1 − µ 1 − µ(5)Vectorul −→ r X asociat punctului X de intersecţieseobţine din (4) sau (5) pentru usau v luat dintr-o soluţie (u, v) a sistemului liniar de ecuaţii1 − u = − µv1 − µ , uλu=1− v, −1 − λ 1 − λ = v1 − µ . (6)Găsim, cu uşurinţă, ca soluţie a sistemului (6) perechea (u, v) cu1 − λu =λµ − λ +1 , v = λµ − λλµ − λ +1 . (7)După înlocuirea lui u sau v din (7) în (4) sau (5), obţinem pentru −→ r X reprezentarea(3), q.e.d.Propoziţia 2. Triunghiurile ABC, X α X β X γ şi Y α Y β Y γ au acelaşi centru degreutate.Demonstraţie. Vom arăta că 4X α X β X γ şi 4ABC au acelaşi centru de greutate(la fel se procedează cu perechea <strong>format</strong>ă din4Y α Y β Y γ şi 4ABC). Este suficient săstabilim că−→ r Xα + −→ r Xβ + −→ r Xγ = −→ r A + −→ r B + −→ r C . (8)Într-adevăr, utilizând Lema relativ la 4ABC şi cevienele AA α şi BB β ,obţinem−→ 1r Xα =αβ − α +1 (αβ−→ r A + −→ r B − α −→ r C ); (9)similar obţinem şi relaţiile:−→ 1r Xβ =αβ − α +1 (−→ r A − α −→ r B + αβ −→ r C ) (4CAB şi CC α , AA β ), (10)−→ 1r Xγ =αβ − α +1 (−α−→ r A + αβ −→ r B + −→ r C ) (4BCA şi BB α , CC β ). (11)Ţinând seama de (9), (10) şi (11), avem−→ r Xα + −→ r Xβ + −→ 1 £r Xγ =(αβ +1− α)−→ r Aαβ − α +1+(1− α + αβ) −→ r B +(−α + αβ +1) −→ ¤r C= −→ r A + −→ r B + −→ r C .adică areloc(8), q.e.d.3. Să presupunem că triunghiul ABC este echilateral. Se constată uşor, pe caleelementară şi ca o consecinţă arelaţiilor (1), că triunghiurile X α X β X γ şi Y α Y β Y γsunt, la rândul lor, echilaterale. Conform Propoziţiei 2, aceste triunghiuri au acelaşicentru ca şi triunghiul ABC. Esteevidentă, în acest caz particular, înrudirea cu unrezultat remarcabil, teorema lui Barbilian: două triunghiuri echilaterale cu acelaşicentru sunt triomologice.15


Construcţii aproximative cu rigla şi compasul alenumărului πAlexandru MOSCALIUC 1Notaţia π pentru raportul dintre lungimea unui cerc şi diametrul său s-a încetăţenitîn matematică datorităluiL. Euler, care a utilizat-o în tratatul său Introductio inanalysis infinitorum (1748). Valori aproximative ale lui π au fost utilizate încă dinantichitatea timpurie de multe popoare. Amintim doar că Arhimede, întratatulAsupra măsurării cercului, agăsit că 3 1071


Cumsin 18 ◦ = 1 4³√5 − 1´, cos 18 ◦ = 1 q4³√ √6 − 2´, cos 15 ◦ =10 + 2 √ 5, tg 18 ◦ = 1 −5√ 2 5,sin 15 ◦ = 1 √ √6+ 2, tg 15 ◦ =2− √ 3,44inegalităţile (4) se scriu:qq60·³√ √5 − 1´³√6+ 2´− 10 + 2 √ √ ¡ √ ¢³√ √ 1−5 6 − 2´¸2 5 5 − 2 − 3


Aplicaţia 2. Fie cercul C (O, 1) şi punctele A, B,C şi D ca în figura de mai jos: BC =1, BD = AC.Arătaţi că lungimea semicercului AB (aria semicercului)este aproximativ egală culungimeasegmentului[AD] (respectiv aria triunghiului ABD), eroarea fiindmai mică decât 0, 01.Soluţie. Deoarece AB =2şi BC =1,rezultăcăAC = √ 3;lafel,dinBC =1şi BD = AC = √ 3,deducem că CD = √ 2. Atunci, AD = AC + CD =√ √2+ 3 ' π şi AABD = 1 2 AD · BC = 1 ¡√ √ ¢ π2+ 3 '2AOC11¡π · 12 ¢ etc.2 = 1 2Aplicaţia 3. Dat un pătrat de latură 1, construiţi numai cu compasul un cercde lungime aproximativ egală cu perimetrul pătratului.Soluţie. Mai întâi, să observăm că uncercdelungimeegală cu perimetrul pătratuluidat are raza 2 π .Dar,ţinând cont de Propoziţie, 2 π ' 2√ √ =2 ¡√ 3 − √ 2 ¢ .2+ 3Aşadar, urmează să construim cu compasul un cerc de rază 2 ¡√ 3 − √ 2 ¢ .Etapele unei posibile construcţii sunt:D1. Construim simetricul E al punctului B faţă deA:{E} = C (A, 1) ∩ C (D, DB).2. Construim punctul F astfel încât 4BEF să fieechilateral, iar F şi D să fiedeoparteşi de alta aEAdreptei BE: {F } = C (B,BE) ∩ C (E,EB); evident,A,D, F sunt coliniare şi AF = √ 3 (înălţime în 4BEF delatură 2).3. Construim punctul G de partea dreptei BE în careGse află F prin {G} = C (A, AC) ∩ C (B,AF). DeoareceAB =1, AG = √ 2 şi BG = √ 3, rezultă că 4AGB esteFdreptunghic în A şi, ca urmare, punctele A, F , G suntHcoliniare, iar FG = AF − AG = √ 3 − √ 2.4. Construm simetricul H al lui G faţădeF (construcţia,numai cu compasul, a simetricului M 0 al punctului M faţăde un punct O poate fi urmărităpefiguraalăturată); evidentGH =2 ¡√ 3 − √ 2 ¢ .5. Construim C(H, HG), carevaficerculcăutat: lungimealui este 4π ¡√ 3 − √ 2 ¢ ' 4 ¡√ 2+ √ 3 ¢¡√ 3 − √ 2 ¢ =4,cu o eroare de ³√ √ ³√ √ ³√ √4π 3 − 2´− 4=4 3 − 2´hπ − 3+ 2´i< 4M ′OP1BMQ³√ √3 − 2´· 0, 01,conform cu (1). Cum √ 3 − √ 2 < 1 2 ,vomavea³√ √4π 3 − 2´− 4 < 4 · 1 · 0, 01 = 0, 02 ,2adică eroarea cu care lungimea cercului construit este aproximată deperimetrulpătratuluieste mai mică decât0, 02.18CDB


Inegalităţi generatoare de noi inegalităţiI. V. MAFTEI 1Pornind de la anumite inegalităţi cunoscute ne propunem să obţinem noi inegalităţi.Propoziţia 1. Să se demonstreze că¡x 1 x 2 ···x k xn−11 + x n−12 + ···+ x n−1 ¢k ≤ xn+k−11 + x n+k−12 + ···+ x n+k−1k, (1)∀x 1 ,x 2 ,...,x k ∈ R ∗ +, ∀n, k ∈ N, n,k ≥ 2.Demonstraţie. Utilizând relaţia dintre mediile aritmeticăşi geometrică, aplicatănumerelor a 1 ,a 2 ,...,a k ∈ R ∗ +, n, k ∈ N, n, k ≥ 2, obţinem succesiv:pn+k−1a n 1 a 2 ···a k ≤ na 1 + a 2 + ···+ a k,p n + k − 1n+k−1a 1 a n 2 ···a k ≤ a 1 + na 2 + ···+ a k,n + k − 1(2).......................................pn+k−1a 1 a 2 ···a n k ≤ a 1 + a 2 + ···+ na k.n + k − 1Sumând p inegalităţile (2), rezultă căn+k−1a n 1 a 2 ···a k + n+k−1p a 1 a n 2 ···a k + ···+ n+k−1p a 1 a 2 ···a n k ≤ a 1 + a 2 + ···+ a k .Dacă notăm n+k−1√ a i = x i , i = 1,k,obţinemx n 1 x 2 ···x k + x 1 x n 2 ···x k + ···+ x 1 x 2 ···x n k ≤ x n+k−11 + x n+k−12 + ···+ x n+k−1k,care este tocmai inegalitatea (1).Pentru k =2şi n =2h, h ∈ N ∗ , inegalitatea (1) devinex 2h+11 + x 2h+1 ¡2 ≥ x 1 x 2 x2h−11 + x 2h−1 ¢2 . (3)Propoziţia 2. Fie a, b, c ∈ R ∗ + şi k ∈ N. Atunci, are loc inegalitatea(ab) k−1a 2k+1 + b 2k+1 +(ab) k−1 + (bc) k−1b 2k+1 + c 2k+1 +(bc) k−1 + (ac) k−1a 2k+1 + c 2k+1 +(ac) k−1 ≤1≤ab (a + b)+1 + 1bc (b + c)+1 + 1ac (a + c)+1 . (4)Demonstraţie. Aplicând inegalitatea (3) de k ori, obţinemx1 2k+1 + x2 2k+1 ≥ (x 1 x 2 ) k (x 1 + x 2 ) , ∀x 1 ,x 2 ∈ R ∗ +, ∀k ∈ N. (5)Ţinând seama de (5), putemscriea 2k+1 + b 2k+1 ≥ (ab) k (a + b) , (6)de undea 2k+1 + b 2k+1 +(ab) k−1 ≥ (ab) k−1 [ab (a + b)+1]1 Profesor, Colegiul Naţional "Sf. Sava", Bucureşti19


sau(ab) k−1a 2k+1 + b 2k+1 +(ab) k−1 ≤ 1ab (a + b)+1 .Sumând această inegalitate cu analoagele ei, obţinem (4).Observaţie. Dacă în(4) luăm k =2şi considerăm abc =1, suntem conduşi lainegalitateaaba 5 + b 5 + ab + bcb 5 + c 5 + bc + cac 5 + a 5 ≤ 1, (7)+ cacare a fost discutată la O. I. M. din anul 1996, India.Propoziţia 3. Fie numerele a, b, c ∈ R ∗ +.Să se demonstreze căa 2n+1b n + b2n+1c n+ c2n+1a n ≥ a n b + b n c + c n a, ∀n ∈ N. (8)Demonstraţie. Înmulţind inegalitatea (6), consideratăpentruk = n, cua n c n ,iar analoagele ei cu b n a n şi respectiv b n c n , vom obţine relaţiilea 3n+1 c n + b 2n+1 a n c n ≥ a 2n+1 b n c n + a 2n b n+1 c n ,b 3n+1 a n + c 2n+1 b n c n ≥ b 2n+1 a n c n + b 2n c n+1 a n ,c 3n+1 b n + a 2n+1 c n b n ≥ c 2n+1 b n a n + c 2n a n+1 b n ,din care, prin adunare, deducem căa 3n+1 c n + b 3n+1 a n + c 3n+1 b n ≥ a n b n c n (a n b + b n c + c n a) ,adică (8) .Procedând ca în Propoziţia 3 se obţinePropoziţia 4. Pentru ∀n ∈ N şi ∀a, b, c ∈ R ∗ + avema b2n+2c n+ b c2n+2a n+ c a2n+2b n ≥ ab n+2 + bc n+2 + ca n+2 . (9)Propoziţia 5. Să searatecă ∀n ∈ N ∗ avem: qa) 1 2k+1 +2 2k+1 + ···+ n 2k+1 n (n +1) n≥ (n!) 2k , (10)2b) 1 2n+1 +2 2n+1 + ···+ n 2n+1 n (n +1)≥ (n!) 2 . (11)2Demonstraţie. Pentru n =1avem egalitate. Considerăm n ≥ 2 şi înlocuim îninegalitatea (6) succesiv a =1şi b = n, a =2şi b = n − 1, ... , a = n şi b =1.Sumând inegalităţile rezultate, vom obţine2 ¡ 1 2k+1 +2 2k+1 + ···+ n 2k+1¢ h≥ (n +1) 1 k n k +2 k (n − 1) k + ···+ n k 1 ki .qCum paranteza pătrată este≥ n n (1 k · 2 k · ··· ·n k ) 2 = nq(n!) n 2k ,avem2 ¡ 1 2k+1 +2 2k+1 + ···+ n 2k+1¢ ≥ n (n +1)q(n!) n 2k ,adică (10). Luând în (10) k = n, obţinem inegalitatea (11).20


Asupra unei probleme dată laONM,Bistriţa, 2005Claudiu-Ştefan POPA 1Cele ce urmează au ca punct de plecare o problemă datălaONM,Bistriţa, 2005[1] aparţinând autorului acestei note şi pe care o vom nota în continuare cu (P ):(P ) Fie ABCD un trapez cu bazele AB şi CD, având diagonalele perpendiculareîn O. Pe semidreptele (OA şi (OB se consideră puncteleM şi respectiv N astfelîncât unghiurile \ANC şi \BMD să fie drepte. Notăm cu E mijlocul segmentuluiMN.Săsearatecă:a) triunghiurile OMN şi OBA sunt asemenea;b) dreapta OE este perpendiculară pedreaptaAB.Rezolvarea acestei probleme poate fi găsită deasemeneaîn[1].Considerăm configuraţia geometrică pusă în valoare de (P ) îndeajuns de generoasăpentru a prezenta alte câteva rezultate legate de ea. Dăm întâi o caracterizare atrapezului ortodiagonal, interesantă şi în sine.Propoziţia 1. Fie ABCD un patrulater convex şi AB k CD. DacăpunctulOeste intersecţia diagonalelor sale, patrulaterul este ortodiagonal dacă şi numai dacăAB · CD = AO · CO + BO · DO.Demonstraţie. AB k CD ⇒ 4AOB ∼ 4COD ⇒AOCO = BODO = AB AO · CO BO · DO AB · CD AO · CO + BO · DO=CD CO 2 =DO 2 =CD 2 =CO 2 + DO 2 .Acum AO · CO + BO · DO = AB · CD ⇔ CD 2 = CO 2 + DO 2 ⇔ AC ⊥ BD, q.e.d.Adăugăm la ipoteza problemei (P ): punctele K, L sunt mijloacele bazelor [AB],respectiv [CD] iar punctul D 0 este simetricul punctului D faţă de punctul O. Înaceste condiţii, pentru cele ce urmează presupunem cunoscute următoarele: puncteleK, O şi L sunt coliniare, A AOD = A BOC , A 2 AOD = A AOBA COD ([2], p. 243).Propoziţia 2. În ipoteza problemei (P ), aulocurmătoarele:i) MN = √ AB · CD şi MN < KL;ii) MN ⊥ KL;iii) A OMN = √ A AOB ·A COD ;iv) AN k MD 0 ;Demonstraţie. i) 4ANC şi 4BMD sunt dreptunghice în N, respectiv M şiNO ⊥ AC, MO ⊥ BD. Aplicând teorema înălţimii obţinem NO 2 = AO · CO şiMO 2 = BO · DO. Cum m( \MON)=90 ◦ ,avemMO 2 + NO 2 = MN 2 şi obţinemMN = √ AB · CD. Deoarece KL = KO + LO = AB 2 + CD AB + CD= şi2 2√ AB + CDAB · CD < (ABCD trapez, deci AB 6= CD), rezultă că MN < KL.2ii) Fie R ∈ (OL astfel încât L ∈ (OR) şi (OL) ≡ (RL). Cum (DL) ≡ (CL),urmeazăcă OCRD este paralelogram. Dar CO ⊥ DO,deciOCRD este dreptunghi şi1 Profesor, Şcoala "Alecu Russo", Iaşi21


avem \CDO ≡ \CRO. Din(P ), punctul a) avem \CDO ≡ \NMO; deci \CRO ≡ \NMO.Aceasta şi MO ⊥ CR conduc la NM ⊥ RO ⇔ NM ⊥ KL.iii)ON · OMA MON = =DC√ √2AO · CO · BO · DO==O√2AO · CO · BO · DON=.2D′Cum 4AOB ∼ 4COD, avemAO · DO =PAO · DOEBO · CO, deci A MON = = A AOD . A2K BDar A 2 AOD = A AOBA COD , deci A OMN = M= √ A AOB A COD .iv) La fel ca la iii), OM · ON = OA · OD. Deci ONOA = OD ONsauOM OA = OD0OM ,adică AN k MD 0 , q.e.d.Observaţie. Dacă MN ∩ LK = {P }, propunem cititorului să demonstreze căOE = √ LO · KO şi OP = p dist (O; AB) · dist (O; CD).Bibliografie1. G.M. seria B, nr. 7/2005, p.298 şi p. 30<strong>1.2</strong>. D. Mihalca, I. Chiţescu, M. Chiriţă - Geometria patrulaterului, Ed. Teora, Bucureşti,1998.ERATAMai mulţti colaboratori aduc la cunoştinţă Redacţiei revistei următoarea greşealăîn scrierea numelui marelui matematician Leonhard Euler: în loc de Leonhard s-ascris Leonard atât în titlul materialului din nr. 2/2004, p. 129, cât şi în cel din nr.2/2005, p. 119 (prin preluarea primului pe calculator).COMENTARIUD-l D. Plăeşu din Iaşi semnalează Redacţiei faptul că Problema L.62, autorM.Bîrsan, publicată în nr. 1/2004 este cunoscută — apare în cartea lui W. Sierpińskiintitulată Ce ştim şi ce nu ştim despre numerele prime (în l.rom.la Editura Ştiinţifică,Bucureşti, 1966) la p. 104. Cele două soluţii date acestei probleme în nr.1/2005, pp. 67-68, diferă desoluţia prezentată încarteamenţionată.Vizitaţi pe Internet revista "Recreaţii Matematice" la adresahttp://www.recreatiimatematice.uv.ro22


Asupra criteriului de congruenţă LLUMarius TIBA 1Aşacumsearatăîn[1], următorul criteriu de congruenţă a triunghiurilor întindenumeroase capcane prin aplicarea sa incorectă. Redăm aici rezultatul din care decurgeacest criteriu.Propoziţie. Dacă două laturişi unghiul opus uneia dintre ele ale unui triunghisunt respectiv congruente cu două laturişi unghiul opus uneia dintre ele ale altuitriunghi, iar bα şi β b notează unghiurile opuse celorlalte laturi congruente,a) atunci bα şi β b sunt sau congruiente sau suplementare;b) (Criteriu LLU) dacă înplusbα şi β b sunt de acelaşi tip (adică ambelesuntascuţite, obtuze sau drepte), atunci cele două triunghiuri sunt congruente.Problema 1. Pe laturile (OX şi (OY ale unui unghi ascuţit \XOY se iaupunctele A şi B astfel încât [OA] ≡ [OB]. În interiorul unghiului se ia un punct Msupus condiţiei \OMA ≡ \O<strong>MB</strong>. Găsiţi locul geometric descris de M.Rezolvare. Vom arăta că loculcăutat este <strong>format</strong> din Obisectoarea interioară a unghiului \XOY şi arcul cercului circumscris4AOB cuprins în interiorul unghiului, pe care-lnotăm AB. FieM un punct ce satisface condiţiile din enunţ.Ca urmare 4OAM şi 4OBM au unghiurile \OAM şi \OBMsau congruente sau suplimentare (conform punctului a) alPropoziţiei). În cazul în care \OAM ≡ \OBM, punctul M se ABaflă pe bisectoarea unghiului O. b Dacă aceste unghiuri suntsuplimentare, atunci patrulaterul OA<strong>MB</strong> este inscriptibil şi,MXYca urmare, M ∈ AB. Reciproca rezultă imediat.Menţionăm că lafazajudeţeană a O. M. din Vaslui, 2005, cl. a VI-a, s-a cerutsă searatecă locul geometric este doar bisectoarea interioară a unghiului \XOY .Nerespectarea Propoziţiei conduce la erori ca aceasta (prezentă şi în barem) sau ceadin problema T12, [2, pag. 10].Problema 1 ne sugereazăProblema 2. Găsiţi locul geometric al punctelor M pentru care \OAM ≡ \OBM,unde notaţiile sunt aceleaşi ca în Problema 1.Rezolvare. Judecând analog ca la Problema 1, obţinem locul căutat <strong>format</strong> dinbisectoarea interioară a unghiului \XOY şi segmentul [AB] (fără capete).Ca o extindere a Problemei 1, dăm următoareaProblema 3. Fie ABCD un trapez isoscel şi fie O mijlocul bazei mici [CD].Găsiţi locul geometric al punctelor M situate în interiorul liniei frânte <strong>format</strong>e dinsemidreptele (DA, (CB şi segmentul [CD], astfelîncât \OMA ≡ \O<strong>MB</strong>.1 Elev,cl.aVII-a,ColegiulNaţional "C. Negruzzi", Iaşi23


Rezolvare. Această problemă se reduce la Problema 1,deoarece [OA] ≡ [OB] (4OBC ≡ 4OAD (LUL)). Astfel,locul geometric cerut este <strong>format</strong> din punctele mediatoareisegmentului CD şi ale arcului cercului circumscris 4OAB,aflate în interiorul liniei frânte date.Problema 3 ne sugerează următoarele două probleme, pecare le propunem cititorului spre rezolvare:Problema 4. Se modifică Problema3luândpunctulOla intersecţia laturilor neparalele ale trapezului.Problema 5. Modificăm Problema 3, luând condiţia\AMD ≡ \BMC.ADOC<strong>MB</strong>Bibliografie1. D. Miheţ - Criteriul de congruenţă LLU, RMT an II (seria a 4-a), nr. 2, 1997,pag. 3-7.2. I. Pătraşcu - Probleme de geometrie plană, Editura Cardinal, Craiova, 1996.IMPORTANT• În scopul unei legături rapide cu redacţia revistei, pot fi utilizate următoareleadrese e-mail: tbirsan@math.tuiasi.ro, profgpopa@yahoo.co.uk .Peaceastă cale colaboratorii pot purta cu redacţiaundialogprivitorlamaterialeletrimise acesteia, procurarea numerelor revistei etc. Sugerăm colaboratorilorcare trimit probleme originale pentru publicare să le numeroteze şisă-şi reţină o copie xerox a lor pentru a putea purta cu uşurinţă odiscuţieprin e-mail asupra acceptării/neacceptării acestora de către redacţia revistei.• La problemele de tip L se primesc soluţii de la orice iubitor de matematicielementare (indiferent de preocupare profesională sau vârstă). Fiecare dintresoluţiile acestor probleme - ce sunt publicate în revistă dupăunan-vafiurmată de numele tuturor celor care au rezolvat-o.• Adresăm cu insistenţă rugămintea ca materialele trimise revisteisă nufie(sănufifost)trimiseşi altor publicaţii.• Rugăm ca materialele tehnoredactate să fie trimise pe adresa redacţiei însoţitede fişierele lor (de preferinţă înL A TEX).24


O generalizare a identităţii Botez - CatalanIoana OLAN 1În 1872, N. Şt. Botez publică o lucrare originală în care apare identitatea1n +1 + 1n +2 + ···+ 12n +1 =1− 1 µ 12 1 · 3 + 12 · 5 + ···+ 1, n ∈ N ∗ ,n (2n +1)care, dacă ţinem seama de formula de descompunere12k (2k +1) = 12k − 12k +1 , k ∈ N∗ ,se aduce la forma1n +1 + 1n +2 + ···+ 12n =1− 1 2 + 1 3 − 1 4 + ···+ 12n − 1 − 12n , n ∈ N∗ , (1)numită identitatea Botez - Catalan. Ne propunem să-i dăm o generalizare.Amintim o demonstraţie a formulei (1), generalizarea obţinându-se în acelaşi fel:1 − 1 2 + 1 3 − 1 4 + ···+ 12n − 1 − 1 µ12n = − 1 µ 1+2 3 4− 1 µ 1+ ···+2n − 1 − 1 =µ2n= 1+ 1 2 − 2 · 1 µ 1+2 3 + 1 4 − 2 · 1 µ 1+ ···+42n − 1 + 1 2n − 2 · 1=2n=1+ 1 2 + 1 3 + 1 4 + ···+ 12n − 1 + 1 µ 12n − 2 2 + 1 4 + ···+ 1 =2n=1+ 1 2 + 1 3 + ···+ 12n−1 + 1 µ1+2n − 1 2 + ···+ 1 = 1n n+1 + 1n+2 + ···+ 12n .Propoziţie. Pentru n ∈ N ∗ şi m ∈ N, are loc egalitatea1− 2m − 12 m + 13 m − 2m − 14 m +···+ 1(2n − 1) m − 2m − 1 1(2n) m =(n +1) m +···+ 1(2n) m . (2)(Pentru m =1se obţine identitatea (1).)Demonstraţie. Într-adevăr, avem:1 − 2m − 12 m + 13 m − 2m − 14 m + ···+ 1(2n − 1) m − 2m−1(2n) m =µ= 1+ 1 1 µ 12 m −2m 2 m +3 m + 1 1 µ14 m −2m 4 m + ···+(2n − 1) m + 1(2n) m −2 m 1 (2n) m ==1+ 12 m + 13 m + 14 m + ···+ 1(2n − 1) m + 1 µ 1(2n) m − 2 m 2 m + 14 m + ···+ 1 (2n) m ==1+ 12 m + 13 m + ···+ 1(2n − 1) m + 11 1=(n +1) m +(n +2) m + ···+ 1(2n) m ,(2n) m −q.e.d.µ1+ 12 m + ···+ 1n m =Cazuri particulare. Pentru m =2şi m = n, formula(2) devine:1 − 3 2 2 + 1 3 2 − 3 4 2 + ···+ 1(2n−1) 2 − 3(2n) 2 = 1(n+1) 2 + 1(n+2) 2 + ···+ 1(2n) 2 ,1− 2n −12 n + 13 n − 2n −14 n + ···+ 1(2n−1) n − 2n −1 1 1(2n) n =(n+1) n +(n+2) n + ···+ 1(2n) n .1 Elevă, cl. a VIII-a, Colegiul Naţional "C. Negruzzi", Iaşi25


Acoperiri ale planului laticial cu figuriMarius PACHIŢARIU 1Există numeroase probleme de concurs care implică acoperiri ale unor figuri laticiale,de exemplu dreptunghiuri, cu un număr de copii ale unei alte figuri date (dominouri,trominouri) sau cu alte copii scalate ale însuşi dreptunghiului.Observând metodele si tehnicile acestor tipuri de probleme, putem analiza şiacoperiri ale întregului plan cu diferite figuri. Vom lucra în continuare doar în planullaticial. Pentru aceasta, să definim planul laticial şi să dăm coordonate pătrăţelelorcare îl alcătuiesc. Introducem în continuare o serie de noţiuni.Considerăm dreptele de ecuaţie x = r, r ∈ Z, y = q, q ∈ Z. Pătratele de latură 1determinate de punctele lor de intersecţie vor constitui elementele planului laticial.Vom atribui coordonate acestor pătrate în următorul mod: dacă unpătrat se află laintersecţia benzilor determinate de x = r, x = r +1şi respectiv, y = q, y = q +1,atunci vom spune că pătratul are coordonate r, q sau mai simplu vom numi pătratul[r, q]. Vomnumivector între două pătrate vectorul ai + bj, cui, j versorii axelor şia, b numărul de pătrăţele orizontale, respectiv verticale care separă cele două pătrateconsiderate, cu semnul asociat corespunzător.Numim plantaţie orice colecţie de pătrate ale planului laticial, conexă însensulcă dinoricepătrat putem ajunge în oricare altul printr-o succesiune finită dedeplasăriunitare (translaţii după una din cele 4 direcţii),astfelîncâtdupă fiecare pas(translaţie) ne aflăm încă înunuldintrepătratele colecţiei. Numim figură mulţimeamaximală deplantaţii cu proprietatea că fiecare plantaţiepoatefiobţinută din oricarealta a figurii prin translaţii, rotaţii şi simetrii faţă dedrepteparalelecuaxele.Cu alte cuvinte, figura reprezintă oclasădeechivalenţă.Fie un set X de figuri. Numim acoperire a planului orice set Y de plantaţii cuelemente aparţinând figurilor din X, astfel încât fiecare punct al planului aparţinela cel puţin o plantaţie din Y .Numimmăsura unei acoperiri sup n[i, j], unden[i, j]este numărul de plantaţii cărora îi aparţine patratul [i, j]. Numim n-acoperire oacoperire în care fiecare pătrat al planului aparţine aceluiaşi număr de plantaţiimodulo n. Pentru 2-acoperiri vom considera acoperire impară cea în care fiecarepătrat e acoperit de un număr impar de ori. 2-acoperirile pare nu ne interesează,întrucât considerând acceaşi plantaţiededouăorişi considerând oricâte astfel deperechi, vom obţine întotdeauna o 2-acoperire pară.Observaţia 1. Reuniunea a două acoperiri este o acoperire şi măsura reuniuniiadouă acoperiri este cel mult suma măsurilor celor 2 acoperiri.Putem acum să nepunemoseriedeîntrebări:Întrebarea 1. Care sunt seturile X de cardinal 1 pentru care există acoperiri cumăsura 1?Un exemplu netrivial de figuri în spaţiucuaceastă proprietate îl oferă următoareaProblemă. Lipim câte un cub unitate pe fiecare faţă a unui cub unitate. Arătaţică putem umple spaţiul folosind copii ale solidului rezultat. (Austrian-Polish 2000)1 Elev, Colegiul Naţional, Iaşi26


Întrebarea 2. Care sunt seturile X de cardinal 2 pentru care există acoperiri cumăsura 1?Întrebarea 2 0 . Dati exemple de doua seturi Y şi Z de cardinal 1 pentru care nuexista acoperire cu măsura 1 cu setul Y şi nu există acoperire cu măsura 1 cu setulZ, dar pentru care există acoperire cu măsura 1 cu setul Y S Z.Vezi la pagina 74 exemplul 1. Demonstraţi că exemplulesteîntr-adevăr bun!Întrebarea 2 00 . Daţi exemple de două seturi de figuri de cardinal 1: Y ,pentrucare nu există acoperire cu măsura 1 şi Z, pentrucareexistă o astfel de acoperire şipentru care există acoperire cu setul Y S Z cu măsura 1.Vezi la pagina 74 exemplul 2. Demonstraţi că exemplulesteîntr-adevăr bun!Întrebarea 3. Care sunt seturile X de figuri de cardinal n, n ∈ N, pentrucareexistă acoperiri cu măsura 1?Întrebarea 3 0 . Daţiexempledeseturiden seturi X i , i = 1,n de figuri, decardinale 1, cu proprietatea că nici unul dintre ele nu poate genera o acoperire cumăsura 1, dar reuniunea lor da.Vezi la pagina 74 exemplul 3. Demonstraţi că exemplulesteîntr-adevăr bun!Întrebarea 4. Orice figură poate genera o 2-acoperire impară a planului cumăsura finită? Dacă vom considera minimul măsurii peste toate acoperirile posibile,putem găsi un maxim pentru acesta în funcţiedefigurafolosită?Nu vom răspunde aici întrebărilor 1, 2, 3, fiind prea generale. Desigur, seturileX care constituie răspunsurile primelor 2 sunt particularizări ale seturilor din a treiaîntrebare. Caracterizări ale primului tip de seturi din anumite puncte de vedere suntcu siguranţă posibile, pe când o caracterizare în cazul general pare imposibilă (veziexemplele întrebărilor 2 0 , 2 00 , 3 0 ).Vom răspunde în schimb ultimei întrebări. Răspunsurile sunt DA şi DA. Acoperirilemodulo 2 ne dau într-adevăr de ajunsă libertate. Pentru demonstraţiile următoarevom renunţa la condiţia de conexitate din definiţia plantaţiilor.Demonstraţia 1. Vom numi cardinal al unei plantaţii (figuri) numărul depătrăţele pe care le conţine. Pentru o figură de cardinal impar putem lua următoareaacoperire: Fie o plantaţie oarecare şi toate translaţiile ei care se păstrează pe latice.Atunci plantaţiile rezultate şi cu cea iniţială reprezintă o 2-acoperire. Într-adevăr,fiecare pătrat al planului laticial este acoperit de exact (cardinalul plantaţiei)-ori,deci pentru cardinalul impar avem o 2-acoperire. Mai mult, avem măsura acopeririiegală cu cardinalul figurii.Din păcate această cale nu pare să furnizeze soluţie pentru figurile de cardinalpar.Demonstraţia 2. Figura F fiind finită, o putem include într-un pătrat cu laturilepe latice, deci o putem include într-un pătrat cu latura putere a lui 2. Fie 2 k laturacelui mai mic astfel de pătrat. Să considerăm acoperirea de măsură 1cuastfeldepătrate de latură 2 k (chiar o omotetie a planului laticial). Plasăm în fiecare pătrat delatură 2 k al acestei acoperiri figura F corespunzătoare. Asociem pătrăţelelor unitateale pătratului de latură 2 k valoarea 1 dacă pătrăţelul este în F şi 0 altfel. Cu altecuvinte am făcut o primă acoperire a planului cu figuri F . Vom face o serie de paşi.27


Primul pas: Considerăm acoperirea obţinută ca mai sus, dar înlocuind figura Fcu simetrica ei faţă de(Oy, astfel încât această acoperire să suprapună seturiledepătrate de latura 2 k peste cele considerate anterior. Fie F 0 noua figură obţinută dinsuprapunerea celor două acoperiri (reuniunea) şi considerarea modulo 2 (un patrăţelare asociat 1 dacă esteacoperitdeunnumăr impar de plantaţii şi 0 altfel). Procedămanalog cu F 0 dar faţă deaxa(Ox. Obţinem o nouă figură F 00 ,careestesimetricăşifaţădeorizontalăşi faţă de verticală, încadratăînpătratul de latură 2 k .Săobservămcă fiecare pătraţel a fost acoperit de cel mult 4 ori.Realizăm acoperirile determinate de translaţii ale acoperirii de mai sus cu figuraF 00 ,devectori2 k−1 i, 2 k−1 j, 2 k−1 i +2 k−1 j.Să considerăm reuniunea celor 4 acoperiri. Privind mai atent, observăm că amobţinut astfel o nouă acoperire cu pătrate de latură 2 k−1 ,cuofigură F (2) în fiecarepătrat, simetrică orizontalşi vertical. Acest proces reprezintă de fapt echivalentulprimei părţi, întrucât acoperim figura din pătratul de latură 2 k−1 cu simetrica eifaţă deaxaverticalăşi apoi cu simetricele faţă de axa orizontală a celor două figuriobţinute. Fiecare pătrat a fost acoperit de cel mult 4+4+4+4=4· 4 ori.Putem continua acum cu următorii paşi analogi cu partea a doua a pasului 1(partea cu translaţiile) pănă ajungem la acoperirea cu pătratul de latura 1, care areîn interiorul lui figura F (k +1), care poate fi ori mulţimea vidă oriînsuşi pătrăţelul,cazîncareamobţinut o 2-acoperire impară a planului cu F (şi o măsură de cel mult4 · 4 · 4 · ...· 4=4 k ).Pentru a determina numărul la care ajungem (0 sau 1) în F (k +1), vom consideraaceeaşiseriedepaşi ca mai sus, urmărind în acelaşi timp cum evoluează suma tuturorvalorilor asociate patrăţelelor interiorului pătratului de latură 2 k .Să presupunem că amconsideratpătratul de vârfuri [1, 1], [1, 2 k ], [2 k , 1], [2 k , 2 k ](ne referim aici la cele 4 pătrate unitare) şi fie a i,j =1dacă pătratul [i, j] este înfigura F şi 0 altfel.Vomdaacumunexemplude"paşi" făcuţi ca mai sus:Exemplul 4.1 111000101100 00 00 1 1 01 1 1 11 0 0 10 1 1 00 0 0 00 1 1 00 1 1 00 0 0 01111Să notăm b i,j , i = 1, 2 k−1 , j = 1, 2 k−1 valorile asociate acoperirii rezultate dinsuma celor 4 acoperiri (fără a le reduce modulo 2; adunăm a i,j -urile corespunzătoarepentru a proba dacă [i, j] estesaunuînF (2)). Acum este uşor să observăm căb i,j = a i,j + a i,2k +1−j + a 2k +1−i,j + a 2k +1−i,2 k +1−j. Dar aceasta înseamnă că:Xb i,j =X ¡ ¢ Xai,j + a i,2 k +1−j + a 2 k +1−i,j + a 2 k +1−i,2 k +1−j = a i,ji≤2 k−1 i≤2 k−1i≤2 kj≤2 k−1 j≤2 k−1 j≤2 kAm demonstrat prin aceasta că suma valorilor asociate rămâne aceeaşi. Cu altecuvinte, valoarea din ultimul pătrat după ultimul pas este egală cu suma valoriloriniţiale ale figurii F . Deci, dacă figuraF are un număr impar de pătrăţele, atunci28


figura F (k +1) este constituită dintr-unpătrăţel, adică avem o acoperire a planului.Dar nu am obţinut acelaşi lucru în demonstraţia 1 şi mult mai uşor?Ba da, dar demonstraţia 1 nu ne perminte o rafinare pentru a o face să funcţionezeşi pe figuri de cardinal par, pe când aceasta da.Dacă lasfârşit nu obţinem un pătrăţel, ci mulţimea vidă, rezultă călaunmomentdat, după o simetrizare, figura s-a anulat pe sine însăşi. Dar aceasta nu se întâmplădecât în cazul în care figura era deja simetrică faţă deaxafaţă decares-afăcut simetria.Dar suprimând aceastăetapă din pasul corespunzător putem continua operaţiilede simetrizare, întrucât figura e deja simetrică faţă deaceaaxă. Suprimând toateetapele în care am face o astfel de simetrie nedorită, ne păstrăm pe linia raţionamentuluianterior şi,maimult,putemfisigurică ajungem la o figură unitarănevidă. Săconsiderăm, de exemplu, ultimul pas, plecând de la cele 16 posibilităţi şi suprimândacele simetrii care nu sunt favorabile.Exemplul 5. 1 11 1STOPExemplul 6.1 11 00 01 11 11 1STOPExemplul 7. 1 1 1 1 1 1STOP0 0 0 0 1 1Exemplul 8. 1 0 1 1STOP1 0 1 1Exemplul 9. 1 0 1 1 1 1STOP0 0 0 0 1 1Nu este greu de observat că fără a suprima nici o simetrizare avem exact 4 kplantaţii peste fiecare pătrăţel. Acest număr reprezintă toate pătrăţelele pătratului2 k × 2 k , deci ne furnizează omăsură mai mare decât cea furnizată dedemonstraţia1 pentru figuri de cardinal impar. Pentru cardinal par în schimb, putem consideraaceastămăsură ca un bun majorant pentru valoarea minimă posibilăamăsurii. Chiarmai mult, ţinând cont că am suprimat cel puţin o etapă, înseamnă cănuammaidublat în momentul acela măsura, deci obţinem cel mult 2·4 k−1 care pentru figuri deajuns de compacte şi de bine încadrate în pătratul de latură 2 k corespunzător poatefi chiar mai mic decât cardinalul plantaţiei.Problemă. Colorăm planul în alb şi negru ca tabla de şah. Fie o plantaţiepentru care numărul de pătrăţele albe pe care le conţine, A, esteimparşi numărulde pătrăţelenegrepecareleconţine, B, estetotimpar.Să se gasească o acoperire aplanului cu figura reprezentată deaceastăplantaţie cu măsura egală cumax(A, B).Conjenctură. Putem acoperi modulo 2 planul cu translaţii ale unei aceleeaşiplantaţii, oricare ar fi aceasta. Mai mult, măsura obţinută poate fi mai mică decâtcea dată dedemonstraţia 2 şi chiar decât cardinalul plantaţiei.Un început de demonstraţie pentru conjenctură este sugerat de problema anterioară.29


Metoda normăriiMarian TETIVA 1Introducere. În această notăvremsădăm câteva exemple de utilizare a metodeinormării, pe care am preluat-o, cu tot cu acest nume, din excelenta carte [3]; ampornit de la faptul că acolo nu există prea multe aplicaţii şi, la început mai multîn glumă, am demonstrat pe această cale câteva inegalităţi (nu tocmai uşoare). Cutimpul s-au adunat din ce în ce mai multe asemenea inegalităţi (şidinceîncemaigrele). Metoda s-a dovedit extrem de eficientă pentru demonstrarea inegalităţiloromogene dar şi pentru obţinerea unor identităţi altfel greu de găsit, iatădecevremsăo prezentăm aici; totuşi, trebuie s-o spunem, metoda normării nu este recomandatăcelor care au "alergie" la calcule: este pentru cei răbdători şi stăpâni pe tehnicamatematică (ne referim la calculul elementar). De asemenea, se aplică inegalităţilorîn care variabilele implicate sunt numere reale pozitive (sau nenegative).Să începem cu o inegalitate foarte cunoscută (asevedeaşi [3], capitolul 6):a 2 + b 2 + c 2 ≥ ab + ac + bc;se ştie căaceastaevalabilă pentru orice numere reale a, b, c, dar noi o vom demonstra(complicat, veţi spune, dar e numai pentru a da un exemplu) doar pentru a, b, c ≥ 0(iată un dezavantaj; nu unul mare, pentru că majoritatea inegalităţilor care vorurma prezintă interes pentru cazul variabilelor nenegative şi, uneori, se pot extinde,plecând de la acesta, la orice valori reale ale variabilelor). Datorită simetriei, putempresupune fără aparticulariza,că c =min{a, b, c}. Pentru c =0inegalitatea esteevidentă:a 2 + b 2 ≥ ab ⇔ (a − b) 2 + a 2 + b 2 ≥ 0.Fie c>0 şi să notăm a c =1+x, b =1+y. Conform presupunerilor făcute, avemcx ≥ 0 şi y ≥ 0, iar inegalitatea de demonstrat devine (după împărţirea cu c 2 şi cunoile notaţii)(1 + x) 2 +(1+y) 2 +1≥ (1 + x)(1+y)+1+x +1+y ⇔ x 2 + y 2 ≥ xy,evidentă (ca mai sus) chiar pentru orice x, y ∈ R, nu doar pentru x, y ≥ 0. Se vedeuşor că egalitatea are loc doar pentru x = y =0,decinumaidacă a = b = c. Şi maimult, plecând de la(1 + x) 2 + ¡ 1+y 2¢ +1− (1 + x)(1+y) − 1 − x − 1 − y = x 2 + y 2 − xyşi revenind la a, b, c (cu x = a − cc, y = b − c )găsim identitateaca 2 + b 2 + c 2 − ab − ac − bc =(a − c) 2 +(b − c) 2 − (a − c)(b − c) ,care permite demonstrarea inegalităţii pentru orice a, b, c ∈ R şi chiar obţinerea uneirafinări:a 2 + b 2 + c 2 − ab − ac − bc ≥ 1 ³(a 2´− c) 2 +(b − c) , a,b,c∈ R !21 Profesor, Colegiul Naţional "Gheorghe Roşca Codreanu", Bârlad30


Desigur, toate acestea se puteau face şi altfel şi sunt cunoscute, dar . . . nu e tocmaidemonstraţia obişnuită a acestei inegalităţi, nu-i aşa?Mai departe să considerăm inegalitateaa 4 + b 4 + c 4 ≥ a 3 b + b 3 c + c 3 a, a, b, c ≥ 0,căreia îi aplicăm acelaşi tratament. Simetria (de data asta, doar circulară) ne permitesă presupunem, fără a restrânge generalitatea, că c =min{a, b, c}; c =0ne lasăinegalitatea în forma a 4 + b 4 ≥ a 3 b, pentru orice a, b ≥ 0 (exerciţiul 1: demonstraţiacest caz particular!). Mai departe fie c>0 şi să facem aceleaşi notaţii ca mai sus.Împărţim cu c 4 şi inegalitatea devine(1 + x) 4 +(1+y) 4 +1≥ (1 + x) 3 (1 + y)+(1+y) 3 +1+x,de demonstrat pentru x, y ≥ 0. Calcule simple (exerciţiul 2: verificaţi-le!) o transformăîn3 ¡ x 2 + y 2 − xy ¢ +3 ¡ x 3 + y 3 − x 2 y ¢ + x 4 + y 4 − x 3 y ≥ 0;aceasta este adevărată, ba chiar se poate întări, ţinând cont dex 2 + y 2 − xy ≥ 1 ¡x 2 + y 2¢ ,2dex 3 + y 3 − x 2 y ≥ xy 2 (⇔ (x − y) 2 (x + y) ≥ 0)şi dex 4 + y 4 − x 3 y ≥ xy 3 (⇔ (x − y) 2 ¡ x 2 + xy + y 2¢ ≥ 0).Astfel am obţinut de fapt(1 + x) 4 +(1+y) 4 +1− (1 + x) 3 (1 + y) − (1 + y) 3 − 1 − x ≥≥ 3 ¡x 2 + y 2¢ +2xy 2 + xy 3 , x,y ≥ 0;2aici să revenim la variabilele iniţiale a, b, c şi să înmulţim cu c 4 . Exerciţiul 3: arătaţică ajungem la inegalitateaa 4 + b 4 + c 4 − ¡ a 3 b + b 3 c + c 3 a ¢ ≥≥ 3 2 c2 ³ (a − c) 2 +(b − c) 2´ +3c (a − c)(b − c) 2 +(a − c)(b − c) 3 ,pentru orice numere nenegative a, b, c, c fiind cel mai mic dintre ele (este importantacest lucru?). Iar exerciţiul 4 vă ceresă demonstraţi identitateaa 4 + b 4 + c 4 − ¡ a 3 b + b 3 c + c 3 a ¢ = 3 ³(a 2´− b) 2 +(a − c) 2 +(b − c) +2+3c (a − b) 2 (a + b − 2c)+3c (a − c)(b − c) 2³ +´+(a − b) 2 (a − c) 2 +(b − c) 2 +(a − c)(b − c) +(a − c)(b − c) 3(mai contează cumsuntnumerelea, b, c?) şi să obţineţi şi alte întăriri ale inegalităţiiconsiderate.Acum, că aţi cam înţeles în ce constă metodanormării şi, în plus, aţi căpătatantrenament la calcule de acest tip, puteţi exersa chiar singuri şi ceva mai serios:31


Exerciţiul 5. Arătaţi că, pentru orice numere reale nenegative a, b, c, d, cud =min{a, b, c, d} are loc inegalitateaa 4 + b 4 + c 4 + d 4 +2abcd − a 2 b 2 − a 2 c 2 − a 2 d 2 − b 2 c 2 − b 2 d 2 − c 2 d 2 ≥≥ d 2 (a − d) 2 + d 2 (b − d) 2 + d 2 (c − d) 2 +2d (a − d)(b − d)(c − d) .Deduceţi inegalitatea lui Turkevicia 4 + b 4 + c 4 + d 4 +2abcd ≥ a 2 b 2 + a 2 c 2 + a 2 d 2 + b 2 c 2 + b 2 d 2 + c 2 d 2(pentru orice a, b, c, d ≥ 0), identitate a cărei consecinţă esteşi, eventual, alte rafinăriale ei.Metoda normării şi o demonstraţie a inegalităţii mediilor. După cums-a văzut, metoda descrisă în această notă se aplică în cazul inegalităţilor simetriceşi omogene în n +1 variabile, să le spunem a 1 ,a 2 ,...,a n+1 . Simetria ne permitesă considerăm, nerestrictiv, că, de exemplu, a n+1 este cel mai mic dintre toatenumerele a 1 ,a 2 ,...,a n+1 (chiar şi simetria circulară nepermiteoasemeneapresupunere).După ceverificăm inegalitatea pentru a n+1 =0(dacă e cazul) folosimsubstituţiilea 1a 2a n=1+x i , =1+x 2 , ... , =1+x n ,a n+1 a n+1 a n+1unde, desigur, x 1 ,x 2 ,...,x n sunt nenegative (ceea ce, de obicei, ajută în noua formăainegalităţii). Se poate alege şi a n+1 =max{a 1 ,a 2 ,...,a n+1 }, dar e doar o chestiunede gust, nu schimbă esenţial calculele. Apoi, folosind transformările inversex 1 = a 1 − a n+1,x 2 = a 2 − a n+1,...,x n = a n − a n+1,a n+1 a n+1 a n+1ne putem întoarce la inegalitatea noastră pentruaobţine chiar întăriri ale ei (căci,de obicei, rămân în diferenţa dintre cei doi membri ai inegalităţii trans<strong>format</strong>e mulţitermeni nenegativi care pot fi utilizaţi în acest scop), sau identităţi interesante (careo implică). Cine a citit cu atenţie descrierea metodei în [3] a observat deja că noi amconsiderat o formă particulară a acesteia. Asta pentru că aşa am lucrat noi şi amobţinut destule rezultate interesante (chiar mai multe decât cele expuse aici). Dar,citiţi [3]şi veţi afla şi mai multe (chiar dacă în capitolul dedicat normării sunt puţineexemple)!În această secţiune am ales pentru exemplificare demonstraţia inegalităţiia n 1 + a n 2 + ···+ a n n ≥ na 1 a 2 ···a n , a 1 ,a 2 ,...,a n ≥ 0,adică ainegalităţii mediilor, o demonstraţie care s-ar putea să pară complicată (şichiar este!) faţă de multe alte demonstraţii cunoscute (căci, nu-i aşa? se cunoscfoarte multe demonstraţii ale inegalităţii mediilor); totuşi să ofacem.Cum verificarea în cazurile n ∈ {1, 2} (sau chiar n =3)numaiconstituieoproblemătrecem direct la pasul de inducţie; pe care o facem după schema: presupunemcă amdemonstratcă, pentru fiecare k ≤ n inegalitateaa k 1 + a k 2 + ···+ a k k ≥ ka 1 a 2 ···a kare loc (pentru orice a 1 ,a 2 ,...,a k ≥ 0)şi o dovedim pentru k = n+1. În inegalitateade demonstrata n+11 + a n+12 + ···+ a n+1n + a n+1n+1 ≥ (n +1)a 1a 2 ···a n a n+1 , a 1 ,a 2 ,...a n ,a n+1 ≥ 0,32


putem presupune, cum am spus, a n+1 =min{a 1 ,a 2 ,...,a n+1 }.Cuma n+1 =0ducela o inegalitate absolut banală, putem considera a n+1 > 0, săîmpărţim cu a n+1n+1 şisă facem substituţiile de mai sus; vom avea de demonstrat că(1 + x 1 ) n+1 +(1 + x 2 ) n+1 +···+(1 + x n ) n+1 +1 ≥ (n +1)(1+x 1 )(1+x 2 ) ···(1 + x n ) ,pentru orice x 1 ,x 2 ,...,x n ≥ 0.Exerciţiul 6. Arătaţi că, după câtevacalcule,nerămâne inegalitatea⎛⎞nXnX⎝C n+1k x k j − (n +1) X nXx 1 x 2 ···x k⎠ + x n+1j ≥ 0.k=2j=1Prin P x 1 x 2 ···x k înţelegem suma tuturor celor Cn k produse de câte k factori (cu indicidistincţi) aleşi dintre x 1 ,x 2 ,...,x n (pentru k = n suma conţine doar un termen,produsul x 1 x 2 ···x n ).Pentru a demonstra această inegalitate, săobservăm întâi că, dacă folosim ipotezade inducţie, avem (pentru 2 ≤ k ≤ n)x k 1 + x k 2 + ···+ x k k ≥ kx 1 x 2 ···x kşi chiar putem scrie Cn k inegalităţi de acest tip (câte una pentru fiecare grup de kdintre numerele x 1 ,x 2 ,...,x n ); adunăm toate aceste inegalităţi şi avemX ¡xk1 + x k 2 + ···+ x k ¢ Xk ≥ k x1 x 2 ···x k .Cum în membrul stâng fiecare x j , 1 ≤ j ≤ n, apare de Cn−1 k−1nXnXC k−1n−1j=1x k j ≥ k X x 1 x 2 ···x k ⇔ n +1nCk−1 n−1j=1j=1ori, de fapt am obţinutx k j ≥ (n +1) X x 1 x 2 ···x k ,pentru fiecare k ∈ {2,...,n}. Prin urmare, membrul stâng al inegalităţii de demonstratse minorează astfel⎛⎞nXnXx k j − (n +1) X nXx 1 x 2 ···x k⎠ + ≥⎝C n+1kk=2 j=1≥⎛nXµ⎝k=2C k n+1 − n +1kX nCn−1k−1j=1x k j⎞⎠ +j=1nXj=1x n+1jx n+1j .Evident, mai avem să demonstrăm că expresia din membrul drept este ≥ 0.calcul simplu ne arată căCn+1 k − n +1nCk−1 n−1 = n +1 ¡Ck−1n − C k−1 ¢ n +1n−1 = Cn−1 k−2nk= k − 1nCk n+1,deci, de fapt, ne-a mai rămas⎛⎞nX⎝ k − 1 nXnXx k ⎠j + x n+1j ≥ 0,k=2n Ck n+1j=1care este evidentă, datorită faptuluică x 1 ,x 2 ,...,x n sunt nenegative; demonstraţiaprin inducţieesteîncheiată.33j=1Un


Exerciţiul 7. Demonstraţi identitateaa n 1 + a n 2 + ···+ a n n − na 1 a 2 ···a n =Xµ n−1Xa n−kn Cnk (a j − a n ) k −n X (a 1 − a n )(a 2 − a n ) ···(a k − a n )n−1=k=1j=1X(a j − a n ) nn−1+pentru orice numere (chiar numere complexe) a 1 ,a 2 ,...,a n . (Ceade-adouasumădin paranteza mare cuprinde toate cele Cn−1 k produse de k factori aleşi dintre a 1 −a n ,a 2 − a n , ... , a n−1 − a n .) De exemplu,a 4 +b 4 + c 4 + d 4 − 4abcd =2d 2¡ 3(a − d) 2 +3(b − d) 2 +3(c − d) 2 −− 2(a − d)(b − d) − 2(a − d)(c − d) − 2(b − d)(c − d) ¢ +4d ¡ (a − d) 3 ++(b − d) 3 +(c − d) 3 − (a − d)(b − d)(c − d) ¢ +(a − d) 4 +(b − d) 4 +(c − d) 4 ,pentru orice numere (complexe) a, b, c, d. Şi atunci nu vă vafigreucuExerciţiul 8. Arătaţi că pentru orice numere reale nenegative a, b, c, d, d fiindcel mai mic dintre ele, are loc inegalitateaa 4 + b 4 + c 4 + d 4 − 4abcd ≥ 2d 2¡ (a − b) 2 +(a − c) 2 +(a − d) 2 ++(b − c) 2 +(b − d) 2 +(c − d) 2 ¢ .(Puteţi rezolva asta prin metoda normării?)În încheiere vă mai propunem câteva exerciţii (grele!). Primul dintre ele estetotuşi mai simplu şi vine în completarea celei de-a doua inegalităţi din introducere.Exerciţiul 9. Demonstraţi cea de a doua inegalitate dina 4 + b 4 + c 4 ≥ a 3 b + b 3 c + c 3 a ≥ abc (a + b + c) , a,b,c≥ 0.Nu uitaţi să căutaţi îmbunătăţiri ale acestei inegalităţi, precum şi identitatea a căreiconsecinţă este!Exerciţiul 10. Demonstraţi, folosind metoda normării, inegalitatea lui Surányi [2](n − 1) (a n 1 + an 2 + ···+ an n )+na 1a 2 ···a n ≥≥ (a 1 + a 2 + ···+ a n ) ¡ a n−11 + a n−12 + ···+ a n−1 ¢nşi obţineţi întăriri ale ei.Şi, în sfârşit, pentru ultimul exerciţiu aveţi nevoie şi de ceva inspiraţie pentru ademonstra inegalitatea la care se ajunge după normare (nu întotdeauna ea va rezultafără probleme!).Exerciţiul 11. Demonstraţi că inegalitateaa 2 (a − b)(a − 2b)+b 2 (b − c)(b − 2c)+c 2 (c − a)(c − 2a) ≥ 0arelocpentruoricenumererealea, b, c. Această inegalitate a fost publicată deVasile Cârtoaje în Gazeta Matematică, înurmăcumaimulţi ani [1].Bibliografie1. V. Cârtoaje - Problema 22694, Gazeta Matematică, seria B, 7-8/1992.2. G. Dospinescu - Oteoremăuitată - inegalitatea lui Surányi, RecMat - 1/2005.3. M. Onucu Drimbe - Inegalităţi - idei şi metode, EdituraGil,Zalău, 2003.34j=1


Asupra unei recurenţe de ordin doiGheorghe IUREA 1În [1], la pag. 59, apare următoarea problemă (autorVasile Berinde):Fie şiruldenumerereale(x n ) n∈Ndefinit prin relaţiile: x 0 ∈ R, x n+1 =2x 2 n +2x n − 1, ∀n ∈ N.a) Aflaţi numerele M ∈R cu · proprietatea că, dacă x 0 ≤M, atuncix n ≤M, ∀n∈N ∗ .b) Arătaţi că pentru x 0 ∈ − 3 2 , 1 ¸şirul (x n )2n∈Nconverge şi calculaţi lim x n.n→∞c) Determinaţi expresia lui x n în funcţie de x 0 şi n.Întrucât în rezolvarea acestei probleme datăîn[1] sunt unele scăpări, ne propunemsă neocupăm mai amănunţit cu acest şir interesant.În privinţa punctului a), fieM ∈ R cu proprietatea cerută; fie x 0 = M − t ≤ M,t ≥ 0 şi trebuie să avemx 1 ≤ M. Darx 1 =2t 2 − 2t (1 + 2M)+2M 2 +2M − 1 şi cumlim x 1 = ∞, deducem că pentrut suficient de mare avem x 1 >M. Prin urmare, nut→∞există M cu proprietatea cerută.Să observăm că recurenţa dată esteechivalentăcux n+1 + 1 µ2 =2 x n +2 1 2− 1,∀n ∈ N ∗ . Notând y n = x n + 1 2 ,obţinem y n+1 =2yn 2 − 1, y 0 = x 0 + 1 2 . Evident,studiul şirului (x n ) n∈Nse reduce la studiul şirului (y n ) n∈N.Faptul că cerinţa de la punctul b) este greşită rezultă dinurmătoareaPropoziţie. Fie (y n ) n∈Nun şir de numere reale, definit priny 0 ∈ R, y n+1 =2yn 2 − 1, n∈ N.a) Pentru y 0 ∈ (−∞, −1) ∪ (1, ∞), (y n ) n∈Neste divergent şi lim y n = ∞.n→∞b) Pentru y 0 ∈ [−1, 1], (y n ) n∈Neste convergent dacă şi numai dacă există n 0 ∈ Nastfel încât şirul (y n ) n≥n0este constant.c) y n =cos(2 n t 0t 0 ), t 0 =arccosy 0 pentru y 0 ∈ [−1, 1] şi y n = e2n + e −2n t 0,2t 0 =ln³|y 0 | + p 1´y0 2 − , pentru y 0 ∈ (−∞, −1) ∪ (1, ∞).d) Dacă M = © y 0 ∈ R; (y n ) n∈Neste convergent ª atunci:1. M este nevidă, finită.2. M =A ∪ B, unde½A= cos 2kπ¾ ½2 n ; k =0, 0 1,...,2n0 − 1 , B= cos ± 2π 3+2kπ¾; k =0, 1,...,2 n0 0 − 1 ,2 n0 0cu n 0 , n 0 0 numere naturale fixate.e) Şirul (y n ) n∈Neste convergent dacă şi numai dacă y 0 ∈ A 0 ∪ B 0 ,undev½svuA 0 t1= ±2 ± 1 112 2 2r ±···±1 2 + 1 ¾ ½su,B 0 t1= ±22 ± 1 112 2 2r ±···±1 2 + 1 ¾,4| {z }| {z }n 0 radicalin 0 0 radicali1 Profesor, Liceul Teoretic "Dimitrie Cantemir", Iaşi35


semnele ± fiind alese în toate modurile posibile.Demonstraţie. a) Dacă y 0 ∈ (−∞, −1) ∪ (1, ∞) , atunci y0 2 > 1 şi inductivrezultă că y n > 1 pentru orice n ∈ N. Cum y n+1 − y n = 2yn 2 − 1 − y n =(2y n +1)(y n − 1) > 0, rezultă că (y n ) este strict crescător. Prin urmare, există½1, − 1 2¾, imposibil, întrucâtl = lim y n;dacă l ∈ R, rezultă l =2l 2 − 1, deci l ∈n→∞y n > 1, ∀n ∈ N şi (y n ) este strict crescător. Conchidem că lim y n = ∞.n→∞½b) Fie (y n ) convergent. Dacă lim y n = l, rezultă l =2l 2 − 1, deci l ∈ 1, − 1 ¾.n→∞ 2Dacă l = 1, din relaţia de recurenţă rezultă y n+1 − 1 = 2(y n − 1) (y n +1),deci |y n+1 − 1| =2|y n − 1||y n +1|. Presupunând că y n 6=1pentru orice n ∈ N,avem călimn→∞|y n+1 − 1||y n − 1|= limn→∞ 2 |y n +1| = 4 şi din criteriul raportului rezultălimn→∞ |y n − 1| = ∞, absurd. Aşadar, există n 0 ∈N cu y n0 =1 şi atunci y n =1, n≥n 0 .În cazul l = − 1 2 scriem relaţia de recurenţăsubforma2y n+1+1 =(2y n +1)(2y n −1)şi continuăm ca mai sus.Presupunem acum că există n 0 ∈ N astfel încât (y n ) n≥n0este constant. Din½relaţia de recurenţă deducem y n0 ∈ 1, − 1 ¾.Cumy n0 =1implică y n =1, n ≥ n 0 ,2rezultă că (y n ) n∈Neste convergent cu limita egală cu1. La fel, dacă y n0 = − 1 2deducem y n = − 1 2 , n ≥ n 0 şi lim y n = − 1n→∞ 2 .c) Se demonstrează prin inducţie matematică.d) 1. Cum 1 ∈ M, rezultăcă M 6= ∅. Folosind a) şi b), y 0 ∈ M implică existenţaunui n 0 astfel încât (y n ) n≥n0este constant (egal cu 1 sau − 1 2 ). Dacă y n =1, n ≥ n 0 ,din sistemul y n0 =2yn 2 − 1, y 0−1 n 0 −1 =2yn 2 − 1, ... , y 0−2 1 =2y0 2 − 1, din aproapeîn aproape, găsim un număr finit de valori pentru y 0 . La fel se analizează cazuly n = − 1 2 , n ≥ n 0.2. y 0 ∈ M implică faptulcă y n =1, n ≥ n 0 sau y n = − 1 2 , n ≥ n0 0 (n 0 , n 0 0 numerenaturale arbitrare, dar fixate).½ Folosind c) şi y 0 ∈ M ⊂ [−1, 1], deducem cos (2 n arccos y 0 )=1, de unde y 0 ∈cos 2kπ ¾2 ; k ∈ Z . Folosind periodicitatea funcţiei cosinus rezultă că y 0 ∈ A. Lafel,n0din y n = − 1 2 , n ≥ n0 0 , deducem că y 0 ∈ B. Înconcluzie,M = A ∪ B.e) Folosind formula cos x r12 = ± 2 + 1 2 cos x, x ∈ R, deducem că A ⊂ A0 şi cummulţimile au fiecare câte 2 n 0elemente, rezultă că A = A 0 .Lafel,B = B 0 .Bibliografie1. Gh. Eckstein et al. - Olimpiadele şi concursurile de matematică IX—XII,EdituraBîrchi, 2005.36


Olimpiada Internaţională de Matematică"B. O. Zhautykov"Ediţia I, Alma-Ata, 2005Enunţuri şi Soluţii — junioriPrima zi — 13 ianuarie 20051. Pe o tablă 9 × 9 sunt marcate 40 celule. O linie orizontală sau verticală<strong>format</strong>ă din9 celule se spune că estebună, dacă ea are mai multe celule marcatedecât nemarcate. Care este cel mai mare număr de linii bune (orizontale şi verticale)pe care-l poate avea tabla?2. Arătaţi că numărul 2 2n+2 +2 m+2 +1,undem, n ∈ Z şi 0 ≤ m ≤ 2n, estepătrat perfect dacă şi numai dacă m = n.3. Fie A omulţime <strong>format</strong>ă din2n puncte dintr-un plan astfel încât oricaretrei dintre acestea nu sunt coliniare. Arătaţi că pentru orice două puncte distinctea, b ∈ A există odreaptăceîmparteA în două submulţimi conţinând n puncte fiecareşi astfel încât a şi b se află depărţi diferite în raport cu această dreaptă.A doua zi — 14 ianuarie 20054. Pentru orice numere a, b, c reale şi pozitive, arătaţi inegalitateaca +2b + ab +2c + bc +2a ≥ 1.5. Cercul înscris triunghiului ABC este tangent laturii AB în punctul D, iarM este mijlocul acestei laturi. Arătaţi că M, centrul cercului înscris şi mijloculsegmentului [CD] sunt coliniare.6. Determinaţi numerele prime p, q mai mici ca 2005 şi astfel încât p 2 +4 sedivide cu q, iarq 2 +4se divide cu p.* **1. Deoarece sunt marcate 40 celule şi o linie are celpuţin 5 celule marcate, rezultă că putem avea cel mult 8linii orizontale bune şi cel mult 8 linii verticale bune. Întotal, putem avea cel mult 16 linii bune. Alăturat este datun exemplu de tablă cu16 linii bune. Deci 16 este numărulmaxim de linii bune.2. Dacă m = n, atunci2 2n+2 +2 m+2 +1 = ¡ 2 n+1 +1 ¢ 2.Dacă m


Deoarece (x − 1,x+1)=2,numărul 2 m+1 divide una dintre parantezele din membrulstâng, iar cealaltă nuvafimaimicădecât2 m+1 − 2. Cum m ≥ 2n − m +2şi2 2n−m+1 ≥ 2, urmeazăcă2 m+1 − 2 ≥ 2 2n−m+3 − 2=4· 2 2n−m+1 − 2 > 2 2n−m+1 +2.În consecinţă, (x − 1) (x +1)≥ 2 ¡ m+1 2 m+1 − 2 ¢ > 2 ¡ m+2 2 2n−m +1 ¢ ,ceeacecontrazice(∗). Aşadar, nici în acest caz numărul dat nu-i pătrat perfect.3. Fie d ab dreapta determinată de punctele a şi b. Pe segmentul de extremităţia şi b alegem un punct O astfel încât orice dreaptă cetreceprinO şi nu coincide cud ab conţine cel puţinunpunctdinA; pentrucă A este mulţime finită, o astfel dedreaptă există.Notăm cu d ϕ dreapta obţinută rotindd ab cu unghiul ϕ (în sens contrar acelor deceasornic, de exemplu); avem d 0 = d π = d ab . Dacă d ab împarte A \{a, b} în douăsubmulţimi cu n − 1 elemente fiecare, atunci rotind-o cu un unghi ϕ suficient de micobţinem dreaptă căutată.Presupunem că într-un semiplan determinat de d ϕ împreunăcua se află m punctedin A, iar în celălalt împreunăcub se află 2n−m (m 6= n). Dacă unghiul de rotaţie vafi suficient de aproape de π, atunci situaţia se inversează: într-un semiplan împreunăcu a se află 2n − m puncte din A, încelălalt împreună cub sunt m puncte.Deoarece trecerea de la perechea (m, 2n − m) la (2n − m, m) prin rotaţie în jurullui O se face printr-o compunere de transformări de tipul (x, y) → (x ± 1,y∓ 1), vaexista o valoare ϕ 0 pentru care corespunde perechea (n, n); d ϕ0 este dreapta căutată.4. Fie a +2b = x, b +2c = y, c +2a = z. Rezolvând în raport cu a, b, c, obţinem:a = 4 9 z + 1 9 x − 2 9 y, b = 4 9 x + 1 9 y − 2 9 z, c = 4 9 y + 1 9 z − 2 x. Inegalitatea dată se rescrie9µ4 y9 x + z y + x + 1 µ zz 9 x + x y + y − 3 · 2z 9 ≥ 1.Cum sumele din paranteze sunt ≥ 3 (inegalitatea mediilor), deducem că ultima inegalitateeste adevărată.5. Dacă M coincide cu D, atuncisearatăuşor că BC = AC, adică triunghiuleste isoscel. În acest caz [CD este bisectoarea unghiului C şi coliniaritatea celor treipuncte este evidentă.Dacă M 6= D, fieE punctul diametral opus lui D pe cercul înscris şi {F } =CE ∩ AB. Se ştie că F este punctul de tangenţă culaturaAB a cercului exînscriscorespunzător acestei laturi şi că M este mijlocul segmentului [FD] (eventual,demonstraţi!). Atunci MI este linie mijlocie în 4DEF (I notează centrul cerculuiînscris). Rezultă că MI k CF şi, în final, MI trece prin mijlocul segmentului [CD].6. Dacă p = q, atunciacestenumeredivid4 şi, deci, p = q =2. În acest caz,obţinem soluţia (p, q) =(2, 2).Să determinăm soluţiile (p, q) cu p 6= q. Vom spune că perechea (x, y) de numerenaturale este admisă, dacă îndeplineşte condiţiile: (A) x, y sunt relativ prime şix ≤ y; (B) x 2 +4se divide cu y şi y 2 +4se divide cu x. Observăm că operecheadmisă este<strong>format</strong>ă din numere impare.38


Arătăm mai întâi că, dacă (x, y) este o pereche admisă, atunci perechea ¡ y, ¡ y 2 +4 ¢ /x ¢este de asemenea admisă. În acest scop, fie z = ¡ y 2 +4 ¢ /x. Deoarece xy ≤ y 2 xavem şi y ≥ x +2,putemscriex 2 +4 = ay ≥ (x +2) 2 = x 2 +4x +4,dinnouocontradicţie.În sfârşit, să scriem termenii şirului (a i ) i≥0ce nu depăşesc 2005; aceştia sunt:a 0 = a 1 =1, a 2 =5, a 3 =29, a 4 = 169 şi a 5 = 985. Sunt numere prime numai 5 şi29. Soluţiile problemei sunt perechile (p, q) ∈ {(2, 2) , (5, 29) , (29, 5)}.Enunţuri şi Soluţii — senioriPrima zi — 13 ianuarie 20051. Arătaţi că ecuaţia x 5 +31=y 2 nu are soluţii întregi.2. Fie r un număr real astfel încât pentru orice şir (a n ) n≥1de numere realepozitive are loc inegalitateaa 1 + a 2 + ···+ a m+1 ≤ ra m ,oricare ar fi m ∈ N ∗ .Arătaţi că r ≥ 4.3. Fie SABC o piramidă triunghiulară regulată, i.e. SA = SB = SC şi AB =BC = AC. Determinaţi mulţimea punctelor D (D 6= S) dinspaţiu ce satisfaccondiţia|cos δ A − 2cosδ B − 2cosδ C | =3,unde δ X = ]XSD pentru X ∈ {A, B, C}.A doua zi — 14 ianuarie 20054. Pentru orice numere a, b, c, d reale şi pozitive, arătaţi inegalitateaca +2b + db +2c + ac +2d + bd +2a ≥ 4 3 .5. Se spune că punctul X interior unui patrulater (convex) este observabil dinlatura YZ dacă piciorul perpendicularei din X pe dreapta YZ aparţine segmentului39


[YZ]. Un punct interior patrulaterului se spune că estek-punct dacă este observabildin exact k laturi ale patrulaterului (de exemplu, orice punct din interiorul unuipătrat este 4-punct). Arătaţi că, dacă în interiorul unui patrulater există un1-punct,atunci există şi un k-punct pentru k ∈ {2, 3, 4}.6. Determinaţi numerele prime p, q mai mici ca 2005 şi astfel încât p 2 +8 sedivide cu q, iarq 2 +8se divide cu p.* **1. Dacă x este par, atunci x 5 +31 ≡ 3 (mod4) şi nu poate fi pătrat perfect.Urmează că x este impar şi, deci, y este par. Mai mult, x 5 ≡ 1 (mod4) implicăx ≡ 1(mod4).Să scriem ecuaţia dată înformax 5 +2 5 = y 2 +1.Partea stângă se divide cu x +2 şi x +2≡ 3 (mod4) va avea un divizor prim detipul 4l +3. Dar, conform lemei de mai jos, numărul impar y 2 +1 are divizori priminumai de tipul 4m+1. Înconcluzie,înipotezacă ecuaţia dată araveasoluţii întregi,ajungem la o contradicţie.Lemă. Dacă y 2 +1 admite un divizor prim impar p, atuncip este de tipul 4m+1.Într-adevăr, avem y 2 ≡ −1 (modp). În conformitate cu mica teoremă aluiFermat, avem şi y p−1 ≡ 1(modp). Atunciy p−1 ≡ ¡ y 2¢ (p−1)/2≡ (−1) (p−1)/2 ≡ 1(modp) .Ultima congruenţă spune că (p − 1) /2 este par, adică p ≡ 1(mod4).2. Notăm b m = a 1 + a 2 + ···+ a m . Atunci, şirul (b n ) n≥1este strict crescător şiverifică relaţia b m+1 ≤ r (b m − b m−1 ), m ∈ N ∗ √√. Pentru c m = b m / (b m−1 r) aceastărelaţie devine c m+1 c m +1≤ c m r.Deci, pentru orice m ∈ N ∗ avemc m+1 + 1 ≤ √ r.c mUtilizând inegalitatea c + 1 ≥ 2 (c >0), obţinemcn √ µr ≥ c n+1 + 1 µ+ c n + 1 + ···+µc 2 + 1 ≥c n c n−1 c 1≥ c n+1 +2(n − 1) + 1 c 1≥ 2(n − 1) , ∀n ∈ N ∗ .µ 2 n − 1Deci r ≥ 4 , ∀n ∈ N ∗ ,şi rezultă că r ≥ 4 (numărul n − 1 poate fi oricât dennaproape dorim de 1).−→3. Fie e X versorul vectorului SX, X ∈ {A, B, C, D}. Atunci, ţinând seama căcos δ =(e D ,e X ) (produs scalar), X ∈ {A, B, C}, condiţia din enunţ sescrie1¯3 (e D,e A ) − 2 3 (e D,e B ) − 2 µ3 (e D,e C )¯ =¯ −e D , 1 3 e A − 2 3 e B − 2 3 e C¯¯¯¯ =1.40


Notăm f = 2 3 e B + 2 3 e C − 1 3 e A.Vectorulf este unitar, căci|f| 2 =(f,f) = 1 ¡4e29 B +4e 2 C + e 2 A +8(e B ,e C ) − 4(e A ,e B ) − 4(e A ,e C ) ¢ == 1 (9 + 8 cos α − 4cosα − 4cosα) =1,9unde α = ]ASB. Atunci, condiţia |(−e D , −f)| =1este echivalentă cufaptulcăvectorii e D şi f sunt coliniari.Fie SH înălţimea piramidei şi F simetricul punctului A în raport cu H. Calculaleleurmătoare arată căvectorii −→ SF şi f sunt coliniari:−→AB = −→ SB − −→ −→SA, AC = −→ SC − −→ SA, AH −−→ = 1 ³ −→ −→ AB + AC´;3−→AF =2AH −−→ = 2 ³ −→ −→ AB + AC´= 2 ³ −→ −→ −→SB + SC − 2 SA´;33−→SF = −→ SA + −→ AF = 1 ³2 −→ SB +2 −→ SC − −→ ´SA .3Ca urmare, locul geometric căutat este dreapta d SF din care se exclude punctul S.4. NotămA =ca +2b +db +2c +ac +2d +bd +2a ,B = b +2ca +2b + c +2db +2c + d +2ac +2d + a +2bd +2a ,C = a + ca +2b + b + db +2c + a + cc +2d + b + dd +2a .Constatăm uşor că 2B + C =5A +4. Conform inegalităţii mediilor, avem B ≥ 4.Ţinând seama de inegalitatea 1 u + 1 v ≥ 4 (u, v > 0), obţinemu + v4C ≥ (a + c)a +2b + c +2d +(b + d) 4b +2c + d +2a ≥ 8 3(ultima inegalitate se obţine luând x = a + c şi y = b + d îniar această inegalitate se stabileşte astfel:xx +2y +yy +2x ≥ 2 3 ,xx +2y + yy +2x = 2x2 +2xy +2y 22x 2 +5xy +2y 2 =1− 3xy2x 2 +5xy +2y 2 ≥ 1 − 3xy9xy = 2 3 ).Aşadar, 5A +4≥ 8+ 8 3 , deci A ≥ 4 3 , q.e.d.5. Să numimzonă de observaţie alaturiiXY a patrulaterului convex semibanda,notată Z XY ,careestemărginită desegmentul[XY ], perpendicularele pe aceastaduse în extremităţi şi care este situată în acelaşi semiplan cu patrulaterul. Evident,un punct interior patrulaterului este observabil din latura XY dacă şi numai dacăaparţine Z XY .Să examinăm diferitele cazuri ce apar:a) Dacă patrulaterul nu are unghiuri obtuze, atunci el este dreptunghi şi arenumai 4-puncte.41


) Presupunem că patrulaterul ABCD are un singur unghi obtuz, anume A. bAtunci zonele de observare ale laturilor BC şi CD acoperă patrulaterul şi, deci,1-puncte nu există.c) Fie ABCD cu exact două unghiuri obtuze şi vecine, anume B b şi C. b În acestcaz patrulaterul este situat în întregime în Z AD ,darşi în Z AB ∪ Z BC ∪ Z CD . Caurmare, nu există 1-puncte.d) Fie ABCD cu exact două unghiuri obtuze şi opuse, anume bB şi bD. Atuncipatrulaterul este situat atât în Z AB ∪Z BC cât şi în Z AD ∪Z CD şi nu va avea 1-puncte.e) Fie ABCD cu trei unghiuri obtuze şi fie A b unghiul său ascuţit. Atunci intersecţiaZ BC ∩Z CD este situată înpatrulaterşi formează paralelogramul LMNC (ca şiîn ABCD, vârfurile sunt notate în sensul acelor de ceasornic). Fie {E} = BM ∩ ADşi {F } = DM ∩ AB. Se constată uşor că 4ABE ⊂ Z AB şi 4AF D ⊂ Z AD . Atunci,M este 4-punct, punctele segmentului deschis (MF) sunt 3-puncte, iar cele interioarepatrulaterului AF ME sunt 2-puncte.6. Procedăm ca şi în cazul problemei J 6 (problema 6 de la juniori, prezentatămai sus).Dacă p = q, atunci numerele p şi q divid 8 şi, deci, p = q =2,adică (2, 2) estesoluţie a problemei.Să determinăm soluţiile (p, q) cu p 6= q. O pereche (x, y) de numere naturale senumeşte admisă dacă: (A) x, y sunt relativ prime şi x ≤ y; (B) x 2 +8se divide cuy, iary 2 +8se divide cu x. Mai întâi, observăm că o pereche admisă este<strong>format</strong>ădin numere impare. Apoi, ca şi în problema J 6 se demonstrează că, dacă (x, y) estepereche admisă, atunci şi perechea ¡ y, ¡ y 2 +8 ¢ /x ¢ este admisă. Acest rezultat areurmătoarele consecinţe:1) Dacă (a i ) i≥0este şirul dat de a 0 = a 1 =1şi a i+2 = ¡ a 2 i+1 +8¢ /a i (i ≥ 0),atunci orice pereche (a i ,a i+1 ) este admisă;2) Dacă (b i ) i≥0este şirul dat de b 0 =1, b 1 =3şi b i+2 = ¡ b 2 i+1 +8¢ /b i (i ≥ 0),atunci orice pereche (b i ,b i+1 ) este admisă.Să arătăm acum că orice pereche admisă are forma (a i ,a i+1 ) sau (b i ,b i+1 ) pentruun anumit indice i ≥ 0. Presupunem că arfiadevărată situaţia contrară şi fie (x, y)perechea minimală (în raport cu suma x + y) care nu-i de nici una dintre formeleprecedente. Cum ¡ x 2 +8 = ay, y 2 +8 = bx şi a, x sunt relativ prime, obţinemy 2 +8 = x2 x 2 +16 ¢ +8 ¡ a 2 +8 ¢a 2 = bx şi a 2 +8 se divide cu x. Dacă a ≤ x,atunci (a, x) este pereche admisă şi, datorită minimalităţii, avem (a, x) =(a i ,a i+1 )sau (a, x) =(b i ,b i+1 ); rezultă că (x, y) =(a i+1 ,a i+2 ) sau (x, y) =(b i+1 ,b i+2 ),încontradicţie cu presupunerea făcută. Dacă a>x, atunci x 2 +8=ay ≥ (x +2) 2 =a 2 +4x +4,deundex =1, a = y =3, din nou contradicţie.Să scriem acum termenii şirurilor (a i ) i≥0 şi (b i ) i≥0ce nu depăşesc 2005: a 0 =a 1 =1, a 2 =9, a 3 =89, a 4 = 881; b 0 =1, b 1 =3, b 2 =17, b 3 =99, b 4 = 577. Dintreaceste numere sunt prime numai 3, 17, 89, 881 şi 577. Caurmare,soluţiile problemeisunt perechile (p, q) ∈ {(2, 2) , (3, 17) , (17, 3) , (89, 881) , (881, 89)}.42


triciclete, înseamnă cănumărul roţilor bicicletelor şi maşinuţelor este 34 − 2 · 3=34 − 6=28. În acest caz avem 4 biciclete şi 5 maşinuţe. Dacă avem4 triciclete,atunci numărul roţilor bicicletelor şi maşinuţilor este 34 − 4 · 3=34− 12 = 22. Înacest caz putem avea: o bicicletă şi 5 maşinuţe sau 3 biciclete şi 4 maşinuţe sau 5biciclete şi 3 maşinuţe.P.90. Lungimea laturii unui pătrat este de 17 m. O persoană pleacă dintr-unvârf al pătratului şi, mergând în acelaşi sens pe laturile acestuia, parcurge o distanţăde 637 m. Din punctul în care a ajuns se întoarce şi parcurge 773 m. Aflaţi la cedistanţă se va situa în final persoana faţă de punctul de plecare.(Clasa a III-a)Soluţie. Perimetrul pătratului este 4 × 17m =68m.Considerăm primul sens de parcurs de la A către B. Avem10 · 68m = 680m = 637m +43m. Înseamnăcă persoanaa parcurs conturul în întregime de 9 ori, iar din al zeceleacontur a parcurs 68m − 43m =25m, oprindu-se înpunctul M. În punctul M se întoarce şi parcurge 773m.Avem 773m = 680m +68m +25m, ceea ce înseamnă căa parcurs conturul în întregime de 11 ori, iar din al doisprezeceleacontur a parcurs 25m, adică <strong>MB</strong> + BA. Înfinal călătorul ajunge din nou în punctul de unde a plecatiniţial. Distanţa ceruta este de 0m.Oxana Pascal, elevă, IaşiA 17 m DP.91. Se împart două numere naturale. Dacă împărţitorul, câtul şi restul sunttrei numere consecutive cu suma 30, săseafledeîmpărţitul.(ClasaaIV-a)Vasile Solcanu, Bogdăneşti, SuceavaSoluţie. Cele trei numere consecutive sunt 9, 10 şi 11. Formula împărţirii curest este D = I × C + R, R


ghiului iniţial.(ClasaaIV-a)Petru Asaftei, IaşiSoluţie. După primaseriedeîndoiriseobţine un dreptunghi care are lungimeaegală cu lungimea dreptunghiului iniţial, iar lăţimea este egală culaturapătratuluiobţinut în final, a cărui latură are lungimea 12m :4=3m. Deoarece dreptunghiuliniţial a fost îndoit de-a lungul de 6 ori, înseamnă călăţimea lui este 7 × 3m =21m.Ultimele 9 îndoiri sunt echivalente cu îndoirile de-a latul ale dreptunghiului iniţial,de unde rezultă că lungimea dreptunghiului iniţial este 10 × 3m =30m. Perimetruldreptunghiului iniţial este 2 · (30m +21m) =2· 51m = 102m.Clasa a V-aV.56. Se consideră numărul A =5+5 2 +5 3 + ···+5 2005 .a) Să searatecă A nu este pătrat perfect.b) Să segăsească 5 divizori mai mici decât 100 ai lui A.Andrei Tofan, elev, IaşiSoluţie. a) Evident că A . 5, însă A ./ 5 2 , prin urmare A nu poate fi pătrat perfect.b) Suma A are 2005 termeni, pe care îi vom grupa câte 5:A = ¡ 5+5 2 +5 3 +5 4 +5 5¢ + ···+ ¡ 5 2001 +5 2002 +5 2003 +5 2004 +5 2005¢ == ¡ 5+5 2 +5 3 +5 4 +5 5¢¡ 1+5 5 + ···+5 2000¢ = 3905 ¡ 1+5 5 + ···+5 2000¢ .Cum 3905 = 5 · 11 · 71, numărul A admite ca divizori pe 1, 5, 11, 55, 71.V.57. Aflaţi restul împărţirii prin 47 anumărului N = 1268 99| {z...9}.2005 cifreAlexandru Negrescu, elev, BotoşaniSoluţie. Observăm că N +1 = 1269 00| {z...0}= 1269 · 10 2005 =47· 27 · 10 2005 , deci2005 cifreN +1se divide cu 47. Atunci restul împărţirii lui N prin 47 este 47 − 1=46.V.58. Aflaţi numerele naturale x, y, z cu proprietatea că2 4x+1 +2 3y+1 +2 2z+1 = 9248.Cristian - Cătălin Budeanu, IaşiSoluţie. Deoarece 9248 10 = 10010000100000 2 , egalitatea din enunţ revine, datorităunicităţiiscrieriiunuinumăr în baza 2, lafaptulcă (4x +1, 3y +1, 2z +1)∈{(13, 10, 5) , (13, 5, 10) , (10, 13, 5) , (10, 5, 13) , (5, 10, 13) , (5, 13, 10)}. Cercetând fiecarecaz în parte, obţinem soluţii numai în prima şi penultima situaţie, anume (x, y, z) ∈{(1, 3, 6) , (3, 3, 2)}.V.59. Dacă a 1 a 2 ...a 2 2n − b 1 b 2 ...b n = c1 c 2 ...c 2 n ,săsearatecăa 1 a 2 ...a n a 1 a 2 ...a 2 2n − b 1 b 2 ...b n b 1 b 2 ...b n = c1 c 2 ...c n c 1 c 2 ...c 2 n .Petru Asaftei, IaşiSoluţie. Notăm A = a 1 a 2 ...a n , B = b 1 b 2 ...b n , C = c 1 c 2 ...c n .Observăm căa 1 a 2 ...a n a 1 a 2 ...a 2 n =(A · 10 n + A) 2 = A 2 (10 n +1) 2şi încă douărelaţii similare. Astfel, egalitatea de demonstrat revine laA 2 (10 n +1) 2 − B 2 (10 n +1) 2 = C 2 (10 n +1) 2 ,45


VI.59. Fie 4ABC cu m( B) b = 120 ◦ . Dacămediana[BM] este perpendicularăpe BC, arătaţi că AB =2BC.ABogdan Posa, elev, Motru (Gorj)Soluţie. Fie T mijlocul lui [AB]. Cum[MT] este liniemijlocie în 4ABC, rezultăcă MT = 1 BC şi MT k BC.2Însă <strong>MB</strong> ⊥ BC, deci 4BMT va fi dreptunghic în M, cum( \<strong>MB</strong>T) = 120 ◦ − 90 ◦ =30 ◦ . Prin urmare, MT = 1 2 BT,adică BC = BT, deundeconcluzia.VI.60. Fie 4ABC şi punctele E ∈ (AB), F ∈ (AC) şi M ∈ (BC) astfel încâtAE = EB, iarîntre4AEF şi 4EFM să existeocongruenţă. Să searatecă:a) F este mijlocul lui [AC];b) [AM] este mediană sauînălţime.Ioan Săcăleanu, HârlăuSoluţie. a) Fie {O} = AM ∩EF, iarT , R proiecţiilepe EF ale punctelor A, respectivM. Cum întreA4AEF şi 4EFM există ocongruenţă, cele două triunghiurivor fi echivalente, deciS AEF =S MEF ⇔ 1 2 EF ·AT = 1 EF ·MR ⇔ AT =MR.2Însă AT k MR şi rezultă că AT MR este paralelogram.De aici, O este mijlocul lui [AM], adică [EO] este linieETO R Fmijlocie în 4ABM şi atunci EO k BC. Cu reciprocateoremei liniei mijlocii în 4ABC, obţinem că F estemijlocul lui [AC].b) Deosebim trei situaţii:B M Ci) [AEF ≡ \EFM; atunciAE k FM şi, cu reciproca teoremei liniei mijlocii, rezultăcă M este mijlocul lui [BC], adică [AM] este mediană.ii) [AEF ≡ \FEM; atunci 4AET ≡ 4MER (C.U.), deci ET = ER, prin urmareT = R. Rezultă în acest caz că [AM] este înălţime.iii) [AEF ≡ \EMF; atunci, dacă [AF E ≡ \FEM vom obţine ca la i) că [AM] estemediană, iar dacă [AF E ≡ \EFM, [AM] va fi înălţime.Clasa a VII-aVII.56. Fie x, y ∈ R ∗ cu x 2 − 2y = y 2 + xy =4.Săsearatecă x 2 − 2x = y 2 .Gigel Buth, Satu MareSoluţie. Prin scăderea relaţiilor din enunţ obţinem că x 2 − y 2 = y (x +2) şi,înlocuind y = 1 ¡x 2 − 4 ¢ ,găsim că2x 2 − y 2 = 1 ¡x 3 +2x 2 − 4x − 8 ¢ . (1)2Pe de altă parte, eliminând direct pe y între relaţiile date, obţinemx 3 +2x 2 − 4x − 8=4x. (2)47TBMC


AFSoluţia 2. Fie AC = b şi = k, k ∈ (0, 1).ACîn 4AA 0 C cu transversala B − F − D, obţinem BA0AFFA 0 =2k1 − k . Deoarece 4AF E ∼ 4AA0 C, atunci AEk (1 − k)Prin urmare, DE = AE − AD =1+kdin enunţ esteechivalentăcuDin teorema lui MenelausBC · CDDA ·AC = 2k1+kAFFA 0=1, de undeşi AE =2k1+k · b.· b şi CE = AC − AE = 1 − k · b. Relaţia1+k1+kk (1 − k) = 1 k + 1+k +1,careseverifică imediat.1 − kVII.60. În cercul C se consideră coardele[AM] şi [AN] astfel încât AM < AN.1) Să se determine mulţimea punctelor X ∈ C ce îndeplinesc condiţia AM ≤AX ≤ AN.2) În ce caz mulţimea găsită este un arc de cerc?Temistocle Bîrsan, IaşiSoluţie. 1) Considerăm cercurile de centru A şi raze [AM] şi [AN]. Acesteaintersectează a doua oară cercul C în punctele M 0 şi respectiv N 0 . Punctele X cesatisfac condiţia AM ≤ AX ≤ AN se află în coroana determinată de aceste cercuri.Punctele X ∈ C ce satisfac această condiţie se află pe arcele cercului C interceptatede coroană.M ′N′M ′NAOMNAOMN′Dacă O, centrul cercului C, este exterior unghiului \MAN, atunci mulţimea căutatăesteMN∪M 0 N 0 .Dacă O este interior unghiului \MAN, atunci această mulţimeeste MN 0 ∪ M 0 N (acest caz se reduce la precedentul prin înlocuirea coardei [AN] cu[AN 0 ]).2) În ambele cazuri, obţinem un arc de cerc atunci când [AN] este diametru.Clasa a VIII-aVIII.56. Să sedeterminex, y, z ∈ R ∗ pentru care x 3 −y 3 +z 3 =8, x−y +z =2.Andrei - Sorin Cozma, elev, IaşiSoluţie. Este cunoscută identitatea(a + b + c) 3 = a 3 + b 3 + c 3 +3(a + b)(b + c)(c + a) , a,b,c∈ R.Pentru a = x, b = −y, c = z, folosind şi ipoteza, obţinem că 8=8+3(x − y)(z − y) ·(x + z), decix − y =0, z − y =0sau x + z =0. Înlocuind pe rând în a doua ecuaţie,găsim soluţiile sistemului: {(2,α,α) , (β,−2, −β) , (γ,γ,2) | α, β, γ ∈ R ∗ }.49


VIII.57. Fie a, b, c > 0 astfel încât a + b + c =1.Săsearatecă111p + p + p ≥ 9(1 − a)(1− b) (1 − b)(1− c) (1 − c)(1− a) 2 .Cristian Săvescu, elev, FocşaniSoluţie. Evident că a, b, c sunt subunitare, deci au sens radicalii. Aplicândinegalitatea mediilor MG ≤ MA,obţinem1p(1 − a)(1− b)≥11 − a +1− b2=12 − (1 − c)2= 21+cşi încă douărelaţii analoage. Folosind acum inegalitatea mediilor MH ≤ MA subPforma n 1≥ n 2± nPx i , deducem căi=1 x i i=1Xµ11p ≥ 2(1 − a)(1− b) 1+c + 11+a + 1 9≥ 2 ·1+b 3+(a + b + c) = 9 2 .Notă. Mai general, dacă a 1 ,a 2 ,...,a n > 0, a.î. a 1 + a 2 + ···+ a n = k, atunci1p(k − a1 )(k − a 2 ) + ···+ 1p(k − an )(k − a 1 ) ≥ n 2(n − 1) k .VIII.58. Fie x, y, z ∈ (0, ∞) cu x + y + z ≥ 3. Săsearatecă x n + y n + z n ≥ 3,∀n ∈ N.Romeo Ilie, BraşovSoluţiile. Pentru n = 0, concluzia este evidentă; fie deci n ≥ 1. Conforminegalităţii mediilor MA ≥ MG,areloc:x n +(n − 1) = x n +1+1+···+1| {z }≥ n · n√ x n · 1 · 1 · ...· 1=nx, (1)n−1pentru x ∈ (0, ∞) şi n ≥ 2; pentrun =1, inegalitatea (1) se transformă în egalitate.Aplicând de trei ori (1), obţinem căx n + y n + z n +3(n − 1) ≥ nx + ny + nz = n (x + y + z) ≥ 3n,de unde rezultă concluzia.VIII.59. Determinaţi x, y, z ∈ R, ştiind că x+y+z =1,iarxy+(x + y)(z +1)= 4 3 .Gheorghe Molea, Curtea de ArgeşSoluţia 1. Avem:(x + y + z) 2 =1⇔ x 2 + y 2 + z 2 +2(xy + xz + yz) =1⇔µ 4x 2 + y 2 + z 2 +23 − x − y =1⇔ x 2 + y 2 + z 2 − 2x − 2y = − 5 3 ⇔(x − 1) 2 +(y − 1) 2 + z 2 = 1 3 . (∗)Pe de altă parte, din binecunoscuta inegaliate α 2 + β 2 + γ 2 ≥ 1 3 (α + β + γ)2 (carese reduce, după calcule,la(α − β) 2 +(β − γ) 2 +(γ − α) 2 ≥ 0), pentru α = x − 1,50


β = y − 1, γ = z, obţinem(x − 1) 2 +(y − 1) 2 + z 2 ≥ 1 3 [(x − 1) + (y − 1) + z]2 = 1 3 · (−1)2 = 1 3 ,cu egalitate pentru x − 1=y − 1=z. Relaţia (∗) arată că se atinge egalitate, deciavem, într-adevăr, x−1 =y−1 =z ⇔ x = z+1, y = z+1. Înlocuind în x+y+z =1,obţinem că x = 2 3 , y = 2 3 , z = −1 3 .Soluţia 2. Eliminând pe z întreceledouărelaţii, obţinem xy+(x + y)(2− x − y) =43 sau încă 3x2 +3y 2 +3xy − 6x − 6y +4=0. Considerând această relaţie ca ecuaţiede gradul doi cu necunoescuta x, avem: 3x 2 + x (3y − 6) + 3y 2 − 6y +4=0. Cumx ∈ R, atunci ∆ x ≥ 0; iar∆ x = −3(3y − 2) 2 ≥ 0 implică y = 2 3 . Pentru y = 2 3găsim x = 2 3 şi z = −1 3 .VIII.60. Se consideră prisma triunghiulară regulată ABCA 0 B 0 C 0 cu AB = 4√ 33şi AA 0 =3 √ 3.Săsearatecă pentru fiecare număr a ∈ ¡ 0, 3 √ 3 ¢ ,există exact douăpuncte Ma, 0 Ma 00 pe dreapta CC 0 astfel încât d (B 0 , (MaAB)) 0 = d (B 0 , (Ma 00 AB)) = a.Soluţie. Fie M ∈ CC 0 şi vom nota cu x lungimeasegmentului CM, considerând x ≥ 0 dacă M ∈ [CC 0 şix


Soluţie. Conform inegalităţii mediilor,µ 4 x + y + z + t≥ xyzt ⇔ xyzt ≤ 625. (1)4Tot conform inegalităţii mediilor, obţinemxyzt = xy + xz + xt + yz + yt + zt + 475 ≥ 6 6√ xy · xz · xt · yz · yt · zt + 475 ==6 √ xyzt + 475.Cu notaţia a = √ xyzt, inegalitatea precedentă devinea 2 ≥ 6a + 475 ⇔ (a − 25) (a + 19) ≥ 0 ⇔ a ∈ (−∞, −19] ∪ [25, +∞).Însă evidentcă a>0, prin urmare a ∈ [25, +∞), i.e. xyzt ≥ 625. Rezultăcăseatinge egalitatea în (1), fapt care are loc atunci când x = y = z = t =5.IX.57. Să sedeterminetoatefuncţiile f : R → R cu proprietateaf (f (f (x)+y) · f (x − f (y))) = x 2 − y 2 , ∀x, y ∈ R.Adrian Zahariuc, elev, BacăuSoluţie. Pentru x = y =0,obţinem că f (f (f (0)) · f (−f (0))) = 0; notăma = f (f (0)) · f (−f (0)), deci f (a) =0.Pentrux = y = a, găsim căf (f (f (a)+a) · f (a − f (a))) = 0,adică f (f (a) · f (a)) = 0, prin urmare f (0) = 0.Luăm acum y oarecare şi x = f (y) şi obţinem f 2 (y) − y 2 =0,decif (x) =±x,∀x ∈ R. Considerăm mulţimile A = {x ∈ R ∗ | f(x) =x} şi B = {x ∈ R ∗ | f(x) =−x}.Dacă A, B 6= R ∗ ,există x ∈ A şi y ∈ B cu x 6= ±y şi, întorcându-ne în ecuaţia iniţială,x 2 − y 2 = f (f (f (x)+y) · f (x − f (y))) = ± (x + y) 2 ⇔ x − y = ± (x + y) ,ceea ce contrazice x 6= ±y.Rezultă că A = R ∗ sau B = R ∗ ,adică f (x) =x, ∀x ∈ R sau f (x) =−x, ∀x ∈ R.Aceste două funcţii verifică ecuaţia, deci sunt soluţii.IX.58. Fie n ∈ N, n ≥ 2. Determinaţi a 1 ,a 2 ,...,a n−1 ,c∈ R pentru care[x]+[x + a 1 ]+···+[x + a n−1 ]=[cx] , ∀x ∈ R.(În legătură cu G.42 din RecMat 1/2003.)Iuliana Georgescu şi Paul Georgescu, IaşiSoluţie. Notăm k i =[a i ], b i = {a i }, i = 0,n− 1. Cumb i ∈ [0, 1) pentru orice i,luând x =0în relaţia din ipoteză obţinem k 1 + k 2 + ···+ k n−1 =0, de unde[x]+[x + b 1 ]+···+[x + b n−1 ]=[cx] , ∀x ∈ R. (1)Să presupunem, pentru fixarea ideilor, că b 1 ≤ b 2 ≤ ···≤ b n−1 ; în orice alt caz seraţionează analog. Luând, pe rând, x =1,apoix = −1 în (1), obţinem că [c] =n,respectiv [−c] =−n, prin urmare c = n.Pentru x = −b 1 în (1), obţinem că −1 =[−nb 1 ],decib 1 ≤ 1 n .Apoi, pentrux = −b 2 în (1), obţinem că −2 =[−nb 2 ], deci b 2 ≤ 2 n şi, în continuare, b i ≤ i n ,∀i = 1,n− 1.52


CD = CE = p − c (2p = a + b + c). Cu acestepregătiri, avem:D 0 D 00 = DD 0 − DD 00 =(p − c)tgC − (p − b)tgB,E 0 E 00 = EE 00 − EE 0 =(p − c)tgC − (p − a)tgA,F 0 F 00 = FF 0 − FF 00 =(p − b)tgB − (p − a)tgAşi, ca urmare, stabilirea egalităţii E 0 E 00 = D 0 D 00 +F 0 F 00 este imediată.E′′FE′BD′D′′IDAEF′′C F′Notă. Prof. Titu Zvonaru observă că, dacă înlocuimcercul înscris cu cercul A-exînscris, concluziaproblemei rămâne adevărată. Şi pentru celelalte cercuriexînscrise are loc concluzia, în sensul că cel mai mare dintre segmente este sumacelorlaltor două.Clasa a X-aX.56. Fie tetraedrul ABCD şi M un punct în spaţiu. Dacă G, G A , G B , G C ,G D sunt centrele de greutate ale tetraedrelor ABCD, <strong>MB</strong>CD, MACD, MABD,respectiv MABC,săsearatecă −−−→ AG A + −−−→ BG B + −−−→ CG C + −−−→ DG D = −→ 0 dacă şi numaidacă M ≡ G.Marius Olteanu, Râmnicu VâlceaSoluţie (Alexandru Negrescu, elev, Botoşani). Avem:X −−−→AG A = −→ 0 ⇔ X ³ −→ −−−→ AG + GG A´= −→ 0 ⇔ X −→ X −−−→AG + GG A = −→ 0 ⇔X −−−→GG A = −→ 0 ⇔ 1 X ³ −−→ −→ −→ −−→GM + GB + GC + GD´= −→ 0 ⇔4−−→GM + 3 X −→ −→ −−→ −→ GA = 0 ⇔ GM = 0 ⇔ G ≡ M.4X.57. Dacă x, y, a ∈ (1, ∞), săsearatecă(x + y +log a x) ¡ xy +log a x x+y¢ +(x + y +log a y) ¡ xy +log a y x+y¢ ≥≥ (x + √ xy + y) 2 log a xy.Mihail Bencze, BraşovSoluţie. Observăm µ că1(x + y + z)x + 1 y + 1 =3+ x z y + y µ xx + z + z ³ z+y x + y ´≥z≥ 3+ x y + y r r r r 2x y x yµ1+x +2 y +2 x = y + .xConsiderând, pe rând, z =log a x,apoiz =log a y,obţinem:(x + y +log a x) ¡ xy +log a x x+y¢ ≥ (x + √ xy + y) 2 log a x;(x + y +log a y) ¡ xy +log a y x+y¢ ≥ (x + √ xy + y) 2 log a y.Adunăm membru cu membru cele două inegalităţi şi rezultă concluzia.X.58. Let n ∈ N, n ≥ 1. ProvethatnXµ 2nlnk k2 + n 2 µ 2n − 2n 2 < .n − 1k=1José Luis Díaz - Barrero, Barcelona, Spain54


Fn 2 Fn+1 2 (F n+1 + F n ) 2 − F 2 n+2∆ n =Fn+2 2 −2F n+1 F n+2 − Fn 2 (F n+2 − F n+1 ) 2 − Fn2 .¯−2F n F n+2 − Fn+1 2 Fn+2 2 (F n+2 − F n ) 2 − Fn+12 ¯Folosind relaţia de recurenţă, observăm că toţi termenii de pe a treia coloană suntnuli, prin urmare ∆ n =0.XI.57. Fie pătratul ABCD circumscris cercului C. În pătrat se înscrie octogonulEFGHIJKL, circumscris cercului C, astfelîncâtE,F ∈ (AB), E ∈ (AF ),G, H ∈ (BC), G ∈ (BH), I,J ∈ (CD), I ∈ (CJ), K, L ∈ (DA), K ∈ (DL). Fie{M} = EL∩ AC, {N} = FG∩ BD, {P } = HI ∩ AC, {Q} = JK ∩ BD. Săsearatecă sumaS = AEEB + BFFA + BGGC + CHHB + CIID + DJJC + DKKA + ALLD + AMMC + BNND + CPPA + DQQBnudepindedealegereavârfuriloroctogonuluipelaturilepătratului.Cătălin Calistru, IaşiSoluţie. Vom demonstra căDKKA + DQQB + DJ =1. (1)JCOdată justificată relaţia (1), scriindîncă trei egalităţi similare corespunzătoare vârfurilorA, B, C şi adunându-le, obţinem concluzia.Vom demonstra (1) analitic: fixăm un reper cu originea în centrul pătratului,astfel încât A (−1, 1), B (−1, −1), C (1, ³ −1), D (1, 1). Ecuaţia cercului esteC : x 2 + y 2 =1şi fie T (cos t, sin t), t ∈ 0, π ´, punctul de contact cu cercul al2 µdreptei KJ : x cos t + y sin t =1; obţinem că J 1, 1 − cos t µ 1 − sin t; K , 1 ;µsin tcos t1Qsin t +cost , 1.Atunci DJ sin t +cost − 1=sin t +cost JC sin t − cos t +1 ; DQ sin t +cost − 1=QB sin t +cost +1 ;DK sin t +cost − 1= şi, după calculederutinăîncarenuintervinedecâtformulaKA cos t − sin t +1fundamentală sin 2 t +cos 2 t =1,rezultăcă (1) este adevărată.Notă. Dl Cătălin Calistru, autorul problemei, face cunoscut redacţiei revisteiurmătoarea posibilă generalizare a relaţiei (1):Fie A 1 A 2 ...A n un poligon regulat, d odreaptă tangentă cercului înscris poligonuluişi P 1 , P 2 , ... , P n−1 punctele determinate prin {P i } = d ∩ A 1 A i+1 , i = 1,n− 1.Arătaţi căA 1 P+ A 1P 2+ ···+ A 1P n−1=1.P 1 A 2 P 2 A 3 P n−1 A nXI.58. Dacă n este un număr natural iar p un număr prim, atunci şirul(x n+1 (p) − x n (p)) n≥0este divergent, unde x n (p) reprezintă exponentul cu care aparenumărul p în descompunerea lui n!.Sorin Puşpană, CraiovaSoluţia 1. ¸ Notăm · ¸ y n · = x¸n+1 (p) − x n (p), n ∈ N. Conform teoremei lui Legendre·n n nx n (p) = +p p 2 +p 3 + ... .Obţinem că x p n (p) =x pn +1 (p) = pn − 1p − 1 , deci56


y p n =0şi x p n +p−1 (p) = pn − 1p − 1 ; x p n +p = pn − 1p − 1 +1, deci y p n +p−1 =1. Astfel,(y p n) n∈N şi (y p n +p−1) n∈Nsunt subşiruri cu limite diferite ale şirului (y n ), prin urmare(y n ) este divergent.Soluţia 2. Cazul p =2se tratează camaisus. Să presupunem că există p ≥ 3astfel încât (x n+1 (p) − x n (p)) n≥0să fie convergent. Atunci, conform lemei Cesaro-µ xn (p)Stolz, şiruleste convergent şi ultimele două şiruri au aceeaşi limită.nn≥1x n (p)Cum lim = 1 ,obţinem limn→∞ n p − 1 (x n+1 (p) − x n (p)) = 1n→∞ p − 1 , contradicţie:pentru p ≥ 3,1p − 1 /∈ N, iarx n+1 (p) − x n (p) ∈ N.XI.59. Fie şirul de numere supraunitare (a n ) n≥1,astfelcalim a n =1. Săsen→∞studieze continuitatea funcţiei f : R → R, f (x) = lim {xa n}, unde{x} reprezintăn→∞partea fracţionară anumărului real x.Dan Popescu, SuceavaSoluţie. Se stabileşte cu uşurinţă că:a) pentru x 0 /∈ Z, lim [x] =[x 0 ];x→x 0b) pentru x 0 ∈ Z, lim [x] =x 0 − 1, lim [x] =x 0 .x%x 0 x&x 0Cum f (x) = lim (a nx − [a n x]) = x − lim [a nx], obţinem:n→∞ n→∞1. pentru x/∈ Z, f (x) =x − [x] ={x};2. pentru x ∈ Z, f (x) =x − x =0={x}, ∀x >0 (deoarece a n x → x şi a n x>x),f (0) = 0, iarf (x) =x − x +1=1, ∀x − 2 este strict crescătoare.ln af 0 (x) =ln(1− a x ) − x ax ln a1 − a x = (1 − ax )ln(1− a x ) − a x ln a x1 − a x .Cum 1 − a x > 0 pentru x>− 2 , a ∈ (0, 1), este suficient să demonstrăm căln a(1 − a x )ln(1− a x )−a x ln a x > 0 pentru x>− 2ln a .Notăm t = ax şi cum x>− 2ln a ,a ∈ (0, 1), atunci t ∈ ¡ 0, 1/e 2¢ . Prin urmare, trebuie să demonstrăm că g (t) =(1 − t)ln(1− t) − t ln t>0 pentru t ∈ ¡ 0, 1/e 2¢ .Avemg 0 (t) =− ln (1 − t) − ln t − 2.Cum, pentru t ∈ ¡ 0, 1/e 2¢ , (1 − t) t


− ln t − ln (1 − t) − 2 > 0, deci g 0 (t) > 0 pentru t ∈ ¡ 0, 1/e 2¢ ,adică g este strictcrescătoare pe ¡ 0, 1/e 2¢ .Cumlim g (t) =0, deducem g (t) > 0, t ∈ ¡ 0, 1/e 2¢ , ceea cet&0trebuia demonstrat.Clasa a XII-aXII.56. Fie S n mulţimea permutărilor de ordin n, iarσ ∈ S n . Se considerăfuncţia mărginită f : R → R. Săsecalculeze:1limµf (σ (1)) + 1n→∞ n2 f (σ (2)) + ···+ 1 n f (σ (n)) .(O generalizare a problemei 24131, G. M. 5-6/1999.)Marius Olteanu, Râmnicu VâlceaSoluţie. Fie a n = 1 µf (σ (1)) + 1 n2 f (σ (2)) + ···+ 1 n f (σ (n)) . Deoarece feste mărginită există m, M ∈ R astfel încât m ≤ f (x) ≤ M, ∀x ∈ R. Prin urmare,µm1+ 1 n 2 + ···+ 1 ≤ a n ≤ M µ1+ 1 n n 2 + ···+ 1 nµ1şi cum lim 1+ 1n→∞ n 2 + ···+ 1 =0, deducem limna n =0.n→∞µ 1 1XII.57. Considerăm matricea A = şi mulţimea1 1G =½X a | X a = I 2 + aA, a ∈µ− 1 2 , ∞ ¾.Arătaţi că (G, ·) este grup izomorf cu (R, +). Calculaţi X 12 · X 3 2···X 2n−1 , n ∈ N ∗ .2Gheorghe Iurea, IaşiSoluţie. Deoarece A 2 =2A, deducem uşor că X a X b = X a+b+2ab şi cum a + b +2ab ∈ ¡ − 1 2 , ∞¢ pentru a, b ∈ ¡ − 1 2 , ∞¢ , înmulţireamatricelorestelegedecompoziţiepe G. (G, ·) este grup cu elementul neutru I 2 ,iarinversulfiecărui element X a ∈ Geste X − a ∈ G. Funcţia f : G → R, f (X 1+2aa)=ln(2a +1) realizează izomorfismulcerut. Fie X t = X 1,atunci:· ...· X 2n−12X 3 2 2³´ ³ ´ ³ ´ ³f (X t )=f X 1 · X 3 · ...· X 2n−1 = f X 1 + f X 3 + ···+ f2 2 222ln (1 + 2t) = ln 2 + ln 4 + ···+ln2n.Deducem că 1+2t =2· 4 · ...· 2n =2 n · n!, deci t = 2n n! − <strong>1.2</strong>XII.58. Let f : R → [−1, 1] be a continuous function. Prove that√Z 1 qZ 13 1 − (f (x)) 2 dx + f (x) dx ≤ 4.−1−1X 2n−12´⇔Zdravko Starc, Vršac, Serbia and MontenegroSoluţie. Pentru orice a ∈ [−1, 1] avem: p 3(1− a 2 )+a ≤ 2, cu egalitate pentrua = 1 2 . Prin urmare p 3(1− f 2 (x)) + f (x) ≤ 2, ∀x ∈ R; integrând această relaţieîntre −1 şi 1 obţinem inegalitatea cerută. Egalitate avem pentru f : R → [−1, 1],f (x) = 1 2 . 58


XII.59. Fie f : R → R, f derivabilă şi neconstantă peniciunintervalalluiR.Dacăf 0 (x)1+f 2 (x) ≥ f 0 (sin x)cosx + f 0 (cos x)sinx, ∀x ∈ R,demonstraţi că nuexistălimx→∞ f (x).Paul Georgescu şi Gabriel Popa, IaşiSoluţie. Pentru orice x 1 ,x 2 ∈ R, x 2 >x 1 , integrând membru cu membru inegalitateadin enunţ avemarctg f (x 2 ) − arctg f (x 1 ) ≥ f (sin x 2 ) − f (sin x 1 ) − f (cos x 2 )+f (cos x 1 ) ,prin urmare g (x 2 ) ≥ g (x 1 ), unde g (x) =arctgf (x) − f (sin x) +f (cos x), x ∈ R,deci g este monoton crescătoare pe R. Cum g este şi mărginită, deducem că existălim g (x) ∈ R. Presupunem că există lim f (x); atunciexistăx→∞ x→∞lim (f (sin x) − f (cos x)) = lim (arctg f (x) − g (x)) ∈ R.x→∞ x→∞Deoarece h (x) =f (sin x) − f (cos x) este periodică şi are limită la∞, h este constantă;există deci c ∈ R astfel încât f (sin x) − f (cos x) =c. Pentrux =0şi x = π 2găsim c = f (0) − f (1) = f (1) − f (0), decic =0,adică f (sin x) − f (cos x) =0,x ∈ R.Prin urmare g (x) =arctgf (x) şi cum g este monotonă, rezultăcă f este monotonă;dar f (0) = f (1) şi atunci f (x) =f (0), ∀x ∈ [0, 1], contradicţie.XII.60. Fie n>1 şi a 1 ,a 2 ,...,a n ∈ (0, 1) astfel încât a 1 + a 2 + ···+ a n =1.Săf (x)se determine funcţiile f : R → R, dacă lim =1şi f (x) =f (a 1 x)+f (a 2 x)+x→0 x···+ f (a n x) pentru orice x ∈ R.Gabriel Dospinescu, ParisSoluţie. Vom demonstra prin inducţie căXf (x) =k 1,...,k n≥0k 1 +k 2 +···+k n =pp!³k 1 !k 2 ! ···k n ! f a k 11 ak 22 ···ak n n xpentru orice x şi orice p.Pentru p =1este chiar relaţia din enunţ. Presupunem că relaţia este adevăratăpentru p şi o demonstrăm pentru p +1.Înlocuim în relaţia (∗) pe rând pe x cu a 1 x,... , a n x şi însumăm relaţiile obţinute, ţinând cont de relaţia din enunţ; rezultă:f (x) ==Xk 1 ,...,k n ≥0k 1 +k 2 +···+k n =p+Xk 1 ,...,k n ≥0k 1+k 2+···+k n=pXk 1,...,k n≥0k 1 +k 2 +···+k n =pp!³k 1 ! ···k n ! f a k1+11 ···a knn xp!³k 1 ! ···k n ! f a k11 ···ak n+1n´´+ ···+´x =p!(k 1 +1)³(k 1 +1)!···k n ! f a k 1+11 ···a k n n x59´+ ···+(∗)


+Xk 1 ,...,k n ≥0k 1+k 2+···+k n=pk 1,...,k n≥0k 1 +k 2 +···+k n =p+1p!(k n +1)³k 1 ! ···k n−1 !(k n +1)! f a k1´1 ···ak n+1n x .Notând k 1 +1=k1,... 0 ,k n +1=kn,obţinem:0X p!k 0 ³ ´1f (x) =k 0 k1 0 +···+k 1 ! ···k n! f a k0 11 ···ak n n x + ···+n=p+1X p!k 0 ³ ´n+k 1 ! ···kk 1 +···+k n! 0 f a k11 ···ak0 n n x .n 0 =p+1Renotând indicii de sumare obţinem:X (k 1 + k 2 + ···+ k n ) p!f (x) =fk 1 !k 2 ! ···k n !=Xk 1 ,...,k n ≥0k 1 +k 2 +···+k n =p+1(p +1)!³´k 1 !k 2 ! ···k n ! f a k11 ak2 2 ···akn n x ,³´a k 11 ak 22 ···ak n n x =ceea ce trebuia demonstrat.f (x)Fie acum un ε > 0, arbitrar şi un a > 0 fixat. Cum lim = 1, existăx→0 xδ > 0 astfel încât dacă x ∈ (0,δ), atunci 1 − ε < f (x) < 1+ε. Deoarecexk 1 + k 2 + ··· + k n = p, atunci a k11 ak2 2 ···ak n n < (max (a 1 ,a 2 ,...,a n )) p ; dar cummax (a 1 ,a 2 ,...,a n ) < 1, avem lim (max (a 1,a 2 ,...,a n )) p =0, deci există p 0 astfelp→∞încât (max (a 1 ,a 2 ,...,a n )) p 0< δ a .Deci pentru orice x ∈ (0,a) şi orice k 1 ,... ,k n cu suma p 0 ,avem³ ´f a k11 ···ak n n x1 − ε


Soluţiile problemelor pentru pregătirea concursurilordinnr. 1/2005A. Nivel gimnazial⎧⎨ x 2 − y = u 2G76. Rezolvaţi în mulţimea numerelor naturale sistemul y 2 − z = v 2 .⎩z 2 − x = t 2Adrian Zanoschi, IaşiSoluţia 1. Dacă x =0,atunci−y = u 2 şi apoi y =0, u =0, z =0, v =0,t =0. Obţinem astfel soluţia (x, y, z, u, v, t) =(0, 0, 0, 0, 0, 0). Dacă x 6= 0atunciy 6= 0 şi z 6= 0. Cum x 2 − y este pătrat perfect mai mic decât x 2 , rezultăcăx 2 − y ≤ (x − 1) 2 ⇒ y ≥ 2x − 1. Lafelz ≥ 2y − 1 şi x ≥ 2z − 1. Adunând ultimeletrei relaţii, găsim x + y + z ≤ 3 şi cum x, y, z ∈ N ∗ ,rezultă x = y = z =1. Găsimsoluţia (x, y, z, u, v, t) =(1, 1, 1, 0, 0, 0).Soluţia 2. Fie x 6= 0(deci y 6= 0şi z 6= 0); atunci¡x 2 − u 2¢¡ y 2 − v 2¢¡ z 2 − t 2¢ = xyz, (∗)deci(x − u)(y − v)(z − t)(x + u)(y + v)(z + t) =xyz.Prin urmare (x − u)(y − v)(z − t) este număr natural nenul, deci (x − u)(y − v) ·· (z − t) ≥ 1 şi cum (x + u)(y + v)(z + t) ≥ xyz, egalitate în (∗) obţinem pentruu = v = t =0şi x − u = y − v = z − t =1, deci x = y = z =1. Avem soluţiile(x, y, z, u, v, t) ∈ {(0, 0, 0, 0, 0, 0) , (1, 1, 1, 0, 0, 0)}.G77. i) Fie a, b, c ∈ R cu a>b>c;atuncia − b + >a+2b + c.b − ca 2 − b 2ii) Fie a, b, c ∈ R cu a ≥ b ≥ c>0; atunci + c2 − b 2+ a2 − c 2≥c a b3a − 4b + c.Ioan Şerdean, Orăştiea 2Soluţie. i) Avem evident căa − b ≥ a2 − b 2a − b = a+b, iar b 2b − c ≥ b2 − c 2b − c = b +c(egalitate avem dacă b =0,respectivc =0), de unde, prin adunare, obţinem căa 2b2a − b + >a+2b+c. Inegalitatea este strictă deoarece nu putem avea b = c =0.b − cii) Majorăm pe rând cei trei termeni ai membrului stâng:a 2 − b 2 (a − b)(a + b)= ≥ c + c (a − b) =2(a − b); (1)cccc 2 − b 2 (c − b)(c + b)= ≥ a + a (c − b) =2(c − b); (2)aaa(am folosit faptul că c − b ≤ 0, schimbând sensul inegalităţii!)a 2 − c 2 (a − c)(a + c)³ a= =bbb + c (a − c) ≥ (1 + 0) (a − c) =a − c. (3)b´Adunând membru cu membru relaţiile (1), (2) şi (3), obţinem concluzia.61a 2b2


G78. Dacă a, b, c, d ∈ (0, ∞), să se demonstreze inegalitateab (a + c) c (b + d) d (a + c) a (b + d)+ + +c (a + b) d (b + c) a (d + c) b (a + d) ≥ 4.Artur Bălăucă, BotoşaniSoluţia 1. Prelucrăm fiecare raport din stânga astfel:a + c a + c 1b (a + c)c (a + b) = c = ac = a + 1 ca + b a + b 1b ab a + 1 .bÎn aceste condiţii, cu substituţiile x = 1 a , y = 1 b , z = 1 c , d = 1 , inegalitatea detdemonstrat se scrie succesiv:x + zx + y + y + ty + z + z + xz + t + t + yt + x ≥ 4 ⇔(x + z)µ 1x + y + 1z + t(x + z)(x + y + z + t)(x + y)(z + t)Însă, conform inegalităţii mediilor,(x + y)(z + t) ≤(x + y + z + t)24(x + z)(x + y + z + t)(x + y)(z + t)≥+(y + t)+⇔µ 1y + z + 1t + x(y + t)(x + y + z + t)(y + z)(t + x)≥ 4 ⇔≥ 4.1(x + y)(z + t) ≥ 4(x + y + z + t) 2 ⇔4(x + z)(x + y + z + t) 4(x + z)(x + y + z + t) 2 =x + y + z + t .Scriind încă o inegalitate similară şi adunându-le, obţinem (∗).Soluţia 2. Cu substituţiile x = 1 a ; y = 1 b ; z = 1 c ; d = 1 inegalitatea estetechivalentă cu:x + zx + y + y + ty + z + z + xz + t + t + y ≥ 4, x,y,z,t∈ (0, ∞) .t + xDara 2 1+ a2 2+ ···+ a2 n≥ (a 1 + a 2 + ···+ a n ) 2, ∀a i ,b i ∈ (0, ∞)b 1 b 2 b n b 1 + b 2 + ···+ bşi atuncinx + zx + y + y + ty + z + z + xz + t + t + yt + x ==(x + z) 2(x + y)(x + z) + (y + t) 2(y + t)(y + z) + (z + x) 2(z + t)(z + x) + (t + y) 2(t + y)(t + x) ≥(z + x + y + t + z + x + t + y) 2≥(x + y)(x + z)+(y + t)(y + z)+(z + t)(z + x)+(t + y)(t + x) =4.G79. Dacă x, y, z ∈ (0, ∞) sunt astfel încât x + y + z = xyz, atuncixy + yz + zx ≥ 3+ p x 2 +1+ p y 2 +1+ p z 2 +1.Florina Cârlan şi Marian Tetiva, Bârlad62(∗)


Soluţia 1 (a autorilor). Plecând de la cunoscuta a 2 + b 2 + c 2 ≥ ab + bc + ac,avem:1x 2 + 1 y 2 + 1 z 2 ≥ 1xy + 1xz + 1yz =1⇔ x2 y 2 + x 2 z 2 + y 2 z 2 ≥ x 2 y 2 z 2 ⇔(xy + xz + yz) 2 ≥ 2xyz (x + y + z)+x 2 y 2 z 2 =3(x + y + z) 2 .Mai departe,(xy + xz + yz − 3) 2 =(xy + xz + yz) 2 − 6(xy + xz + yz)+9≥≥ 3(x + y + z) 2 − 6(xy + xz + yz)+9=3 ¡ x 2 + y 2 + z 2¢ +9şi(xy + xz + yz)(x + y + z) ≥ 9xyz ⇒ xy + xz + yz ≥ 9 > 3,prin urmare xy + xz + yz ≥ 3+ p 3(x 2 + y 2 + z 2 )+9.Se arată însăuşor căp3(x2 + y 2 + z 2 )+9≥ p x 2 +1+ p y 2 +1+ p z 2 +1,de unde concluzia rezultă prin intercalare.Soluţia 2 (a autorilor). Avemxyz = x + y + z ≥ 2 √ xy + z ⇒ z ( √ xy) 2 − 2 √ xy − z ≥ 0.Rădăcina pozitivă atrinomuluizt 2 − 2t − z fiind 1+√ 1+z 2,obţinem de aici căz√ 1+ √ 1+z 2xy ≥ ⇔ z √ xy ≥ 1+ p 1+zz2 .Scriind încă două inegalităţi analoage, prin sumare găsim:xy + xz + yz ≥ x √ yz + y √ xz + z √ xy ≥ 3+ p x 2 +1+ p y 2 +1+ p z 2 +1.Soluţia 3 (Gheorghe Iurea, Iaşi). Din ipoteză xy − 1= x + y > 0, decizxy > 1. Analogyz > 1, zx > 1. Deasemeni,(xy − 1) (yz − 1) = xy 2 z − xy − yz +1=y (x + y + z) − xy − xz +1=y 2 +1 (∗)şi încă douărelaţii analoage.În inegalitatea a 2 + b 2 + c 2 ≥ ab + ac + bc, adevărată pentruoricea, b, c numerereale, punem a = √ xy − 1, b = √ yz − 1, c = √ zx − 1 şi folosind relaţiile (∗) obţineminegalitatea cerută.Egalitate pentru a = b = c, deci xy = yz = zx sau încă x = y = z; folosindipoteza, găsim x = y = z = √ 3.Soluţia 4 (Paul Georgescu, Iaşi). Deoarece x, y, z ∈ (0, ∞) există a, b, c ∈(0,π/2) astfel încât x =tga, y =tgb, z =tgc. Folosind relaţia din ipoteză găsimtg (a + b + c) =0şi apoi a + b + c = π.Inegalitatea de demonstrat este echivalentă cu:tg a tg b +tgb tg c +tgc tg a ≥ 3+ 1cos a + 1cos b + 1cos c . (∗)63


cos (a + b)Cum tg a tg b − 1=−coa cos b = cos ccos a cos bcos ccos a cos b +cos bcos a cos c +, (∗) este echivalentă cucos acos b cos c ≥ 1cos a + 1cos b + 1cos csau încă:cos 2 a +cos 2 b +cos 2 c ≥ cos a cos b +cosb cos c +cosa cos c,adevărată pe baza inegalităţii x 2 + y 2 + z 2 ≥ xy + yz + zx, ∀x, y, z ∈ R.Egalitate dacă cos a =cosb =cosc, decia = b = c = π 3 şi apoi x = y = z = √ 3.G80. Fie A mulţimea tuturor sumelor de tipul ±1 2 ± 3 2 ± 5 2 ±···±(2n +1) 2 ,n ∈ N, undesemnele± pot fi alese în orice combinaţie posibilă. Să searatecăA = Z. (În legătură cu teorema Erdös-Surányi.)Petru Asaftei, IaşiSoluţie. Să observăm că (2k +1) 2 − (2k +3) 2 − (2k +5) 2 +(2k +7) 2 =16,∀k ∈ N. Prin urmare, dacă unnumăr întreg n se scrie sub forma n = ±1 2 ± 3 2 ±···±(2k − 1) 2 pentru o anumită alegereasemnelor+, −, numărul n +16 admite şiel o sciere de aceeaşi formă:n +16=±1 2 ± 3 2 ±···±(2k − 1) 2 +(2k +1) 2 − (2k +3) 2 − (2k +5) 2 +(2k +7) 2 .De asemeni, dacă n are o exprimare de forma dată, atunci şi −n au o exprimarede aceeaşi formă (obţinută din exprimarea pe care o are n, prin schimbarea tuturorsemnelor).Notăm cu P (n), n ∈ N, afirmaţia: "n areoexprimaredeforman = ±1 2 ±3 2 ±···±(2k − 1) 2 , k ∈ N ∗ ". Cum P (n) ⇒ P (n + 16), conform unei variante deinducţie matematică este suficient să verificăm P (0), P (1), ... ,P (15):0=1 2 − 3 2 − 5 2 +7 2 − ¡ 9 2 − 11 2 − 13 2 +15 2¢ ,1=1 2 ,2=1 2 +3 2 +5 2 − 7 2 +9 2 − 11 2 − 13 2 +15 2 ,3=1 2 +3 2 +5 2 +7 2 − 9 2 ,X h4=1 2 +3 2 +5 2 +7 2 −(2k +1) 2 − (2k +3) 2 − (2k +5) 2 +(2k +7) 2i ,k∈{4,8,...,20}5=1 2 +3 2 +5 2 +7 2 +9 2 − Xk∈{5,9,...,41}6=−1 2 − 3 2 +5 2 − 7 2 − 9 2 +11 2 ,7=1 2 +3 2 +5 2 + ···+13 2 − Xk∈{8,12,...,116}h(2k +1) 2 − (2k +3) 2 − (2k +5) 2 +(2k +7) 2i ,h(2k +1) 2 − (2k +3) 2 − (2k +5) 2 +(2k +7) 2i ,8=1 2 − 3 2 +5 2 − 7 2 − 9 2 +11 2 ,9=−7+16=−1 2 − 3 2 − ···− 13 2 +X h+(2k +1) 2 − (2k +3) 2 − (2k +5) 2 +(2k +7) 2i +k∈{8,12,...,116}64


+ Xk=120h(2k +1) 2 − (2k +3) 2 − (2k +5) 2 +(2k +7) 2i .La fel procedăm şi cu numerele 10, 11, ... ,15.Prin urmare, orice număr natural are o exprimare de forma cerutăşi atunci oricenumăr întreg are o exprimare de forma cerută. Rezultă A = Z.G81. Fie n ∈ N ∗ şi k ∈ {0, 1,...,2 n − 1}. Săsearatecăexistăomulţime A ⊂ Rcu n elemente care are exact k submulţimi cu suma elementelor strict pozitivă.Adrian Zahariuc, elev, BacăuSoluţie. Fie k = a n a n−1 ...a 1 , a i ∈ {0, 1}, ∀i = 1,n,scriereaînbaza2 aluik (admitem ca primele cifre să poată fi0, astfel încât scrierea să sefacă folosind ncifre). Pentru fiecare i ∈ {0, 1,...,n− 1}, fieε i = −1 dacă a i+1 =0şi ε i =1dacăa i+1 =1;construimmulţimea A = © ε i 2 i | 0 ≤ i ≤ n − 1 ª şi vom demonstra că eaverifică proprietatea dorită.Pentru fiecare submulţime B ⊂ A,numimindicele dominant al lui B, cel mai marei ∈ {0, 1,...,n− 1} pentru care ε i 2 i ∈ B. Dacă ε i =1, atunci suma elementelor luiB este cel puţin 2 i − 2 i−1 − ···− 2 0 =1> 0. Dacă ε i = −1, atunci suma elementelorlui B este cel mult −2 i +2 i−1 + ···+2 0 = −1 < 0. În concluzie, semnul sumeielementelor lui B depinde numai de indicele dominat. Cum există 2 i submulţimi cuindicele dominant i, numărul submulţimilor cu suma elementelor strict pozitive este2 0 a 1 +2 1 a 2 + ···+2 n−1 a n = k şi astfel problema este rezolvată.G82. Un cal se află pe tabla de şah în câmpul A1 şi dorim să-l ducem pe poziţiaH8 într-un număr minim de sărituri. Aflaţi care este acest număr minim, precumşi câte trasee de lungime minimă există.Gheorghe Crăciun, Plopeni şi Gabriel Popa, IaşiSoluţie (Gheorghe Iurea, Iaşi). Pentru a ajunge din A1 în H8, calul trebuiesă câştige 7 coloane şi 7 linii, în total 14 poziţii. La fiecare săritură, el câştigă maxim3 poziţii (două linii şi o coloană sau o linie şi două coloane); rezultă că sunt necesarecel puţin 5 sărituri. Însă calul nu poate ajunge în cinci sărituri din A1 în H8: câmpulde pe care pleacă şi cel pe care soseşte au culori diferite, ceea ce implică necesitateaefectuării unui număr par de sărituri (la o săritură, calul merge de pe alb pe negrusaudepenegrupealb).Există traseede6 sărituri (de exemplu, A1 − B3 − C5 − E6 − G5 − F 7 − H8),prin urmare numărul minim de sărituri necesare este 6.87654321A B C D E F G H65876543214944610832314 59 543 44 14335 454 42 3 2 3 41 3 1 2 1833492 3 2 3 41 4 2 2 14314921 134213412133322121A B C D E F G H


Numim drum un traseu <strong>format</strong> din 6 sărituri care pleacă dinA1 şi ajunge înH8. Dupăprimasăritură putem ajunge în B3 sau în C2, iar la ultima drumultrebuie să ajungăînF 7 sau G6. Laadouăsăritură drumulajungeînunuldintrepătrăţelele haşurate pe figura 1, iar la a cincea drumul trebuie să treacăprinunuldintre pătrăţelele marcate cu ∗. Săriturile 3 şi 4 ne duc dintr-un pătrăţel haşurat înunul marcat cu ∗.În figura 2, am notat numărul săriturii cu cifră mică din dreapta sus. În centrulcăsuţelor am notat numărul de drumuri parţiale care duc din A1 până în acel pătrat.Se observă cănumărul de drumuri până într-unpătrat la săritura k este egal cu sumanumerelor pătratelor de la săritura k − 1, aflatepeunpătrat de latură 5 cu centrulîn acel pătrat. Recurent, se obţin 108 drumuri de lungime minimă.Notă. Faptul că numărul minim de sărituri este 6 este un rezultat clasic; vezi,de exemplu, L. Panaitopol, D. Şerbănescu - Probleme de teoria numerelor şi combinatoricăpentru juniori, GIL,Zalău, 2003. Problema numărului traseelor minimalenu ne este însă cunoscută.G83. Fie ABCD patrulater convex şi punctele M,N ∈ (AB), P, R ∈ (CD)astfel încât AD ∩ BC ∩ MR∩ NP = {O}. Săsearatecă BMMN · NADP · PRRC · CDAB =1.Andrei-Sorin Cozma, elev, IaşiSoluţie. Se demonstrează cuuşurinţă următoarele două proprietăţi:1. Fiind dat triunghiul ABC şi D ∈ (BC), atunci BDDC = S ABCşi BDS ACD BC = S ABD.S ABC2. Fiind dat triunghiul ABC şi D ∈ (AB), E ∈ (AC), avem că S ADE=S ABCAD · AEAB · AC . OFolosind aceste rezultate avemBM ·NACMN DP · PRRC · CDAB =PD R= BMMN · NAAB · PRRC · CDDP == S O<strong>MB</strong>· SOAN · SOPR · SOCD =S ONM S OAB S ORC S ODP A N M B= S O<strong>MB</strong>· SOAN · SOPR · SOCD OM · OB OA · ON OP · OR OC · OD= · · ·S ORC S ODP S ONM S OAB OR · OC OD · OP ON · OM OA · OB =1.G84. Fie ABCD un trapez cu ABkCD, AB < CD. Se consideră puncteleAEE ∈ (AD) şi F ∈ (BC) astfel încâtED = CF . Dreapta EF intersecteză BD şiFBAC în M, respectiv N. Săsearatecă MNEFSoluţie. Notăm k = ABCD = AOOC = BDOD66=DC − ABDC + AB . Andrei Nedelcu, Iaşi, k


paralelei prin E la AC cu CD, respectiv BD.Cum AEED = CPAE(teorema lui Thales) şiPD ED =CF, urmează căCPFB PD = CF , adică PFFB kBD. Aplicând Thales în 4EPF cu TM k PF,obţinem că ETTP = EM ET;însăMF TP = AOOC = k,deci EM = k, i.e. EM =k EF. Analog seMF k +1arată că NF =k EF. Astfel,k +1MN = EF − (EM + NF)=EF −EDATM2k MNEF ⇒k +1 EF = 1 − k CD − AB=1+k CD + AB .G85. Fie A 0 , B 0 , C 0 picioarele bisectoarelor unghiurilor 4ABC. Pelatura(BC)considerăm punctele D şi E astfel încât D ∈ (BE) şi cevienele AD şi AE suntizogonale. Să sedemonstrezecă DB 0 şi EC 0 se intersecteză peAA 0 . (În legăturăcu Propoziţia 1, p. 99, RecMat - 2/2004.)Titu Zvonaru, ComăneştiSoluţie. Notăm m(\BAD) =m( [EAC) =α şi m( \DAA 0 )=m(\EAA 0 )=β. Fie{X} = C 0 E ∩ AD şi {Y } = DB 0 ∩ AE. Folosind teorema lui Menelaus în 4ABD cutransversala C 0 XE obţinem:EDEB · BC0C 0 A · AXXD =1.DarEDEB = S DEAS BEA=şi BC0C 0 A = BC (din teorema bisectoarei).ACPrin urmare:AXXDAD · AE · sin 2βAB · AE · sin (2β + α) =PONAD sin 2βAB sin (2β + α)AB · AC · sin (2β + α)= . (1)AD · BC · sin 2βLa felAY AB · AC · sin (2β + α)= . (2)YE AE · BC · sin 2βDin teorema bisectoarei în triunghiul ADE,DA 0A 0 E = ADAE . (3)Folosind (1), (2), (3) deducem AXXD · DA0A 0 E · YE =1şi din teorema lui Ceva rezultăAYcă drepteleEX, DY , AA 0 sunt concurente.Notă. Soluţia autorului, în esenţă aceeaşi, foloseşte explicit relaţia lui Steiner.Se poate arăta în aceeaşi manieră cărezultatulrămâne valabil şi dacă D ∈ (CE).B. Nivel licealL76. Fie cercurile C 1 şi C 2 tangente interior unui cerc C în punctele distincte M,respectiv N. CercurileC 1 şi C 2 sunt secante sau tangente exterior iar axa radicală a67BFC


cercurilor C 1 şi C 2 taie cercul C în A şi B. DrepteleAM şi AN taie din nou cercurileC 1 şi C 2 în K, respectivL. Arătaţi că AB ≥ 2KL. În ce caz avem egalitate?Neculai Roman, Mirceşti (Iaşi)Soluţie. Fie C 1 = C (O 1 ,r 1 ), C 2 = C (O 2 ,r 2 ), C = C (O, r) şi {P, Q} = C 1 ∩ C 2(fig. 1). Dacă cercurileC 1 şi C 2 sunt tangente exterior (fig. 2) atunci P = Q.AAMKOPLO O12NQ<strong>MB</strong>BFig. 1 Fig. 2Din AB axă radicală a cercurilor C 1 şi C 2 rezultă că AK · AM = AL · AN, deci4AKL ∼ 4ANM, de unde obţinem \AKL ≡ \ANM, adică m(\AKL) =m( \ANM) =12 m( AM).Pe de altă parte,dinm( O\1 KM)=m( O\1 MK)=m( \OMA) =90 ◦ − 1 2 m( \MOA)=90 ◦ − 1 2 m( AM)obţinem m( \O 1 KL) =90 ◦ ,deciKL este tangentă laC 1 . Din O 1 K k OA k O 2 Lrezultă KL ⊥ O 2 L, de unde KL este o tangentă comună exterioară cercurilor C 1şi C 2 . Aplicăm teorema lui Casey pentru cercurile A, C 1 , B, C 2 (A, B degenerate)tangente interior cercului C şi obţinem:d AO1 · d BO2 + d AO2 · d BO1 = d AB · d O1O 2⇔(am notat d O1O 2lungimea tangentei comune exterioare cercurilor C 1 şi C 2 )p p p pAP · AQ · BP · BQ + AP · AQ · BP · BQ = AB · KL ⇔AB · KL =2 √ AP · BP · pAQ AP + BP AQ + BQ· BQ ⇒ AB · KL ≤ 2 · · ⇒2 2AB · KL ≤ AB2 ⇒ AB ≥ 2KL.2Dacă cercurile C 1 şi C 2 sunt tangente exterior (fig. 2), atunci avem:µ 2 AP + BPAB·KL =2AP ·BP ⇒ AB·KL ≤ 2⇒ AB·KL ≤ AB2 ⇒ AB ≥ 2KL22(egalitate dacă AP = BP).Egaliate avem dacă cercurile C 1 şi C 2 sunt tangente exterior în mijlocul segmentului[AB].L77. Fie punctele P 1 , P 2 ,...,P 13 în plan astfel încât oricare trei sunt necoliniare68KO 1OPLO 2N


şi toate au coordonate întregi. Să searatecăexistăcelpuţin un triunghi P i P j P k astfelîncât centrul său de greutate să aibă coordonate întregi.Vasile Pravăţşi Titu Zvonaru, Comăneşti (Bacău)Soluţie (Daniel Văcaru). Există trei resturi modulo 3,aşa că cel puţin 5 abscisex i dau acelaşi rest la împărţirea prin 3 (principiul cutiei!), ceea ce impune că abscisacentrului de greutate este număr întreg, oricum am alege trei indici din mulţimeaA acelor5 determinaţi mai sus. Considerăm M mulţimea resturilor modulo 3 alenumerelor {y i | i ∈ A}. Dacă M are trei elemente, alegem i, j, k ∈ A astfel încâty i + y j + y k ≡ 0+1+2 (mod3) şi problema este rezolvată. Dacă M are douăelemente, cel puţin 3 ordonate dau acelaşi rest la împărţirea prin 3 şi le alegem peele. În sfârşit, dacă M are un singur element, concluzia este imediată.Notă. Principial aceeaşi soluţie a dat Vlad Emanuel, elev, Sibiu.L78. Considerăm şirul de puncte (P n ) n∈Npe cercul trigonometric astfel încâtm( P n\ OP n+1 )=arctg 5 12 pentru orice n ∈ N, P n\ OP n+1 fiind considerat ca unghiorientat. Să searatecăpentruoricepunctP µ pe cercul trigonometric există j ∈ N1astfel încât P j ∈ Int C P, .2005Lucian - Georges Lăduncă şi Andrei Nedelcu, IaşiSoluţie. Pentru ca mulţimea {P n ,n∈ N} să fie densă pe cercul trigonometric,de unde va rezulta imediat concluzia problemei folosind lema lui Kronecker, trebuiedemonstrat că arctg 5 12∈ R \ Q. Observăm mai întâi că arctg 5 π12 = arcsin 5 13 .Presupunem prin reducere la absurd că arcsin 513 = p q π, (p, q) =1, p, q ∈ N∗ , q ≥ 2.Fie α =arcsin 5 13 . Atunci sin qα =0şi cos qα =(−1)p .Cum(cos α + i sin α) q =(−1) p ,obţinem căsin q α − Cq 2 sin q−2 α cos 2 α + Cq 4 sin q−4 α cos 4 α − ···=0.Dacă q este par, obţinem că polinomul cu coeficienţi întregi x q − Cq 2 x ¡ q−2 1 − x 2¢ +...(−1) [ q 2] 2[ C q 2] ¡ ¢q 1 − x2 [ q 2] 5are rădăcina raţională sin α = , ceea ce este absurd,13deoarece coeficientul termenului dominant este 2 q−1 , iar termenul liber este (−1) [ q 2] ,deci polinomul respectiv poate avea numai rădăcini de forma ± 12 n , n ∈ N∗ .Cazulqimpar se tratează analog.Nota. Vlad Emanuel demonstrează că, dacă a ∈ [−1, 1] ∩ Q şi arccos a ∈ Q,½πatunci a ∈ 0, ±1, ± 1 ¾.2L79. Fie a 1 ,a 2 ,...,a n ∈ R în aşa fel încât a 1 + a 2 + ···+ a n =0şimax {|a i − a j | ;1≤ i


că a 1 ≤ a 2 ≤ ··· ≤ a n . Conform identităţii lui Lagrange, obţinem ţinând seama deipotezele problemei cănXa 2 i = 1 X(a j − a i ) 2 ≤ 1 X(a j − a i ) ,nni=11≤i


Ţinem seama că într-un palindrom, odată cu gruparea de două litere consecutiveapare şi inversa sa. Atunci grupările interzise în C sunt ab, bc, cd, da, ad, dc, cb, ba.Notăm cu a n , b n , c n , d n numărul cuvintelor corecte de lungime n care se terminăîn a, respectiv în b, c, d. Deoarece orice cuvânt corect de lungime n care se terminăîn a este <strong>format</strong> dintr-un cuvânt corect de lungime n−1 care se termină înc sau a, lacare se adaugă una final, deducem că a n = a n−1 + c n−1 şi analog b n = b n−1 + d n−1 ,c n = c n−1 + a n−1 , d n = d n−1 + b n−1 . Notând cu S n numărul cuvintelor corectede lungime n, avem din cele de mai sus că S n =2S n−1 ,iarcumS 1 =4rezultă căS n =2 n+1 .Notăm acum cu P n numărul palindroamelor de lungime n. Pentrun dinenunţ, distingem două cazuri.1. n este par. Atunci P n = S [n2 ] =2[ n 2 ]+1 .h ni2. n este impar. Între cuvântul de lungime şi inversul său se mai inserează o2literă, care poate fi aleasă îndouă moduri. Atunci P n =2S [n2 ] =2[ n 2 ]+2 .Notă. Soluţie corectă s-a primit de la Vlad Emanuel, elev, Sibiu.L81. Fie n ≥ 1 un număr natural fixat. O tablă infinită deşah este coloratăîn alb şi negru în maniera obişnuită. O mulţime C de căsuţe ale tablei se numeşteconexă dacă putem ajunge din fiecare căsuţă aluiC în fiecare altă căsuţă aluiC printr-o succesiune de deplasări în C dintr-o căsuţă într-ocăsuţă vecină(cuolatură comună). Fie S omulţime conexă cu4n căsuţe. Numim raportul cromatical mulţimii S raportul dintre numărul de căsuţe albe şi numărul de căsuţe negre dinS. Să se afle cea mai mică şi cea mai mare valoare posibilă a raportului cromatic.Adrian Zahariuc, elev, BacăuSoluţie. Fie w numărul de căsuţe albe, b numărul de căsuţe negre, iar N numărulde perechi (ordonate) de căsuţe din S. Cumfiecarecăsuţă albă are cel mult 4 vecininegri în S, rezultă că fiecare căsuţă albă apare în cel mult 4 perechi, deci N ≤ 4w.Analog, N ≤ 4b.Demonstrăm acum că N ≥ 4n − 1. În acest scop, mulţimii S iseasociazăungrafîn felul următor: fiecărei căsuţe îi corespunde un nod, iar nodurile acestui graf suntunite prin muchii dacă acelecăsuţe care le corespund sunt adiacente. Graful astfelobţinut este şi el conex, N reprezintă numărul său de muchii, iar 4n numărul său devârfuri.Observăm că un graf conex cu v vârfuri are cel puţin v − 1 muchii, lucru carese poate demonstra astfel: eliminăm muchiile care aparţin unor cicluri până cândobţinem un arbore, despre care ştim că areexactv − 1 muchii. Atunci conform celorde mai sus, N ≥ 4n − 1.De aici, 4w ≥ 4n − 1, deciw ≥ n şi analog b ≥ n. Cumb + w =4n, obţinem căb ≤ 3n şi w ≤ 3n. De aici, 1 3 ≤ w ≤ 3. Rămâne acum să construim exemple în carebraportul cromatic atinge valorile extreme.Numim o figură <strong>format</strong>ădintr-ocăsuţă şi vecinii săi de sus, stânga şi dreaptaun T -dreptunghi; acesta se va numi T -dreptunghi alb sau negru după cumcăsuţadincentruestealbă sau neagră. Pentru a atinge valoarea 1 , construim o figură3<strong>format</strong>ă dinnT-dreptunghiuri negre suprapuse, în timp ce pentru a atinge valoarea71


3 construim o figură <strong>format</strong>ădinnT-dreptunghiuri albe suprapuse.Notă. Soluţie corectă s-a primit de la Vlad Emanuel, elev, Sibiu.L82. Determinaţi P, Q ∈ R [X] pentru care f : R → R, f (x) ={p (x)+sinq (x)}este periodică, unde p, q : R → R sunt funcţiile polinomiale asociate lui P ,respectivQ.Paul Georgescu şi Gabriel Popa, IaşiSoluţie. Fie T ∈ R ∗ + o perioadă aluif. Atunci {p (x + T )+sinq (x + T )} ={p (x)+sinq (x)}, ∀x ∈ R, deci p (x + T )+sinq (x + T ) − p (x) − sin q (x) ∈ Z,∀x ∈ R. Deoarece g : R → R, g (x) =p (x + T )+sinq (x + T ) − p (x) − sin q (x) estecontinuă, rămâne că ea este identic constantă, deci există k ∈ Z astfel ca p (x + T )+sin q (x + T ) − p (x) − sin q (x) =k, ∀x ∈ R.De aici, p (x + T ) − p (x) − k =sinq (x) − sin q (x + T ), ∀x ∈ R, deundededucemcă p (x + nT ) − p (x) − nk =sinq (x) − sin q (x + nT ), ∀x ∈ R şi n ∈ N ∗ , ceea cepentru x =0conduce la |p (nT ) − p (0) − nk| ≤ 2, ∀n ∈ N ∗ . De aici rezultă că Peste un polinom de grad maxim 1, deci P = aX + b, iark = aT .În concluzie, a (x + T )+b +sinq (x + T ) − ax − b − sin q (x) =aT , ∀x ∈ R, decisin q (x + T ) − sin q (x) =0, ∀x ∈ R. Aceasta implică faptulcăq (x + T ) − q (x) q (x + T )+q (x)2sin cos =0, ∀x ∈ R.22q (x + T ) − q (x) q (x + T )+q (x)Dacă şi nu sunt identic constante, atunci membrul22stângpoateaveacelmultomulţime numărabilă de zerouri, contradicţie. Rezultă deaici că q (x + T ) − q (x) =2k 1 π, ∀x ∈ R, cuk 1 ∈ Z, sauq (x + T )+q (x) =2k 2 π,∀x ∈ R, cuk 2 ∈ Z.În primul caz obţinem că Q este un polinom de grad cel mult 1, Q = a 1 X + b 1 ,a 1 ,b 1 ∈ R, cua 1 T =2k 1 π. În al doilea caz rezultă imediatcă Q ≡ c, caz oricuminclus în primul.În concluzie, P = aX + b, Q = a 1 X + b 1 , unde a şi a 1 au proprietatea că existăT ∈ R ∗ + astfel ca aT, a 1T∈ Z. Se observăimediatcă în acest caz funcţia este2πperiodică.Notă. Soluţie corectă s-a primit de la Vlad Emanuel, elev, Sibiu.L83. Să secalculeze" µlim 1+ 1 1 µ2+ 1+ 1 2 µ3+ ···+ 1+ 1 n#n+1− n .n→∞ nnnSoluţie. Maiîntâi,seobservăcăDe asemenea,nXk=1Marius Olteanu, Râmnicu Vâlceaµ1+ 1 k µk+1− n ≤ n 1+ 1 nn+1− n. (1)nn72


µ1+ 1 kk+1 1+ 1= nnµ1+ 1 1≥k+1nconform inegalităţii lui Bernoulli. Rezultă deaicicănXk=1În concluzie,µ1+ 1 nAvem în plus călimn→∞ kk+1− n ≥ n Xn Xk=1Ãn=k=1nXk=11+ 1 n1+ 1k +1· 1 ,nµ (n +1)(k +1)n (k +1)+1 − 1 =nXk=1kn (k +1)+1 =k +1+ 1 nµ1+n (k +1)+1 − 1 X n1n n (k +1)+1 .k=1µ1+ 1 k µk+1≥ 1 − 1+ 1 1nn nµ1+ 1 n!n+1− n = limnn→∞nXk=11k +1 . (2)¡ ¢1+111+n 1 n− 11=1,nlimita din urmă putându-se calcula cu ajutorul regulii lui l’Hospital, iar1nX 1limn→∞ n k +1 =0k=1conform lemei Cesarò-Stolz. Din (1) şi (2) rezultă conform criteriului cleştelui călimita din enunţ este egală cu1.Notă. Soluţie corectă s-a primit de la Vlad Emanuel, elev, Sibiu.L84. Fie n ∈ N, n ≥ 3 şinA = x>0; x = a 0 + a √ n 1 n + ···+ a √ nn−1 n n−1 ;a 0 ,a 1 ,...,a n ∈ Z; n − 1 | a 0 + a 1 + ···+ a no.Determinaţi inf A.Paul Georgescu şi Gabriel Popa, IaşiSoluţie. Fie (x k ) k≥1definit de x k =( n√ n − 1) k ;evident,x 1 ∈ A.Fie k ≥ 1. Presupunem că x k = A k 0 + Ak n√ n√1 n + ···+ A k n−1 nn−1∈ A şi demonstrămcă x k+1 ∈ A. Avemcăx k+1 = ¡ √ nn−1 ¢ x k = ¡ nA k n−1−A k ¡0¢+ Ak0 −A k √ n¡ √1¢n + ···+ Akn−2 −A k nn−1¢nn−1== A k+10 + A k+1 n√1 n + ···+ Ak+1 n√n−1 nn−1şi A k+10 + A k+11 + ···+ A k+1n−1 =(n − 1) Ak n−1 . n − 1 pentru k ≥ 1. Cum x k → 0pentru k →∞obţinem că inf A =0.Notă. Soluţie corectă s-a primit de la Daniel Văcaru.L85. Fie f : R → R ofuncţie pentru care mulţimea punctelor în care f are limităfinită lastângaestedensăînR. Săsearatecămulţimea punctelor în care f este73


continuă este de asemenea densă înR. (O mulţime D ⊂ R se numeşte densă în Rdacă orice interval deschis al axei reale conţine măcar un element din D.)Gabriel Dospinescu, Paris, şi Marian Tetiva, BârladSoluţie. Vom folosi următoarea lemă, demonstrabilă uşor cu ajutorul caracterizăriicu ε − δ a limitei (finite) a unei funcţii într-un punct t 0 ∈ R:Dacă funcţia h : R → R are limită finită lastângaînt 0 ∈ R, atunci pentru oriceε>0 există un interval compact nedegenerat J, situatlastângaluit 0 ,astfelîncât|h (x) − h (y)|


Probleme propuse 1Clasele primareP.104. Suma dintre predecesorul unui număr şi succesorul numărului următorlui este 29. Careesteacestnumăr?(Clasa I )P.105. Alăturat se află roboţelul "MATE".a) Completaţi casetele goale;b) Aflaţi suma numerelor pe care le ţine în mâini;c) Aflaţi diferenţa numerelor scrise în tălpile picioarelor.(Clasa I )Andrei Stativă, elev, IaşiIrina Luca, elevă, Iaşi3267P.106. Pentru desemnarea campioanei, echipele de hochei pegheaţă A şi B dispută unnumăr de partide până ce una dintre ele câştigă de4ori. Care este numărul maxim de partide care se pot juca, ştiind că nuauavutlocrezultate de egalitate?(ClasaaII-a)Înv. Constanţa Cristea şi Inst. Iulian Cristea, IaşiP107. Un grup de turişti a consumat 17 prăjituri şi 31 îngheţate. Ştiind că 7turişti au consumat câte o îngheţatăşi câte o prăjitură, 5 turişti au consumat numaicâte două îngheţate, iar 4 turiştinuauconsumatnimic,săseaflecâţi turişti sunt îngrup.(ClasaaII-a)Aliona Loghin, elevă, IaşiP108. Prin împărţireaadouă numere naturale rezultă câtul3 şi restul 6. Ştiindcă împărţitorul este un număr mai mic decât 10, aflaţi cele două numere.(Clasa a III-a)+13-13+1Înv. Rica Bucătariu, IaşiP.109. Figura alăturată este<strong>format</strong>ădinbeţişoare.a) Îndepărtează unsingurbeţişor pentru a obţine tot atâtea triunghiurica şi pătrate;b) Mută douăbeţişoare pentru a obţine de două orimaimultedreptunghiuri decât pătrate.(Clasa a III-a)Adina Voinescu, elevă, Iaşi A BP.110. Ce oră indică primul ceas, ştiind că acesta respectă regulaindicatădecelelalte trei?9126391263(Clasa a III-a)Veronica Corbu, elevă, IaşiP.111. Fie numărul N = abc + acb + bac + bca + cab + cba.a) Care este cea mai mică şi cea mai mare valoare a lui N?b) Câte valori diferite poate avea numărul N?(ClasaaIV-a)Oxana Pascal, elevă, IaşiP.112. În urma desfăşurării unui joc didactic matematic, învăţătorul a oferitca recompensă 44 baloane. Câte 4 baloaneauprimitunnumăr de participanţi ce1 Se primesc soluţii până ladatade31decembrie2006.759126391263H+2GIJ-3FEDC


eprezintă aşasea parte din totalul lor, câte două au primit a treia parte, iar restulparticipanţilor au primit câte un balon. Aflaţi numărul participanţilor la joc (soluţiearitmetică!).(ClasaaIV-a)Alexandra Nistor, elevă, IaşiP.113. Dan şi-a pus timbrele în clasor, câte 10 pe unele pagini, câte 30 pe altepagini şi au rămas de 4 ori mai multe pagini goale decât folosite. Dacă ar pune câte 5timbre pe fiecare pagină, toate paginile ar fi folosite. Câte pagini poate avea clasorul,ştiind că nudepăşeşte 60 (soluţie aritmetică!)?(ClasaaIV-a)Petru Asaftei, IaşiClasa a V-aV.66. Săsearatecă, oricare ar fi cifra nenulă a, numărul x =21 31a +32 a13 +43 a31se divide cu 10.Otilia Nemeş, Ocna Mureş (Alba)V.67. a) Să searatecă, scăzând din suma a 2006 numere pare consecutive sumanumerelorsituateîntreacestea,nusepoateobţine rezultatul 2006 2 .b) Să seafle2006 numere pare consecutive astfel încât, scăzând din suma lor sumanumerelor situate între ele, să seobţină 2005 2 .Marian Panţiruc, IaşiV.68. Arătaţi că nuexistă n ∈ N pentru care A n =5 n +89 să fiepătrat perfect.Iulia Pleşca, elevă, IaşiV.69. Să serezolveînN 2 ecuaţia 8 n +15 m =6+6 2 + ···+6 2006 .Alexandru Gabriel Tudorache, elev, IaşiV.70. Determinaţi a ∈ N pentru care numerele a, a +2, a +6, a +12, a +18,a +20, a +26, a +30, a +32, a +36, a +60sunt simultan prime.Lucian Tuţescu, CraiovaClasa a VI-a A 12 m BVI.66. Alăturat este desenată ogrădină avândformaunui poligon cu 7 laturi. În fiecare vârf se află câteopoartă mobilă astfel încât, în oricare două vârfurivecine,porţile să închidă perfect latura pe care acestea o determină.Să seaflelungimileporţilor.Roxana Căpăţână, elevă, Iaşi9 mG11 m6 mC7 mD6 mF 10 m EVI.67. În patrulaterul ABCD construim AP ⊥ BD, CQ ⊥ BD, P, Q ∈ BDşi fie M mijlocul lui (AC). Dacă punctele M, P , Q sunt distincte două câte două,demonstraţi că 4MPQ este isoscel.Marius Farcaş, IaşiVI.68. Fie punctele A, C, M cu m( \AMC) 6= 90 ◦ şi AC =2AM. SăsearatecăM este mijlocul lui [AC] dacă şi numai dacă 2m(\ACM) =m( \MAC).Ioan Săcăleanu, HârlăuVI.69. Să searatecă pentru orice alegere a semnelor în expresia ±1 2 ± 2 2 ±···±2006 2 , rezultatul nu se divide cu 2006.Mihail Bencze, Braşov76


3m +1VI.70. Determinaţi m, n ∈ Z pentru care a =2m +1 + n +23n +5 ∈ Z.Gheorghe Iurea, IaşiClasa a VII-aVII.66. Să serezolveînR 4 ecuaţia30 p x − y + 901 + 25 p y − z + 626 + 20 √ z − x + 401 + 9 √ t − x + 78 = 2006.Ioana Olan, elevă, IaşiVII.67. Aflaţi a, b ∈ N dacă a + b =18şi 10 a+1 − 9b +71. 81.Andrei-Sorin Cozma, elev, IaşiVII.68. Fie 4ABC dreptunghic, cu ipotenuza de lungime a, cateteleb şi c, iararia S. Dacă x, y ∈ (0, ∞), săsearatecă a2S = 2 ¡ x 2 + y 2¢dacă şi numai dacă b şixyc sunt direct sau invers proporţionale cu x şi y.Veronica Plăeşu şi Dan Plăeşu, IaşiVII.69. Fie 4MNP cu m( \NMP)=90 ◦ ;seconsideră punctele S, T, M ∈ (NS),M ∈ (PT), astfelîncâtNS =3MS, PT =3MT.Dacă {Q} = PS ∩ NT, atunci:a) QM = NP; b) QN 2 + QP 2 =5NP 2 .Dorel Luchian, IaşiVII.70. Triunghiul alăturat este considerat fix. În câte moduriputem aşeza numerele 1, 2, 3, 4, 5, 6 în cerculeţe, astfel încâtsuma numerelor de pe fiecare latură a triunghiului să fie aceeaşi?Petru Asaftei, IaşiClasa a VIII-aVIII.66. Să se demonstreze că12 4 +2 2 +1 + 13 4 +3 2 +1 + ···+ 1n 4 + n 2 +1 < n − 13n .Carmen Daniela Tamaş, BârladVIII.67. Fie 0


VIII.70. Se consideră cubul ABCDA 0 B 0 C 0 D 0 şi fie M, N mijloacele muchiilor[AB], respectiv [BC], iar{S} = AN ∩ CD, {T } = DM ∩ BC. Săseaflemăsuraunghiului <strong>format</strong> de D 0 N şi ST.Gabriel Popa, IaşiClasa a IX-aIX.66. Pentru x, y ∈ R, fie a = y + xy − x, b = x 2 + x − xy.a) Dacă a, b ∈ (−∞, 0), să se compare numerele x şi y.b) Arătaţi căexistăoinfinitate de numere raţionale x, y pentru care a, b ∈ (−∞, 0).Ionuţ Onofrei,elev,HârlăuIX.67. Fie n ≥ 2 şi a 1 ,a 2 ,...,a n ≥ 0 astfel încât(a 2 a 3 ···a n ) 2 +(a 1 a 3 a 4 ···a n ) 2 + ···+(a 1 a 2 ···a n−1 ) 2 =1.Să searatecăa 1 + a 2 + ···+ a n + a 2 a 3 ···a n + a 1 a 3 ···a n + ···+ a 1 a 2 ···a n−1 ≥ n.Adrian Zahariuc, elev, BacăuIX.68. În 4ABC se consideră cevienele [AM], [BN], [CP] concurente în T .Săse arate că TATM = TBTN = TC dacă şi numai dacă T este centrul de greutate alTP4ABC.Ovidiu Pop, Satu MareIX.69. Fie 4ABC nedreptunghic. Paralela prin B la AC şi simetrica drepteiAC în raport cu BC se intersectează înA 1 ;analogseobţin punctele B 1 şi C 1 .DacăAA 1 , BB 1 , CC 1 sunt concurente, să searatecă 4ABC este echilateral.Temistocle Bîrsan, IaşiIX.70. Să searatecă tg 15 ◦ +tg25 ◦ +tg35 ◦ +tg85 ◦ > 4.D. M. Bătineţu-Giurgiu, BucureştiClasa a X-aX.66. Notăm cu D mulţimea punctelor P (x, y) din planul xOy situate în interiorulsau pe laturile 4ABC. Fiea, b ∈ R, a 2 + b 2 6=0;definim funcţia f : D → R,f (P )=ax + by + c. SăsearatecăpentruoriceP ∈ D, avemmin {f (A) ,f(B) ,f(C)} ≤ f (P ) ≤ max {f (A) ,f(B) ,f(C)} .Adrian Corduneanu, IaşiX.67. Fie Q £√ 2 ¤ = © a + b √ 2 | a, b ∈ Q ª . Să se determine funcţiile crescătoaref : Q £√ 2 ¤ → (0, ∞) pentru care f (x + y) =f (x) f (y), ∀x, y ∈ Q £√ 2 ¤ .Dan-Ştefan Marinescu şi Viorel Cornea, HunedoaraX.68. Pe cercul trigonometric se consideră punctele A, B, C de afixe 1, ε, ε 2 ,unde ε =cos 2π 3 + i sin 2π 3 .FieM (z) un punct al cercului situat pe arcul BC ce nuconţine A. Săsearatecă ¯¯z2 z+ z +1¯¯ 2 + z +1= − .z Marian Tetiva, BârladX.69. Dacă a, b, c > 1, să se demonstreze inegalitateaa 3√ log a b+ 3√ log c a+ b 3√ log b a+ 3√ log c b+ c 3√ log c a+ 3√ log b c(a + b + c)2≤ .3Titu Zvonaru, Comăneşti78


X.70. Fie pătratul ABCD. Să se determine mulţimea∆ = © P ∈ Int ABCD | PA 2 , 2PB · PD,PC 2 sunt laturile unui triunghi ª .Cătălin Calistru, IaşiClasa a XI-aXI.66. Fie x n , n ∈ N ∗ , cel mai mic număr natural cu proprietatea că există M =a n a n−1 ...a 1 a 0 (10) cu toate cifrele nenule, astfel încât M =(n +1) n+1√ a n a n−1 ···a 0 +x n9x n .Să se calculeze limn→∞ 10 n . Valeriu Braşoveanu, BârladXI.67. Fie şirul (x n ) n≥1definit prin x 1 ∈³− π 0´4 , , x n+1 =2x n − tg x n , ∀n ≥ 1.Să se studieze existenţa limitelor lim x n şi lim n√ −xn .n→∞ n→∞Dan Popescu, SuceavaXI.68. Fie f : I → R, I ⊂ R interval, o funcţiededouăoriderivabilăcuf 00 (x) ≥ f 0 (x), ∀x ∈ I. Săsearatecă f (x) − f (a) ≥ (e x−a − 1) f 0 (a), ∀x, a ∈ I.Pentru f (x) =e αx , α ∈ (−∞, 0] ∪ [1, +∞), să se deducă inegalitatea lui Bernoulli.Dumitru Mihalache, BârladXI.69. Fie A, B ∈ M 3 (R) astfel încât det (AX + B) ≥ 0, ∀X ∈ M 3 (R). Săsearate că există C ∈ M 3 (R) pentru care A = BC.Gheorghe Iurea, IaşiXI.70. Fie a, b, c laturile unui triunghi ale cărui unghiuri au măsurile în radianiA, B, C şi care are raza cercului înscris r. Săsearatecădistanţa de la punctulM (A, B, C) la planul P : ax + by + cz + r =0este mai mare decât π√ 33 .Sorin Puşpană, CraiovaClasa a XII-aXII.66. Fie a, b ∈ R cu 0 ≤ a


) Să se studieze buna definire a şirurilor (a n ) n≥1 şi (b n ) n≥1, unde f (a n ) =1 R nn0 f (x) dx, g (b n)= 1 R n a ng (x) dx şi apoi să se calculeze lim .n0 n→∞ b nMarius Apetrii, IaşiXII.69. Fie f ∈ R [X] polinom reciproc de grad 4n +2, n ∈ N ∗ ,avândrădăciniledistincte, complexe şi nereale. Să searatecă f are cel puţin o rădăcină demodul1.Cătălin Ţigăeru, SuceavaXII.70. Fie G un grup de ordin n ≥ 4 cu proprietatea că există m ∈ N, 1


G103. Pentru a, b, c ∈ (0, 1) cu a + b + c =2,săsearatecăabc ≥ 8(1− a)(1− b)(1− c) .Alexandru Negrescu, elev, BotoşaniG104. Triunghiul ABC are m(\BAC) = 120 ◦ . Fie O ∈ (BC) astfel încât [AOeste bisectoarea unghiului \BAC. Pe [AO se ia punctul D astfel încât [BC estebisectoarea interioară aunghiului\ABD. Săsearatecă AD + BD = AB + AC şiAB + AC ≥ 4 AO.Petru Răducanu, IaşiG105. Se consideră trapezul ABCD cu bazele AB, CD (AB > CD) şi fie Ointersecţia diagonalelor trapezului. Se duce linia mijlocie MN atrapezuluişi paralelaPQ prin O la bazele trapezului (M,P ∈ (AB), N,Q ∈ (BC) ). Să se demonstrezecă trapezeleABMN şi PQCD au diagonalele respectiv paralele.Claudiu-Ştefan Popa, IaşiB. Nivel licealL96. Fie cercurile C 1 , C 2 , C astfel încât C 1 şi C 2 sunt tangente exterior în Dşi fiecare dintre ele este tangent interior lui C în B, respectiv C. Tangenta comunăinterioară cercurilor C 1 şi C 2 taie cercul C în A şi A 1 .DreaptaAB taie cercul C 1 în K,iar dreapta AC taie cercul C 2 în L. Din punctul M de pe cercul C se duc tangenteleMT 1 şi MT 2 la cercurile C 1 ,respectivC 2 (T 1 ∈ C 1 , T 2 ∈ C 2 ). Dacă MA < MA 1 ,arătaţi că MT 1 + MT 2 = A 1MA 1 D · KL şi |MT 1 − MT 2 | = AMAD · KL.Neculai Roman, Mirceşti (Iaşi)L97. Să sedemonstrezecă în orice triunghi are loc inegalitatea1m 2 a (m b + m c − m a ) + 1m 2 b (m c + m a − m b ) 2 + 1m 2 c (m a + m b − m c ) 2 ≥ 1 S 2 .I. V. Maftei şi Dorel Băiţan, BucureştiL98. Se consideră un triunghi oarecare ABC. Demonstraţi că1) sin 4 A +sin 4 B +sin 4 C ≥ 27 ³ r´3;2 R2) cos 4 A +cos 4 B +cos 4 C ≥ 3 ³ r´4 ³5 − r ´³a5 − r ´³b5 − r ´c,8 R r r runde R este raza cercului circumscris, r este raza cercului înscris, iar r a , r b , r c suntrazele cercurilor exînscrise.Oleg Faynshteyn, Leipzig, GermaniaL99. a) Care este numărul minim de puncte din plan de coordonate întregi astfelîncât, oricum ar fi alese, să existe trei puncte cu centrul de greutate de coordonateîntregi.b) Să searatecăîntr-unspaţiu n-dimensional există 2 n+1 puncte de coordonateîntregi astfel încât oricare trei dintre acestea au centrul de greutate cu cel puţin ocoodonată care nu este un întreg İrina Mustaţă, studentă, Bremen, Germania·1L100. Fie x ∈ (0, 1); arătaţi că există n ∈ N ∗ astfel încât {nx} ∈3 , 2 .3Ciprian Baghiu şi Gheorghe Iurea, Iaşi81


L101. Fie a, n ≥ 2 două numereîntregi.Săsearatecă n−1 Q a n − a kk=0 n − k ∈ Z.Adrian Zahariuc, elev, BacăuL102. Fie p =2k +1un număr prim. Atunci2kXS 1 = Cp+i−1 i ≡ 2 p − 2 ¡ mod p 2¢ kX, S 2 = Cp+i−1 i ≡ 2 − 2 p ¡ mod p 2¢ .i=k+1Marius Pachiţariu, elev, IaşiL103. Fie a, b, c, d reale astfel încât ¡ 1+a 2¢¡ 1+b 2¢¡ 1+c 2¢¡ 1+d 2¢ =16.Arătaţi că−3 ≤ ab + bc + cd + da + ac + bd − abcd ≤ 5.Mai mult, avem egalitate în cel puţin una din inegalităţile de mai sus dacă şi numaidacă a + b + c + d = abc + bcd + cda + dab.Gabriel Dospinescu, student, ParisL104. Fie x 0 > 0 şi x n = x [n2 ] + x [ n 3 ] + n , pentru orice n>0.³ 6xn´a) Să searatecăşirul este convergent la 1.n n≥15b) Să searatecădacă α>log 3 , atunci lim2 n→∞i=1x n − nn α =0.Gabriel Dospinescu, student, ParisL105. Să se determine toate funcţiile continue f :(0, ∞) → R, careverificăecuaţia funcţionalănx n−1 f (x n )=(x +1)f (x) , ∀x ∈ (0, ∞) ,unde n ∈ N ∗ , n fixat.Marian Tetiva şi Dumitru Mihalache, BârladTraining problems for mathematical contestsA. Junior highschool levelG96. Let a = x 12m + x 12n ,withm, n ∈ N ∗ .Provethata is divisible by 13 if andonly if x is divisible by 13.Artur Bălăucă, BotoşaniG97. Find a, b ∈ {0, 1, 2,...,9}, a 6= 0, such that A = abb| {z. . .}b, n>2 is a perfectsquare.n timesGheorghe Iurea, IaşiG98. Find m, n ∈ N ∗ such that m n + n +1m 2 ∈ N ∗ .Gabriel Dospinescu, student, ParisG99. Let m, n two positive integer such that m divides n − 1. All positiveintegers between 1 and n are put on a circle in an arbitrary way. One compute thesum of any set of m neighbors numbers. Prove that among all these sums, there aretwo of them for which their difference is strictly grater than m − 1.Titu Zvonaru, Comăneşti82


G100. In how many ways could one colour a square 3 × 3 such that in any littlesquare 2 × 2 to be four different colours?Gabriel Popa, IaşiG101. Prove the following inequalityÃ!14a (1 + bc) 2 + 1b (1 + ca) 2 + 116c (1 + ab) 2 ≤ 1+(1 + bc)(1+ca)(1+ab) ,∀a, b, c ∈ (0, ∞) under the condition abc =1. When does the equality holds?Gabriel MîrşanuandAndreiNedelcu,IaşiG102. Find the maximal value of the parameter m ∈ R ∗ + such thatb 2 + c 2+ c2 + a 2+ a2 + b 2≥ m p 3(aa b c2 + b 2 + c 2 ), ∀a, b, c ∈ R ∗ +.Dorel Băiţan and I. V. Maftei, BucureştiG103. For a, b, c ∈ (0, 1) and a + b + c =2, show thatabc ≥ 8(1− a)(1− b)(1− c) .Alexandru Negrescu, highschool student, BotoşaniG104. The triangle ABC has m(\BAC) = 120 ◦ .LetO ∈ (BC) such that [AO isthe angle bisector of \BAC. Let D be a point on [AO such that [BC to be interiorangle bisector of \ABD. ProvethatAD + BD = AB + AC and AB + AC ≥ 4 AO.Petru Răducanu, IaşiG105. Let ABCD be a trapezium with AB, CD (AB > CD) as the bases andconsider that the diagonals of the trapezium intersect in O. WeconstructMN to bethe mean line of the trapezium and the parallel PQ, through O, to the bases of thetrapezium (M,P ∈ (AB), N,Q ∈ (BC) ). Prove that the trapeziums ABMN andPQCD have the diagonal respectively parallel.Claudiu-Ştefan Popa, IaşiB. Highschool levelL96. Let C 1 , C 2 , C such that C 1 and C 2 touch each other externally in D andeach of them is interior tangent to C in B and C, respectively. The common interiortangent to the circles C 1 and C 2 cuts the circle C in A and A 1 . The line AB cuts thecircle C 1 in K and the line AC cuts the circle C 2 in L. FromthepointM laying onthe circle C we construct the tangent lines MT 1 and MT 2 to the circles C 1 and C 2 ,respectively (T 1 ∈ C 1 , T 2 ∈ C 2 ). If MA < MA 1 prove that MT 1 + MT 2 = A 1MA 1 D · KLand |MT 1 − MT 2 | = AMAD · KL.Neculai Roman, Mirceşti (Iaşi)L97. Prove that, in any triangle, the following inequality holds1m 2 a (m b + m c − m a ) + 1m 2 b (m c + m a − m b ) 2 + 1m 2 c (m a + m b − m c ) 2 ≥ 1 S 2 .I. V. Maftei and Dorel Băiţan, BucureştiL98. Let ABC be an arbitrary triangle. Prove that83


1) sin 4 A +sin 4 B +sin 4 C ≥ 27 ³ r´3;2 R2) cos 4 A +cos 4 B +cos 4 C ≥ 3 ³ r´4 ³5 − r ´³a5 − r ´³b5 − r ´c,8 R r rwhere R is the radius of the circumcircle circle, r is the radius of the inscribed circleand r a , r b , r c are the radius of the exinscribed circles.Oleg Faynshteyn, Leipzig, GermanyL99. a) Which is the minimal number of points of integer coordinates containedin a plane, such that no mater how they are chosen, there exist three of them withtheir centre of gravity expressed by integer coordinates.b) Prove that, in an n-dimensional space exist 2 n+1 points of integer coordinatessuch that any 3 of them have the center of gravity with at least one coordinate notan integer.Irina Mustaţă, student, Bremen, Germany·1L100. Let x ∈ (0, 1); provethatthereexistn ∈ N ∗ such that {nx} ∈3 , 2 .3(By {·} we denoted the fractional part.)Ciprian Baghiu and Gheorghe Iurea, IaşiL101. Let a, n ≥ 2 two integers. Prove that n−1 Qa n − a kk=0 n − k ∈ Z.Adrian Zahariuc, highschool student, BacăuL102. Let p =2k +1be a prime number. Then2kXS 1 = Cp+i−1 i ≡ 2 p − 2 ¡ mod p 2¢ kX, S 2 = Cp+i−1 i ≡ 2 − 2 p ¡ mod p 2¢ .i=k+1Marius Pachiţariu, highschool student, IaşiL103. Let a, b, c, d some real numbers such that¡1+a2 ¢¡ 1+b 2¢¡ 1+c 2¢¡ 1+d 2¢ =16.Prove that−3 ≤ ab + bc + cd + da + ac + bd − abcd ≤ 5.Moreover, at least one from the above inequalities becomes an equality if and only ifa + b + c + d = abc + bcd + cda + dab.Gabriel Dospinescu, student, ParisL104. Let x 0 > 0 and x n = x [n2 ] + x [ n 3 ] + n , for any n>0.³ 6xn´a) Prove that the sequence is convergent to 1.n n≥15b) Prove that if α>log 3 ,then lim2 n→∞i=1x n − nn α =0.Gabriel Dospinescu, student, ParisL105. Find all continuous functions f :(0, ∞) → R, which verify the functionalequationnx n−1 f (x n )=(x +1)f (x) , ∀x ∈ (0, ∞) ,where n ∈ N ∗ , n is fixed.Marian Tetiva and Dumitru Mihalache, Bârlad84


Pagina rezolvitorilorBRAŞOVLiceul "N. Titulescu". Clasa a X-a. CHIRA Roxana: VIII(56,58,61-63), IX(56,61); COSTA Larisa: VIII(57,58,61-63), IX(56,61); LUCACI Adina: VIII(61-63),IX(61,63); MOCANU Vlad: VIII(61-63), IX(61,63,65); NEGOESCU Anamaria: VIII(56,57,61,62), IX(56,61,63,65); NILĂ Iulian: VIII (61-63), IX(61,63,65); NUŢU Cosmin:VIII(61-63), IX(61,63,65); OBANCEA Dragoş: VIII(61-63), IX(61,63,65);ŞCHIOPU Iulian: VIII(56,61-63), IX(56,61); ZBARCEA Adrian: VIII(56,57,61-63),IX(56,61). ClasaaXI-a. BOTH Alexandru: IX(61,63,65), X.62, XI.61; BRĂGARUAndreea: IX(61,63,65), X.62, XI.61; CAIA Claudiu: IX(61,63,65), X.62, XI.61;DELAST-VOINEA Alexandru: IX(61,63,65), X(58,62), XI.61; DIACONESCU Adrian:IX(61,63,65), X.62, XI.61; DUCA Răzvan: IX(61,63, 65), X.62, XI.61; DU-MITRU Silviu: IX(61,63,65), X.62, XI.61; ENACHE Florin: IX(61,63,65), X.62,XI.61; FLOREA Luminiţa: IX(57,61,63,65), X.62, XI.61; GIURGIU Beniamin: IX(61,63,65), X.62, XI.61; GONŢEA Paul: IX(61,63,65), X.62, XI.61; GULIC Mihai:IX(61,63,65), X.62, XI.61; IVAŞCU Andreea: IX(61,63,65), X.62, XI.61; LUKACSRaluca: IX(61,63,65), X.62, XI.61; MARDALE Mihai: IX(61,63,65), X.62, XI.61;MATIŞ Gheorghe: IX(61,63,65), X.62, XI.61; MÂRZESCU Andreea: IX(61,63,65),X.62, XI.61; NAFRADI Jenö: IX(61,63,65), X.62, XI.61; NAN Maria: IX(61,63,65),X.62, XI.61; PANAIT Mihail: IX(61,63,65), X.62, XI.61; POPA Cătălina: IX(61,63,65), X.62, XI.61; RĂCEAN Bogdan: IX(61,63,65), X.62, XI.61; SAVA Cosmin:IX(61,63,65), X.62, XI.61; TĂBUŞCĂ Florin: IX(61,63,65), X.62, XI.61; TREŞCAAna-Maria: IX(61,63,65), X.62, XI.61.CRAIOVAColegiul Naţional "Carol I". ClasaaVI-a. STANCIU Ioan: V(61-65), VI(61-63), VIII.61, G.90.HÂRLĂUŞcoala "Petru Rareş". Clasa a III-a (înv. BUDACEA Maria). NEICU Mara:P(89,95-97,99). ClasaaIV-a. (înv. CREŢU Maria). BOUTIUC Mădălina: P(94,96-101); BUZILĂ Andreea: P(94-101); PINTILII Alina: P(94-101).Liceul "Ştefan cel Mare". Clasa a VII-a. APACHIŢEI Ana-Maria: V(56-58),VI(59,60); ATÎRGOVIŢOAE Anca-Elena: V(56-58), VI(59,60); COJOCARU Iulia:V(56-58), VI(59,60); CURCĂ Ioana: V(56-58), VI(59,60); LENŢER Sonia: V(56-58), VI(59,60); PINTILII Anda: V(56-58), VI(59,60).IAŞIŞcoala nr. 3 "Al. Vlahuţă". Clasa a III-a (inst. MAXIM Gabriela). CEL-MARE Raluca-Iuliana: P(94-96,98,99); NEAGU Ramona-Mihaela: P(94-96,98,99);POPOVICI Ionuţ: P(94-96,98,99); RUSU Ioana-Andreea: P(94-96,98,99); RUSUMădălina: P(94-96,98,99); SAVA Vlad: P(94-96,98,99). Clasa a III-a (înv. MĂRI-UŢĂ Valentina). CULEA Alina: P(94-96,98,99); POPA Iulian: P(94-96,98,99);PROCA Ancuţa-Ioana: P(94-96,98,99). Clasa a VII-a. DODU Corina: V(61-63), VI.61, VII.63; IRIMIA Andreea: V(61-63), VI.61, VII.63; RUSU Laura-Elena:V(61-63), VI.61, VII.63; UNGURU Claudiu: V(61-63), VI.61, VII.63.85


Şcoala nr. 4 "I. Teodoreanu". Clasa a IV-a (înv. BUJOR Lorena). LUPANDana: P(98-100,102,103).Şcoala nr. 13 "Alexandru cel Bun". Clasa II-a (inst. COJOCARIU Ana). AGA-FIŢEI Elena-Roxana: P(94-97,99); CARAMALĂU Andra: P(94-97,99); CĂLINAndreea-Claudia: P(94-97,99); COJOCARIU Andreea: P(94-97,99); DUDUMANLuisa-Ştefania: P(94-97,99); LELEU Alexandrina-Ştefana: P(94-97,99); LUPAŞCUDiana-Maria: P(94-97,99); MANOLACHE Mădălina-Andreea: P(94-97,99); MI-HĂILĂ Narcisa-Lorena: P(94-97,99); PASCU Gabriela: P(94-97,99); PĂDURARUTiberiu-Ştefan: P(94-97,99); RĂDUCEA Marin-Andrei: P(94-97,99); SAVIN Cristina-Simona:P(94-97,99); ŞTEFAN Bogdan-Vasile: P(94-97,99); ŞTIUBEI Cosmin-Ionuţ: P(94-97,99). Clasa a III-a (înv. OBREJA Rodica): APETRII Alexandru:P(94-97,99); EŞANU Mihai: P(94-97,99); VATAVU Iulian: P(94-97,99).Şcoala nr. 22 "B. P. Hasdeu". ClasaaIV-a(înv. CHIRILĂ Laura). BUHUVlad: P(94-100,102,103); CHICHIRĂU Alexandra-Elena: P(94-100). ClasaaIVa(înv. TÂRZIORU Iuliana). APOSTOL Ana-Maria: P(94-97,99,100); BALANAndrei: P(94-99); GÎNDU Alexandra: P(94-100); GRIEROSU Claudiu: P(94-100);LĂMĂTIC Ioana: P(94-102); REBEGEA Andrada Elena: P(94-100); UNGUREA-NU Teofana: P(94-100).Şcoala nr. 26 "G. Coşbuc". ClasaaIV-a(înv. BUCATARIU Rica). AELENEIGeorge-Ciprian: P(94-97,100); DUMITRU Ambra-Georgiana: P(94-97,99,100); GOL-DAN Alexandru-Iulian: P(94-100); HRISCU Alexandra: P(94-100); IACOB Robert-Ionuţ: P(94-100); MOISA Adrian-Bogdan: P(94-100); MUSTEAŢĂ Alexandra:P(94-98); SCUTARU Ionela-Cristina: P(94-100); TUDOSĂ Mădălina: P(94-99);ZĂLINCĂ Georgiana: P(94-98).Colegiul Naţional. Clasa a VI-a. BACUŞCĂ Albert: V(56-60); BARON Bogdan:V(56-60); BOŢU Alexandru: P.101, V(61-63), VI.61; CEUCĂ Răzvan: V(56-60); MOCANU Dan Mihai: P.103, V(61,63,65), VI.61; PRISTOPAN Codrin: V(56-60); TIBA Bianca Mădălina: V(59-63,65), VI.63; Clasa a VII-a. CADAR Alexandra:V(61-65), VI(61,62); HUMELNICU Roxana: V(61-64), VI(61-65).Colegiul Naţional "Emil Racoviţă". Clasa a VI-a. TUDORACHE Alexandru-Gabriel: P(101-103), V(61-65), VI(61,63,64).Colegiul Naţional "C. Negruzzi". ClasaaVII-a.TIBA Marius: VII(62,64,65),VIII(61,62), G(89,95); Clasa a VIII-a. OLAN Ioana: VI(61,64), VII(63,65), G(89,95).SIBIUColegiul Naţional "Gh. Lazăr". Clasa a XI-a. VLAD Emanuel: L(77-83).SUCEAVAŞcoala nr. 3. ClasaaII-a(inst. NECHITA Daniela). FECHET Mircea: P(84,85,87-89,94-97,99).86


Premii acordate rezolvitorilorASOCIAŢIA "RECREAŢII <strong>MATEMATICE</strong>" în colaborare cu redacţiarevistei RECREAŢII <strong>MATEMATICE</strong> acordă câteodiplomă şi un premiu încărţi pentru trei apariţii la rubrica "Pagina rezolvitorilor" elevilor următori:Liceul "N. Titulescu", BotoşaniNILĂ Iulian (cl. a X-a): 1/2005(6pb); 2/2005(8pb); 1/2006(6pb);NUŢU Cosmin (cl. a X-a): 1/2005(7pb); 2/2005(6pb); 1/2006(6pb);OBANCEA Dragoş (cl. a X-a): 1/2005(5pb); 2/2005(8pb); 1/2006(6pb).Colegiul N¸aional "Carol I", CraiovaSTANCIU Ioan (cl. a VI-a): 1/2005(10pb); 2/2005(7pb); 1/2006(10pb).Şcoala "P. Rareş", HârlăuNEICU Mara (cl. a III-a): 1/2005(11pb); 2/2005(5pb); 1/2006(5pb).Şcoala nr. 3 "Al. Vlahuţă", IaşiDODU Corina (cl. a VII-a): 2/2004(5pb); 2/2005(6pb); 1/2006(5pb);IRIMIA Andreea (cl. a VII-a): 2/2004(5pb); 2/2005(5pb); 1/2006(5pb);UNGURU Claudiu (cl. a VII-a): 2/2004(5pb); 2/2005(5pb); 1/2006(5pb).Şcoala nr. 4 "I. Teodoreanu", IaşiLUPAN Dana (cl. a IV-a): 1/2005(9pb); 2/2005(6pb); 1/2006(5pb).Şcoala nr. 22 "B. P. Hasdeu", IaşiBUHU Vlad (cl. a IV-a): 1/2005(5pb); 2/2005(7pb); 1/2006(9pb);CHICHIRĂU Alexandra-Elena (cl. a IV-a): 1/2005(5pb); 2/2005(5pb);1/2006(7pb);LĂMĂTIC Ioana (cl. a IV-a): 1/2005(8pb); 2/2005(10pb); 1/2006(9pb);REBEGEA Andrada Elena (cl. a IV-a): 1/2005(6pb); 2/2005(7pb); 1/2006(7pb);Şcoala nr. 26 "G. Coşbuc", IaşiDUMITRU Ambra-Georgiana (cl. a IV-a): 1/2005(8pb); 2/2005(5pb);1/2006(6pb);HRISCU Alexandra (cl. a IV-a): 1/2005(6pb); 2/2005(7pb); 1/2006(7pb);MUSTEAŢĂ Alexandra (cl. a IV-a): 1/2005(6pb); 2/2005(6pb); 1/2006(5pb);ZĂLINCĂ Georgiana(cl. a IV-a): 1/2005(6pb); 2/2005(5pb); 1/2006(5pb).Colegiul Naţional, IaşiCADAR Alexandra (cl. a VII-a): 1/2005(8pb); 2/2005(9pb); 1/2006(7pb).Colegiul Naţional "Emil Racoviţă", IaşiTUDORACHE Alexandru-Gabriel (cl. a VI-a): 1/2005(9pb); 2/2005(8pb);1/2006(12pb).87


ASOCIAŢIA “RECREAŢII <strong>MATEMATICE</strong>”La data de 14.02.2005 a luat fiinţă ASOCIAŢIA “RECREAŢII MATE-MATICE”, cusediulînIaşi (str. Aurora, nr. 3, sc. D, ap. 6), având ca scop sprijinireaactivităţilor de matematică specifice învăţământului preuniversitar, organizareaşi desfăşurarea de activităţi care să contribuie la dezvoltarea gustului pentrumatematică în rândurile elevilor, profesorilor şi iubitorilor de matematică şi stimulareapreocupărilor şi cercetătorilor originale.Obiectivele majore pentru atingerea scopului propus sunt:1. editarea unei reviste destinată elevilor şi profesorilor — revista "RecreaţiiMatematice";2. fondarea unei biblioteci de matematică elementară—biblioteca "RecreaţiiMatematice";3. alcătuirea unei colecţii de cărţi de matematică elementară, cărţi de referinţăşi aflate la prima apariţie — Colecţia "Recreaţii Matematice".Poate deveni membru al Asociaţiei, printr-o simplă completare a unei cerei tip,orice perosană care aderă laobiectiveleacesteiaşi sprijină realizarealor.Membri de onoare, academicienii:Constantin CorduneanuRadu MironContinuarea listei membrilor din RecMat - 2/2005:Numele şi prenumele Locul de muncă Data înscrierii21 Dumitru Neagu Lic. "G. Ibrăileanu", Iaşi 10.03.200522 Tudorache Rodica Univ. Tehn. "Gh. Asachi", Iaşi 14.03.200523 Tiba I. Dan Inst. de Mat. al Acad. Rom., Bucureşti 20.03.200524 Popovici Mihaela S. C. Selgros Cash&Carry, Braşov 20.03.200525 Popovici Florin Lic. "N. Titulescu", Braşov 20.03.200526 Negrescu Alexandru C. N. "A.T. Laurian", Botoşani (elev) 25.03.200527 Şerdean Ioan C. N. "Aurel Vlaicu", Orăştie 28.03.200528 Marinescu Dan-Ştefan C. N. "Iancu de Hunedoara", Huned. 28.03.200529 Popa Vasile C. N. "V. Alecsandri", Galaţi 29.03.200530 Marin Mirela Şc. "Al. Vlahuţă", Iaşi 12.04.200531 Maxim Gabriela Şc. "Al. Vlahuţă", Iaşi 14.04.200532 Bejan Cornelia - Livia Univ. Tehn. "Gh. Asachi", Iaşi 25.04.200533 Brînzei Dan Univ. "Al. I. Cuza", Iaşi 29.04.200534 Teodoru Georgeta Univ. Tehn. "Gh. Asachi", Iaşi 09.05.200535 Roman Neculai Şc. "V. Alecsandri", Mirceşti (Iaşi) 11.05.200536 Borş Dan Mircea Univ. Tehn. "Gh. Asachi", Iaşi 13.05.200537 Haivas Mihai Inst. Cerc. Ec. "Gh. Zane", Iaşi 18.05.200538 Popa Claudiu - Ştefan Şc. "Alecu Russo", Iaşi 27.05.200539 Minuţ Petru Univ. "D. Cantemir", Tg. Mureş 30.06.200588


<strong>Revista</strong> semestrială RECREAŢII <strong>MATEMATICE</strong> este editată deASOCIAŢIA “RECREAŢII <strong>MATEMATICE</strong>”. Apare la datele de 1 martie şi1 septembrie şi se adresează elevilor, profesorilor, studenţilor şi tuturor celorpasionaţi de matematica elementară.În atenţia tuturor colaboratorilorMaterialele trimise redacţiei spre publicare (note şi articole, chestiuni demetodică, probleme propuse etc.) trebuie prezentate îngrijit, clar şi concis; eletrebuie să prezinte interes pentru un cerc cât mai larg de cititori. Se recomandă catextele să nu depăşească patru pagini. Evident, ele trebuie să fie originale şi sănu fi apărut sau să fi fost trimise spre publicare altor reviste. Rugăm ca materialeletehnoredactate să fie însoţite de fişierele lor.Problemele destinate rubricilor: Probleme propuse şi Probleme pentrupregătirea concursurilor vor fi redactate pe foi separate cu enunţ şi demonstraţie/rezolvare(câte una pe fiecare foaie) şi vor fi însoţite de numele autorului, şcoalaşi localitatea unde lucrează/învaţă.Redacţia va decide asupra oportunităţii publicării materialelor primite.În atenţia elevilorNumele elevilor ce vor trimite redacţiei soluţii corecte la problemele dinrubricile de Probleme propuse şi Probleme pentru pregatirea concursurilorvor fi menţionate în Pagina rezolvitorilor. Se va ţine seama de regulile:1. Pot trimite soluţii la minimum cinci probleme propuse în numărulprezent şi cel anterior al revistei; pe o foaie va fi redactată soluţia unei singureprobleme.2. Elevii din clasele VI-XII au dreptul să trimită soluţii la problemelepropuse pentru clasa lor, pentru orice clasă mai mare, din două clase mai mici şiimediat anterioare. Elevii din clasa a V-a pot trimite soluţii la problemele propusepentru clasele a IV-a, a V-a şi orice clasă mai mare, iar elevii claselor I-IV pottrimite soluţii la problemele propuse pentru oricare din clasele primare şi orice clasămai mare. Orice elev poate trimite soluţii la problemele de concurs (tip G şi L).3. Vor fi menţionate următoarele date personale: numele şi prenumele,clasa, şcoala şi localitatea.4. Plicul cu probleme rezolvate se va trimite prin poştă (sau va fi adusdirect) la adresa Redacţiei:Prof. dr. Temistocle BîrsanStr. Aurora, nr. 3, sc. D, ap. 6,700 474, IaşiJud. IAŞIE-mail: tbirsan@math.tuiasi.ro


CUPRINSElogiu adus revistei “Gazeta Matematică” la 110 ani de apariţie neîntreruptă.................. 1100 de ani de la naşterea matematicianului Grigore C. Moisil ............................................. 2ARTICOLE ŞI NOTED. M. BĂTINEŢU-GIURGIU – Asupra problemei 809 din Gazeta Matematică,volumul VIII (1902–1903)................................................................. 5G. DOSPINESCU – Câteva proprietăţi ale subgrupurilor finite din GLn( )................... 8T. BÎRSAN – Ceviene şi triunghiuri triomologice............................................................... 13A. MOSCALIUC – Construcţii aproximative cu rigla şi compasul ale numărului π ...... 16I. V. MAFTEI – Inegalităţi generatoare de noi inegalităţi .................................................. 19C.-Şt. POPA – Asupra unei probleme dată la ONM, Bistriţa, 2005................................. 21NOTA ELEVULUIM. TIBA – Asupra criteriului de congruenţă LLU.............................................................. 23I. OLAN – O generalizare a identităţii Botez - Catalan ...................................................... 25M. PACHIŢARIU – Acoperiri ale planului laticial cu figuri............................................. 26CHESTIUNI METODICEM. TETIVA – Metoda normării............................................................................................. 30Gh. IUREA – Asupra unei recurenţe de ordin doi ............................................................. 35CORESPONDENŢEOlimpiada Internaţională de Matematică “B. O. Zhautykov” ............................................... 37PROBLEME ŞI SOLUŢIISoluţiile problemelor propuse în nr. 1/2005.......................................................................... 43Soluţiile problemelor pentru pregătirea concursurilor din nr. 1/2005 ................................. 61Probleme propuse..................................................................................................................... 75Probleme pentru pregătirea concursurilor .............................................................................. 80Training problems for mathematical contests ....................................................................... 82Pagina rezolvitorilor .............................................................................................................. 85ASOCIAŢIA “RECREAŢII <strong>MATEMATICE</strong>” ...................................................................... 885 lei

Hooray! Your file is uploaded and ready to be published.

Saved successfully!

Ooh no, something went wrong!